Sei sulla pagina 1di 282

PRESENTACION

El Instituto de Ciencias y Humanidades (ICH) est a orientado al ser-


vicio educativo y a la formaci on integral de sus estudiantes mediante el desarrollo
de sus potencialidades. Todos los esfuerzos de nuestra instituci on se centran en la
ensenanza, formacion e investigaci on en matem atica, ciencias naturales, ciencias
sociales y las diversas expresiones del arte.
La matematica tiene un rol protag onico en el desarrollo de las ciencias y la
tecnologa. Su contribucion en las investigaciones de otras ciencias es invalorable.
As por ejemplo, fsicos, qumicos, bi
ologos, sociologos, antropologos, por citar solo
algunos ejemplos, hacen uso en sus trabajos de teoras y herramientas matematicas.
Desde esta perspectiva, desarrollamos una poltica de fomento a la investigacion de
nuestros docentes que tiene como resultado no s olo la elevaci
on del servicio educativo
en las aulas sino tambien la publicaci on de textos academicos como el presente.
Mediante estas lneas, presentamos a la comunidad educativa del pas el libro
Desigualdades, olimpiadas matem aticas, obra del matem atico Victor Hugo
Laurente Artola, docente de nuestra instituci on.
Diferentes delegaciones de estudiantes peruanos han representado al pas en
olimpiadas internacionales de matem aticas y han obtenido valiosos resultados que no
hacen sino renovar nuestra confianza en el gran potencial de nuestros j ovenes y en
su interes por el estudio. Asimismo, se corrobora el trabajo denodado de los docentes,
quienes a pesar de las adversidades participan en cursos, talleres, capacitaciones y
otras jornadas academicas a fin de elevar su dominio de la matem atica.
El Instituto de Ciencias y Humanidades reafirma su compromiso con el trabajo
educativo y la promocion de esfuerzos de investigaci on individuales y colectivos a fin
de que se materialicen sus metas institucionales.

Instituto de Ciencias y Humanidades

i
ii

INTRODUCCION

Cuando a fines de la decada del 90 empezamos, con un grupo de colegas del


Instituto de Ciencias y Humanidades, a colaborar en la ense nanza de jovenes que
representaran al Per
u en las olimpiadas internacionales de matematica, nos topamos
con un grave problema: la falta de bibliografa sobre el tema. Con los a nos, fuimos
acumulando experiencias a partir de la consulta de textos que los propios estudian-
tes traan de sus viajes al extranjero, de algunas publicaciones que ya se podan
encontrar en nuestro medio y de la ense nanza en las aulas.
De este modo, fue germinando en mi persona el interes por realizar una pu-
blicacion de mi especialidad, en particular sobre el tema de desigualdades. As, en
diciembre de 2007 pude iniciar el proyecto que despues de diez meses se materializa
en la presente publicacion titulada Desigualdades, olimpiadas matem aticas.
No hay experiencia comparada a la adquirida por los estudiantes que participan
en las olimpiadas. Su participacion en olimpiadas, permite que los estudiantes ga-
nen mayor conocimiento, que se formen como futuros cientficos y que establezcan
relaciones de colaboracion academica con otras personas e instituciones del extran-
jero. Las pruebas de olimpiadas integran problemas de gran nivel analtico que co-
rresponden a temas propios del algebra, matematica discreta, teora de n umeros y
geometra.
Aparte del talento, quienes participan en olimpiadas requieren dedicacion, per-
severancia y un equipo de profesionales que los ayude en su preparacion. Se trata de
una practica que debera recorrer toda la vida escolar. Es necesario, desde la escuela,
fomentar la organizacion de talleres de matematica, la formacion de equipos de es-
tudiantes que participen en concursos a nivel local, regional y nacional. El presente
texto pretende contribuir con este proposito.
El texto desarrolla el tema de desigualdades, uno de los topicos fundamentales
del algebra. Recoge un conjunto de elementos teoricos y una serie de ejercicios or-
ganizados de lo simple a lo complejo. Asimismo, se han incorporado los problemas

iii
iv

de las pruebas de las olimpiadas internacionales de matematica (IMO, por sus siglas
en ingles) desde el a
no 59 al a
no 2008. En algunos casos, los problemas estan acom-
panados de mas de una solucion de modo que el lector conozca una mayor diversidad
de estrategias para encarar los ejercicios.
Para la elaboracion del texto he utilizado como fuente libros editados en Estados
Unidos, Europa y Asia. Asimismo, he recurrido a valiosa informacion de Internet y he
sistematizado la experiencia adquirida en la ensenanza de estudiantes que participan
en olimpiadas matematicas.
Es recomendable, para abordar el texto, que el lector tenga conocimientos basicos
de desigualdades, de manera que pueda profundizar con mayor facilidad en los temas
propuestos. Es importante que estudie con detenimiento la teora, revise las aplica-
ciones y luego intente resolver los problemas antes de recurrir a la solucion.
Quiero hacer p ublico mi agradecimiento al Instituto de Ciencias y Humanidades
(ICH) por su apoyo en la investigacion y edicion del texto. A Jorge Tipe Villanueva,
por sus importantes sugerencias, a Juan Neyra Faustino, por su colaboracion en el
digitado en LATEXdel material y, en especial, a mi esposa Magda, a mi hijo Taylor y
a mis padres Francisco y Serafina por su constante y siempre desinteresado apoyo.
A todos ellos, mi profundo agradecimiento.

Victor Hugo Laurente Artola


Lima, setiembre de 2008
INDICE GENERAL

1. Numeros Reales 1
1.1. Cuerpos . . . . . . . . . . . . . . . . . . . . . . . . . . . . . . . . . . 1
1.1.1. Axiomas de adicion. . . . . . . . . . . . . . . . . . . . . . . . 1
1.1.2. Axiomas de multiplicacion. . . . . . . . . . . . . . . . . . . . . 2
1.2. Cuerpos Ordenados . . . . . . . . . . . . . . . . . . . . . . . . . . . . 3
1.3. Aplicaciones . . . . . . . . . . . . . . . . . . . . . . . . . . . . . . . . 13

2. Valor Absoluto y M aximo Entero 25


2.1. Valor Absoluto . . . . . . . . . . . . . . . . . . . . . . . . . . . . . . 25
2.2. Maximo Entero o Parte Entera . . . . . . . . . . . . . . . . . . . . . 29
2.3. Aplicaciones . . . . . . . . . . . . . . . . . . . . . . . . . . . . . . . . 32

3. Desigualdad de Cauchy-Schwarz 37
3.1. El Lema de Titu . . . . . . . . . . . . . . . . . . . . . . . . . . . . . 39
3.2. Desigualdad de Sch ur . . . . . . . . . . . . . . . . . . . . . . . . . . . 41
3.3. Dos sustituciones muy u tiles . . . . . . . . . . . . . . . . . . . . . . . 41
3.4. Aplicaciones . . . . . . . . . . . . . . . . . . . . . . . . . . . . . . . . 43

4. Desigualdad de la Media Aritm etica - Media Geom etrica 55


4.1. Aplicaciones . . . . . . . . . . . . . . . . . . . . . . . . . . . . . . . . 58

5. Desigualdad de Bernoulli y la Media Potencial 69


5.1. Media Potencial . . . . . . . . . . . . . . . . . . . . . . . . . . . . . . 72
5.2. Aplicaciones . . . . . . . . . . . . . . . . . . . . . . . . . . . . . . . . 76

6. Desigualdad de Reordenamientos 79

v
vi INDICE GENERAL

7. Desigualdad con Funciones Convexas 89


7.1. Funcion convexa . . . . . . . . . . . . . . . . . . . . . . . . . . . . . . 89

8. Espacio M etrico 97
8.1. Desigualdad de Holder . . . . . . . . . . . . . . . . . . . . . . . . . . 102
8.2. Desigualdad de Minkonski . . . . . . . . . . . . . . . . . . . . . . . . 103

9. M
etodo de la Suma de Cuadrados 109

10.Desigualdades Sim etricas y Cclicas 117


10.1. Polinomios simetricos y cclicos . . . . . . . . . . . . . . . . . . . . . 123

11.Problemas 129

12.Soluciones 149

Bibliografa 275
CAPITULO 1

NUMEROS REALES

1.1. Cuerpos
Un cuerpo es un conjunto K con dos operaciones llamadas adicion y multipli-
cacion, que satisfacen ciertas condiciones llamadas axiomas de cuerpo.

1.1.1. Axiomas de adici


on.
A1 . Clausura.

x + y K, x, y K.

A2 . Conmutatividad.

x + y = y + x, x, y K.

A3 . Asociatividad.

(x + y) + z = x + (y + z), x, y, z K.

A4 . Elemento neutro.
Existe 0 K tal que

x + 0 = x, x K.

1
2 1. N
umeros Reales

A5 . Simetrico.
Para todo x K existe (x) K tal que

x + (x) = 0.

De la conmutatividad, se sigue que

0+x=x+0=x y (x) + x = x + (x) = 0.

A la suma x + (y) se denotara x y y se denomina la diferencia entre x e y.


La operacion se llama sustraccion.

1.1.2. Axiomas de multiplicaci


on.
M1 . Clausura.

x y K, x, y K.

M2 . Conmutatividad.

x y = y x, x, y K.

M3 . Asociatividad.

(x y)z = x(y z), x, y, z K.

M4 . Elemento neutro.
Existe 1 K tal que 1 6= 0 y

x 1 = x, x K.

M5 . Simetrico.
Para todo x 6= 0 K existe el inverso de x denotado por x1 K tal que

x x1 = 1.
1.2. Cuerpos Ordenados 3

De la conmutatividad se sigue que

1x = x1 =x y x1 x = x x1 = 1.
x
umero x y 1 se denota
Dados x, y K; con y 6= 0; el n .
y
Estos axiomas estan relacionados con un axioma llamado distributividad.

D1 . Axioma de distributividad.

x (y + z) = x y + x z, x, y, z K.

Ejemplos de cuerpos
m
1. Q = { | m Z; n Z+ }.
n
2. R = Q Q ; Q : conjunto de los irracionales.

3. Z2 = {0; 1}; veamos que este conjunto es un cuerpo de solo 2 elementos.

0+1= 1 ; 0+0= 0 ; 00 =0 ; 10 =0
1+0= 1 ; 1+1= 0 ; 01 =0 ; 11 =1

1.2. Cuerpos Ordenados


Un cuerpo K es ordenado; si tiene un subconjunto distinguido P K, llamado
el conjunto de elementos positivos de K que satisfacen las siguientes condiciones

P1 . x + y P y x y P, x, y P.

P2 . Dado x K, exactamente ocurre una de las tres condiciones

x = 0 x P x P.

Indicaremos con P al conjunto de elementos x, donde x P. De donde

K = P (P) {0}.

Estos conjuntos son disjuntos y P se llama conjunto de n


umeros negativos.
4 1. N
umeros Reales

En todo cuerpo ordenado si a 6= 0; entonces a2 P; en efecto, siendo a 6= 0


entonces a P a P; en el primer caso

a2 = a a P;

en el segundo caso

a2 = (a) (a) P.

En un cuerpo K: x < y se lee x es menor que y, que equivale a decir y x P.


De donde
y > 0 equivale a decir y P.
x < 0 equivale a decir x P;
x < y y x > 0;
x < y y > x.
La relacion de orden x < y en un conjunto ordenado K goza de los siguientes
teoremas
Teorema 1.1 (Transitividad)

x < y y < z x < z.

 Prueba. En efecto

x < y y x > 0;
y < z z y > 0;

significa que (y x); (z y) P, entonces

(y x) + (z y) P
(z x) P
zx>0
x < z.
2
Teorema 1.2 (Tricotoma)
Dados x, y K, solo se cumple una de las tres condiciones

x = y x < y x > y.
1.2. Cuerpos Ordenados 5

 Prueba. En efecto

x, y K, y x = 0 y x P y x P,

equivale a

y = x x < y (y x) P
xy P
y<x
x>y

y = x x < y x > y.
2
Teorema 1.3 (Monotonicidad de la adici
on)

x < y x + z < y + z, z K.

 Prueba. En efecto

x<y
yx> 0
(y + z) (x + z) > 0
x + z < y + z.
2
Teorema 1.4 (Monotonicidad de la multiplicaci
on)

x < y z > 0 xz < yz,


x < y z < 0 xz > yz.

 Prueba. En efecto

x<y z>0
yx>0 z >0
(y x); z P
6 1. N
umeros Reales

entonces

(y x) z P
yz xz P
yz xz > 0
xz < yz.

Analogamente

x<y z <0 yx>0 z <0


(y x); z P
entonces

(y x) (z) P
yz + xz P
xz yz > 0
xz > yz.
2
Teorema 1.5
x < y a < b x + a < y + b.

 Prueba. En efecto

x < y x + a < y + a, (Teorema 1.3)


a < b y + a < y + b, (Teorema 1.3)

entonces por el teorema 1.1 x + a < y + b. 2


Teorema 1.6
x < y a > b x a < y b.
 Prueba. En efecto, a > b a < b, y como x < y entonces por el teorema
1.5 se tiene que

x + (a) < y + (b)


x a < y b.
2
1.2. Cuerpos Ordenados 7

Teorema 1.7
0 < x < y 0 < a < b xa < yb.

 Prueba. En efecto

x < y xa < ya; a > 0 (teorema 1.4)


a < b ya < yb; y > 0 (teorema 1.4)

por el teorema 1.1 se tiene xa < yb. 2

Teorema 1.8

1
x>0 > 0.
x

 Prueba. En efecto

() Como x x1 = 1 > 0 y x > 0 entonces x1 > 0.


 
1 1
() Como > 0 entonces x2 > 0, de donde x > 0.
x x
2
Teorema 1.9
xy > 0 [(x > 0 y > 0) (x < 0 y < 0)].

 Prueba. En efecto

() Sea x > 0 entonces x1 > 0.


Como xy > 0 entonces x1 (xy) > 0, efectuando tenemos y > 0.
Similarmente si x < 0 tenemos y < 0.

() Si x > 0 y > 0 xy > 0.


Similarmente si x < 0 y < 0 x > 0 y > 0, entonces (x)(y) >
0, de donde xy > 0.

2
Teorema 1.10
xy < 0 (x < 0 y > 0) (x > 0 y < 0).
8 1. N
umeros Reales

 Prueba. La demostracion es analoga al Teorema 1.9. 2


Teorema 1.11
1 1
Si ab > 0 entonces a<b > .
a b
 Prueba.

() a < b.
Como ab > 0 entonces tenemos dos casos:
1 1
i. Si a > 0 b > 0 entonces >0 > 0, luego
a b
1 1 ba
= > 0, pues a < b
a b ab
1 1
>0
a b
1 1
> .
a b
ii. Para a < 0 b < 0 el procedimiento es analogo.
   
1 1 1 1
() Como > entonces ab > ab, efectuando tenemos b > a, de donde
a b a b
a < b.

2
Teorema 1.12
En un cuerpo ordenado K, las siguientes afirmaciones son equivalentes:

(i) N K no es acotado superiormente.


(ii) Dados a, b K, con a > 0, existe n N tal que na > b.
1
(iii) Dado cualquier a > 0 en K, con a > 0, existe n N tal que 0 < < a.
n
 Prueba. (i) (ii). Como N no es acotado superiormente, entonces dados
b
a > 0 y b en K, existe n N tal que < n, de donde b < na.
a
1
Veamos (ii) (iii), dado a > 0, existe n N tal que na > 1, de donde < a;
n
1
luego tenemos 0 < < a.
n
1.2. Cuerpos Ordenados 9

ltimo veamos (iii) (i).


Por u
1 1
De (iii) dado cualquier b > 0 existe n N tal que < , de donde se tiene
n b
b < n, es decir n > b.
Esto significa que ning
un positivo de K es cota superior de N. 2

Definicion 1.1: Un cuerpo ordenado K es arquimediano cuando se cumple cualquiera


de las tres condiciones del teorema anterior.
Ejemplo 1.1
El cuerpo de los n
umeros racionales es arquimediano.

on 1.2 : Sea K un cuerpo ordenado y X K un subconjunto acotado


Definici
superiormente. Un elemento x0 K se denominara supremo de X cuando x0 es la
menor de las cotas superiores de X en K.

Para que x0 sea supremo de X K es necesario y suficiente que sean satisfechas


las siguientes condiciones:

(i) x x0 , x X.

(ii) Si y K/x y x X, entonces x0 y.

As mismo se denota sup (X) = x0 , y es u


nico.
Similarmente un elemento y0 K se denomina nfimo de Y K, acotado infe-
riormente, cuando y0 es la mayor de todas las cotas inferiores de Y en K.
Para que y0 sea nfimo de Y K es necesario y suficiente que sean satisfechas
las siguientes condiciones:

(i) y Y ; y y0 .

(ii) Si z K/z y y Y , entonces z y0 .

Se denota nf (Y ) = y0 y a la vez es u
nico.

on 1.3 Sea K un cuerpo ordenado y sea X K con X 6= , un elemento


Definici
x0 X, es el maximo de X si y solo si x x0 , para todo x X.

Definici on 1.4 Sea K un cuerpo ordenado y sea Y K con Y 6= , un elemento


y0 Y , es el mnimo de Y si y solo si y y0 , para todo y Y .
10 1. N
umeros Reales

Ejemplo 1.2
Sean A = [3, 5], B = h3, 5i, entonces

max (A) = 5; sup (A) = 5


mn (A) = 3; nf (A) = 3
mn (B) no existe; sup (B) = 5
max (B) no existe; nf (B) = 3.

En efecto, veamos por que el supremo de B es 5.

 Prueba. Supongan que sup (B) no es 5; sea sup (B) = x0 , significa que x0 < 5:
x0 + 5 x0 + 5
Como x0 < < 5, entonces m = B, de donde
2 2
x0 < m; (1.1)
pero x x0 , x B; en particular haciendo x = m, tenemos

m x0 ; (1.2)
de (1.1) y (1.2), tenemos

m x0 < m

lo cual es una contradiccion.

sup (B) = 5.

Similarmente, pruebe que el nf (B) es 3.

Definici
on 1.5 : Un cuerpo ordenado K se denomina completo cuando un subcon-
junto no vaco acotado superiormente, X K, posee supremo en K.
De la definicion resulta que en un cuerpo ordenado completo todo subconjunto
no vaco acotado inferiormente, Y K, posee nfimo; en efecto, dado Y sea X =
Y = {y/ y Y } = 6 , es acotado superiormente, entonces existe sup (X) = x0 ;
de donde nf (Y ) = x0 .

Axioma. Existe un cuerpo ordenado completo R, denominado el cuerpo de los


n
umeros reales.
1.2. Cuerpos Ordenados 11

Lema 1.1 (de Pit agoras)


No existe un n
umero racional cuyo cuadrado sea 2.
 2
p
 Prueba. Supongamos que existe, es decir = 2, p, q Z, primos entre s.
q

p2 = 2q 2 ; entonces p es par, luego p = 2k, reemplazando


(2k)2 = 2q 2 ;
2k 2 = q 2 ; tambien q es par, como p y q son primos entre s,
hemos llegado a una contradiccion.

Por lo tanto, no existe un racional con esas condiciones, es decir, 2 no es
racional. 2
Los numeros reales que no son racionales se denominan irracionales, luego el
conjunto de los numeros irracionales se denota Q , donde Q = R Q; como 2 no
es racional, entonces es un n
umero irracional. [1]
De aqu en adelante el cuerpo donde vamos a desarrollar la teora es el cuerpo
de los n
umeros reales.
Emplearemos las siguientes notaciones: R+ , es el conjunto de los numeros reales
positivos, R , es el conjunto de los numeros reales negativos, R+0 , es el conjunto

de los n
umeros reales no negativos, y R0 , es el conjunto de los n umeros reales no
positivos.
Teorema 1.13
Sean a, b n
umeros reales, entonces:
(i) a2 + b2 2ab,
(ii) 4ab (a + b)2 .
 Prueba.
(i) Sean a, b R, entonces a b R.
Luego

(a b)2 0
a2 2ab + b2 0
a2 + b2 2ab.

La igualdad ocurre si y solo si a = b.


12 1. N
umeros Reales

(ii) De (i)

a2 + b2 2ab
a2 + 2ab + b2 4ab
(a + b)2 4ab
4ab (a + b)2 .

La igualdad ocurre si y solo si a = b. 2


Teorema 1.14
Sean x, y n
umeros reales, tales que x > 0, y < 0, entonces
1
(i) x + 2,
x
1
(ii) y + 2.
y
 Prueba.
1
(i) En el teorema anterior, haciendo a2 = x, b2 = , vemos que x > 0 y a = x,
x
1
b = , entonces
x
1 1 1
x+ 2 x x + 2.
x x x

La igualdad ocurre si y solo si x = 1.


(ii) Si y < 0 y > 0.
Aplicando (i)

1
(y) + 2;
(y)
1
y 2; multiplicando por (1)
y
1
y+ 2.
y

La igualdad ocurre si y solo si y = 1.


2
1.3. Aplicaciones 13

1.3. Aplicaciones
1. Sean a, b n
umeros reales no nulos, determine el mnimo valor de

a8 a4 a2 b8 b4 b2
+ + + + + .
b8 b4 b2 a8 a4 a2

Soluci
on.

     
a8 a4 a2 b8 b4 b2 a8 b8 a4 b4 a2 b2
+ + + + + = + + + + +
b8 b4 b2 a8 a4 a2 b8 a8 b4 a4 b2 a2
2 + 2 + 2 = 6.

La igualdad ocurre si y solo si a = b. 2

umeros reales tales que a b, x y, pruebe que


2. Sean a, b, x, y n

ax + by ay + bx.

 Prueba. Como a b a b 0, luego multiplicando por (a b) en


la desigualdad x y tenemos:

(a b)x (a b)y
ax bx ay by
ax + by ay + bx.

3. Dados a, b, c n
umeros reales, pruebe que

a2 + b2 + c2 ab + ac + bc.
14 1. N
umeros Reales

 Prueba. Aplicando un resultado conocido:

a2 + b2 2ab; similarmente
b2 + c2 2bc;
c2 + a2 2ca.

Sumando miembro a miembro:

2(a2 + b2 + c2 ) 2ab + 2bc + 2ca,


a2 + b2 + c2 ab + bc + ca.

La igualdad ocurre si y solo si a = b = c. 2

4. Sean a, b, c n
umeros reales positivos, pruebe que

(a + b)(a + c)(b + c) 8abc.

 Prueba.
a, b, c R+ a, b, b R+ ,

luego

( a b)2 0

a 2 ab + b 0

a + b 2 ab, similarmente

b + c 2 bc,

c + a 2 ca,

multiplicando miembro a miembro:



(a + b)(b + c)(c + a) 8( abc)2 = 8abc

de donde (a + b)(b + c)(c + a) 8abc.


La igualdad ocurre si y solo si a = b = c. 2
1.3. Aplicaciones 15

5. Sean a, b n
umeros reales positivos, pruebe que

a2 b2
+ a + b.
b a

 Prueba. La desigualdad es equivalente a:

a3 + b3
a+b
ab
a3 + b3 ab(a + b)
(a + b)(a2 ab + b2 ) ab(a + b); a + b > 0
a2 ab + b2 ab
a2 2ab + b2 0
(a b)2 0.

La igualdad ocurre si y solo si a = b = c. 2

6. Sean a, b, c n
umeros reales positivos, pruebe que

a2 + b2 b2 + c2 c2 + a2
+ + 2(a + b + c).
c a b

 Prueba. Del ejercicio anterior tenemos:

a2 b2
+ a + b; similarmente
b a
b2 c2
+ b + c;
c b
c2 a2
+ a + c;
a c

sumando miembro a miembro:

a2 + b2 b2 + c2 c2 + a2
+ + 2(a + b + c).
c a b

La igualdad ocurre si y solo si a = b = c. 2


16 1. N
umeros Reales

7. Sean a, b, c n
umeros reales no negativos, donde dos de ellos simultaneamente
son diferentes de cero, pruebe que

a2 b2 c2
+ + 1.
a2 + ab + b2 b2 + bc + c2 c2 + ca + a2

 Prueba. Sean a2 + ab+ b2 = m, b2 + bc + c2 = n, c2 + ca+ a2 = p, entonces


debemos probar que

a2 b2 c2
+ + 1
m n p
a2 b2 c2
+ + 1 0
m n p
1 1 1 a2 b2 c2
( + + )( + + 1) 0
m n p m n p
2 2 2 2
a b c b c2 a2 c2 a2 b2 1 1 1
2
+ 2
+ 2
+ + + + + + 0
m n p mn mp mn np mp np m n p
2 2 2
a b c (a2 + b2 ) (a2 + c2 ) (b2 + c2 ) 1 1 1
+ + + + + 0
m2 n2 p2 mn mp np m n p
a2 b2 c2 ab (a2 + ab + b2 ) ac (a2 + ac + c2 ) bc
2
+ 2
+ 2
+ +
m n p mn mn mp mp np
2 2
(b + bc + c ) 1 1 1
+ 0
np m n p
a2 b2 c2 ab ac bc m p n 1 1 1
2
+ 2
+ 2
+ + + 0
m n p mn np np mn mp np m n p
 a 2  b 2  c 2  a   b   a   c   b   c 
+ + + + ,
m n p m n m p n p

esta desigualdad es verdadera (ver aplicacion 3).


La igualdad ocurre si y solo si a = b = c. 2

8. Sea z un n
umero real positivo, pruebe que
r
8z 2
+ 3.
z+1 z+1
1.3. Aplicaciones 17

 Prueba. La desigualdad es equivalente a:


r
8z 2
3
z+1 z+1
r
8z 3z + 1

z+1 z+1
8z (3z + 1)2
, pues z > 0
z+1 (z + 1)2
8z(z + 1) (3z + 1)2
8z 2 + 8z 9z 2 + 6z + 1
0 z 2 2z + 1
0 (z 1)2
(z 1)2 0.
2

9. Sean x, y n
umeros reales, pruebe que

(1 + x2 )(1 + y 2) (1 + xy)2 .

 Prueba. Sabemos que

x2 + y 2 2xy, x, y R
1 + x2 + y 2 + x2 y 2 1 + 2xy + x2 y 2
(1 + x2 )(1 + y 2 ) (1 + xy)2 .

La igualdad ocurre si y solo si x = y. 2

10. Pruebe que

(ax + by)2 (a2 + b2 )(x2 + y 2)

para todo x, y, a, b n
umeros reales.
18 1. N
umeros Reales

 Prueba. Como (ay bx)2 0, x, y, a, b R, luego equivale a:

a2 y 2 2abxy + b2 x2 0
a2 y 2 + b2 x2 2abxy
a2 x2 + b2 y 2 + a2 y 2 + b2 x2 a2 x2 + b2 y 2 + 2abxy
a2 (x2 + y 2) + b2 (y 2 + x2 ) (ax)2 + (by)2 + 2(ax)(by)
(a2 + b2 )(x2 + y 2) (ax + by)2

(ax + by)2 (a2 + b2 )(x2 + y 2).

La igualdad ocurre si y solo si ay = bx. 2

x2 + y 2 + 2
11. Sea f (x, y) = p , donde x, y son n
umeros reales, pruebe que
(1 + x2 )(1 + y 2 )
el menor valor de f es 2.

 Prueba. Observamos que x2 + y 2 + 2 = (1 + x2 ) + (1 + y 2 ).


Luego
p
(1 + x2 ) + (1 + y 2) 2 (1 + x2 )(1 + y 2 )
p
x2 + y 2 + 2 2 (1 + x2 )(1 + y 2 )
x2 + y 2 + 2
p 2
(1 + x2 )(1 + y 2 )
f (x, y) 2

el mnimo valor de f es 2 y ocurre si y solo si x = y. 2

12. Pruebe que

p
x2 + y 2 + 1 > x y 2 + 1 + y x2 + 1,

para todo x, y n
umeros reales.
1.3. Aplicaciones 19

p
 Prueba. Como en el segundo miembro tenemos x y 2 + 1, entonces pode-
mos a partir de:
p
(x y 2 + 1)2 0
p
x2 + y 2 + 1 2x 1 + y 2 0
p
x2 + y 2 + 1 2x 1 + y 2; simirlamente

x2 + y 2 + 1 2y 1 + x2 .

Sumando miembro a miembro tenemos:


p
2(x2 + y 2 + 1) 2x 1 + y 2 + 2y 1 + x2
p
x2 + y 2 + 1 x 1 + y 2 + y 1 + x2 .

La igualdad ocurre si y solo si


( p
x = 1 + y2

y = 1 + x2

(
x2 = 1 + y 2

y 2 = 1 + x2
x2 + y 2 = x2 + y 2 + 2,

como esto es imposible, entonces no ocurre la igualdad.

p
x2 + y 2 + 1 > x y 2 + 1 + y x2 + 1.

13. Pruebe que

2(1 a + a2 )(1 b + b2 ) 1 + a2 b2 ,

para todo a, b n
umeros reales.
20 1. N
umeros Reales

 Prueba. Veamos el primer miembro de la desigualdad:

2(1 a + a2 )(1 b + b2 ) = (2 2a + 2a2 )(1 b + b2 )


= (1 2a + a2 + 1 + a2 )(1 b + b2 )
= ((1 a)2 + 1 + a2 )(1 b + b2 )
= (1 a)2 (1 b + b2 ) + (1 + a2 )(1 b + b2 )
= (1 a)2 (1 2b + b2 + b) + (1 + a2 )(1 b + b2 )
= (1 a)2 ((1 b)2 + b) + (1 + a2 )(1 b + b2 )
= (1 a)2 (1 b)2 + b(1 a)2
+(1 + a2 )(1 b + b2 )
= (1 a)2 (1 b)2 + b 2ab + a2 b + 1 + a2 b
a2 b + b2 + a2 b2
= (1 a)2 (1 b)2 + 1 + a2 b2 + a2 2ab + b2
= (1 a)2 (1 b)2 + 1 + a2 b2 + (a b)2
= 1 + a2 b2 + (a b)2 + (1 a)2 (1 b)2
1 + a2 b2 .

La igualdad ocurre si y solo si a = b = 1. 2

14. Resuelva

1 3a + 5a2 11a3 + 12a4 > 0.

Soluci
on. Ordenando en forma descendente

12a4 11a3 + 5a2 3a + 1 > 0


 
4 4 3 a2 9
a + 11 a a + + a2 3a + 1 > 0
4 4
 
a 2 1 2
a4 + 11 a2 + (9a 12a + 4) > 0
2 4
 a  2 1
a4 + 11 a2 + (3a 2)2 > 0,
2 4
1.3. Aplicaciones 21

como el lado izquierdo es una suma de cuadrados, la desigualdad se verifica


para todo a R.

C.S. = R = h, +i.

15. (Rusia 1995). Pruebe que para todo x, y n


umeros reales positivos

x y 1
+ .
x4 + y 2 x2 + y 4 xy

 Prueba. Como

1 1
x4 + y 2 2x2 y 2
+yx4
2 2x y
x 1
4 2

x +y 2xy

similarmente

y 1

x2 +y 4 2xy

sumando miembro a miembro tenemos:

x y 1
+ 2 .
x4 +y 2 x +y 4 xy

La igualdad ocurre si y solo si a = b = c. 2

16. Pruebe que para todo a, b, c n


umeros reales positivos

8a2 4ab + 5b2 8b2 4bc + 5c2 8c2 4ac + 5a2


S= + + 3(a + b + c).
2a + b 2b + c 2c + a
22 1. N
umeros Reales

 Prueba. Como

a2 + b2 2ab
a2 ab + b2 ab
4a2 4ab + 4b2 4ab
8a2 4ab + 5b2 4a2 + 4ab + b2
8a2 4ab + 5b2 (2a + b)2
8a2 4ab + 5b2
2a + b; similarmente
2a + b
8b2 4bc + 5c2
2b + c;
2b + c
8c2 4ac + 5a2
2c + a;
2c + a

sumando miembro a miembro, tenemos que

S 3(a + b + c).

La igualdad ocurre si y solo si a = b = c. 2

17. Pruebe que

 
1 1 2 a2 + b2
2 2
+ 2 2

a + ab + b a ab + b 3 a2 b2

para todo a, b n
umeros reales no nulos.

 Prueba. Efectuando el lado izquierdo de la desigualdad, tenemos:

1 1 2(a2 + b2 )
+ =
a2 + ab + b2 a2 ab + b2 a4 + a2 b2 + b4

acotando el denominador
1.3. Aplicaciones 23

a4 + b4 2a2 b2
a4 + a2 b2 + b4 3a2 b2
1 1
4 2 2 4
2 2
a +a b +b 3a b
2 2
2(a + b ) 2(a2 + b2 )
4 .
a + a2 b2 + b4 3a2 b2

La igualdad ocurre si y solo si a = b. 2

18. Pruebe que

x2 + y 2 + z 2
zx + yz ,
2

para todo x, y, z n
umeros reales.

 Prueba. Como en el lado izquierdo de la desigualdad, z es el termino


com
un; entonces escogemos:
 2
z z
x2 + 2x
2 2
z2
x2 + 2xz; similarmente
2
z2
y2 + 2yz;
2

sumando miembro a miembro, tenemos:



x2 + y 2 + z 2 2(xz + yz)
x2 + y 2 + z 2
xz + yz
2

x2 + y 2 + z 2
zx + yz .
2
z
La igualdad ocurre si y solo si x = y = . 2
2
24 1. N
umeros Reales

19. Sean a, b n
umeros reales positivos, pruebe que
 3
a3 + b3 a+b
.
2 2

 Prueba. La desigualdad es equivalente a:

4(a3 + b3 ) (a + b)3 4a3 + 4b3 a3 + b3 + 3a2 b + 3ab2


3a3 + 3b3 3a2 b + 3ab2
a3 + b3 a2 b + ab2
a3 a2 b + b3 ab2 0
a2 (a b) + b2 (a b) 0
(a2 + b2 )(a b) 0
(a b)2 (a + b) 0.

La igualdad ocurre si y solo si a = b. 2

20. Pruebe que

2(a3 + b3 + c3 ) a2 (b + c) + b2 (a + c) + c2 (a + b),

para todo a, b, c n
umeros reales positivos.

 Prueba. Como a, b R+ , entonces

(a + b)(a b)2 0, efecuando


a3 + b3 a2 b + ab2 , similarmente
b3 + c3 b2 c + bc2
c3 + a3 c2 a + ca2

sumando miembro a miembro, tenemos que

2(a3 + b3 + c3 ) a2 (b + c) + b2 (a + c) + c2 (a + b).

La igualdad ocurre si y solo si a = b = c. 2


CAPITULO 2

VALOR ABSOLUTO Y MAXIMO
ENTERO

2.1. Valor Absoluto


Definicion 2.1: En el conjunto de los n
umeros reales R, definimos el valor absoluto
de un elemento x, denotado por |x| para indicar el valor absoluto

x;
para x > 0;
|x| = 0; para x = 0;


x; para x < 0.

Ejemplo 2.1
|2| = 2; | 3| = ( 3) = 3; |1 2| = (1 2) = 2 1.

Teorema 2.1

|x| = max {x; x}.

 Prueba.
x;
para x > 0;
max {x; x} = 0; para x = 0; = |x|.


x; para x < 0.
2

25
26 2. Valor Absoluto y Maximo Entero

Teorema 2.2
Para elementos arbitrarios x, y reales, se cumplen

(i) | x| = |x|.

(ii) |x2 | = x2 = |x|2 .



(iii) |x| = x2 .

(iv) |xy| = |x| |y|.

x |x|
(v) | | = .
y |y|

 Prueba.

(i) Aplicando el Teorema 2.1 tenemos

| x| = max {x; (x)}


= max {x; x}
= |x|.

(ii) Por definicion |x2 | = x2 , pues x2 0. Similarmente


x;
x > 0;
|x| = 0; x = 0;


x; x < 0;

entonces


2
x ;
x < 0;
|x|2 = 02 ; x = 0;

2
(x) ; x > 0;
= x2 , x R.
2.1. Valor Absoluto 27

(iii)
|x|2 = x2
p
|x|2 = x2

|x| = x2 .

(iv)
(x y)2 = x2 y 2
p p
(x y)2 = x2 y 2
p
|xy| = x2 y 2
|xy| = |x| |y|

(v) Similar a (iv).

Teorema 2.3
Para cualesquiera x, y, z n
umeros reales, se cumplen

(i) |x| x.

(ii) |x| x.

(iii) |x + y| |x| + |y|.

(iv) ||x| |y|| |x y|.

(v) |x z| |x y| + |y z|.

 Prueba.

(i) |x| = max {x; x} x.

(ii) |x| = max {x; x} x.


28 2. Valor Absoluto y Maximo Entero

(iii)
|xy| xy 2|x||y| 2xy
x2 + y 2 + 2|x||y| x2 + y 2 + 2xy
|x|2 + |y|2 + 2|x||y| (x + y)2
(|x| + |y|)2 (x + y)2
|x| + |y| |x + y|
|x + y| |x| + |y|.

La igualdad ocurre si y solo xy 0.

(iv)

|x| = |(x y) + y| |x y| + |y|


|x| |y| |x y| (2.1)

Similarmente

|y| = |(y x) + x| |y x| + |x|


|y x| |x| |y|
|x y| |x| |y| (2.2)

De (2.1) y (2.2)

|x y| |x| |y| |x y|
||x| |y|| |x y|.

(v)
|x z| = |(x y) + (y z)| |x y| + |y z|
|x z| |x y| + |y z|.

2
2.2. Maximo Entero o Parte Entera 29

2.2. M
aximo Entero o Parte Entera
Definicion 2.2: El maximo entero del n
umero real x se denota [x] y se define de la
siguiente manera

[x] = n n x < n + 1; donde n Z.

Ejemplo 2.2

[1,4] = 1; [2,7] = 2; [1,3] = 2;



[ 7] = 2; [] = 3; [] = 4;
[4] = 4; [3] = 3.

Teorema 2.4
Para todo x n
umero real:

(i) [x] x < [x] + 1.

(ii) 0 x [x] < 1.

(iii) [[x]] = [x].

 Prueba.

(i) De la definicion

[x] = n n x < n + 1; n Z

reemplazando n = [x] se tiene

[x] x < [x] + 1.

(ii) En (i), restando [x] a todos los miembros:

0 x [x] < 1.
30 2. Valor Absoluto y Maximo Entero

(iii) Sea [x] = m con m Z [[x]] = [m] = m = [x].

[[x]] = [x].
2
Teorema 2.5
Para todo x n
umero real y n n
umero entero, se cumple que
(i) [x + n] = [x] + n.
(
[x]; si x = [x];
(ii) [x] =
[x] 1; si x 6= [x].
h x i  [x] 
(iii) = , n 1.
n n

 Prueba.
(i) Sea m = [x + n]. De la definicion

[x + n] = m m x + n < m + 1
(m n) x < (m n) + 1; (m n) Z
[x] = m n
[x] + n = m
[x] + n = [x + n]

(ii) Se deja como ejercicio para el lector.


(iii)
x = k + p; k Z; 0 p < 1.
x k p haciendo k = nq + r; q Z; 0 r < n,
= + ;
n n n 0 r n 1.
x r+p pero 0 r+p < n
=q+ ;
n n r+p
0 < 1
n
hxi
= q.
n
2.2. Maximo Entero o Parte Entera 31

Tambien [x] = [k + p] = k

      h
[x] k nq + r ri
= = = q+ = q,
n n n n

hxi 
[x]
de donde = .
n n
2

on 2.3 : La parte residual de x se denota {x} y se define como


Definici

{x} = x [x].

De la definicion se tiene

0 {x} < 1 y {x} = 0 x Z.

Cuales son los valores posibles de {x} + {x}?

Teorema 2.6 (De Hermite)


Sea x un n umero entero 1, entonces
umero real, y n un n

n1 
X 
k
x+ = [nx].
k=0
n

 Prueba. Definimos la funcion f : R R


     
1 2 n1
f (x) = [x] + x + + x+ ++ x + [nx] .
n n n

Vemos que f es nula:

f (x) = 0, x [0; 1/ni.


 
1
Veamos que f x + = f (x). (f de periodo 1/n), en efecto
n
32 2. Valor Absoluto y Maximo Entero

           
1 1 2 3 1 n1 1
f x+ = x+ + x+ + x+ + x+ + n(x + )
n n n n n n n
       
1 1 2 n1
f x+ = x+ + x+ ++ x + + [x + 1] [nx + 1]
n n n n | {z }
[x]+1([nx]+1)
     
1 2 n1
= [x] + x + + x+ ++ x + [nx]
n n n
= f (x),

de donde f (x) = 0, x R.
n1 
X 
k
x+ = [nx].
k=0
n
2
Teorema 2.7
Sea a un n
umero entero y b un n
umero entero positivo, entonces
hai b a 1
+ = 0.
b b

 Prueba. La prueba se deja para el lector. 2

2.3. Aplicaciones
 
x2
1. Resuelva = 1.
16

Soluci
on.  
x2 x2
=1 1 <2
16 16
16 x2 < 32

4 x < 4 2 4 2 < x 4.
D h E
i
C.S. = 4 2, 4 4, 4 2 .

2
2.3. Aplicaciones 33

2. Resuelva la inecuacion
 2 
x 1 > 2.

Soluci
on. La inecuacion:

[x2 1] > 2 [x2 1] 2
x2 1 2
x2 3

x 3 x 3
D
i h E
C.S. = , 3 3, + .

3. Resuelva en Z, la ecuacion
 
5x
 

2 = x2 .
2 4

Soluci on. Por el teorema 2.5, el lado izquierdo de la ecuacion se puede


escribir como:
 
5x 5x  
2 2 5 x
= = .
2 2 4

Luego resolveremos
   
5x x2
=
4 4
   
5x x2
+ = 0.
4 4
34 2. Valor Absoluto y Maximo Entero

En el teorema 2.7, haciendo a = 5 x, b = 4


   
5x 4 (5 x) 1
+ =0
4 4
   
5x x2
+ = 0.
4 4

La ecuacion se verifica para todo x Z, pues es una aplicacion directa del


teorema 2.7. 2

4. Pruebe que


[ n + n + 1] = [ 4n + 2]

para todo n n
umero entero positivo.


 Prueba. Basta probar que entre n+ n+1 y 4n + 2 no existe un
entero.
La prueba la haremos por contradiccion; en efecto, supongamos que existe
r Z+ tal que

n + n + 1 < r < 4n + 2

( n + n + 1)2 < r 2 < ( 4n + 2)2
p
2n + 1 + 2 n(n + 1) < r 2 < 4n + 2

pero

n< n2 + n

2n < 2 n2 + n

2n + 2n + 1 < 2 n2 + n + 2n + 1,

luego tenemos que


4n + 1 < 2 n2 + n + 2n + 1 < r 2 < 4n + 2,
2.3. Aplicaciones 35

como 4n + 1 y 4n + 2 son consecutivos, entonces no existe r 2 entre ellos, lo


cual es una contradiccion a lo supuesto.

Luego n + n + 1 y 4n + 2 tienen el mismo maximo entero.


[ n + n + 1] = [ 4n + 2].
2

5. Si n es un entero positivo, pruebe que


 
n 17
n 12
 
8n + 13 25




25 3

es independiente de n.

 Prueba. En el teorema 2.7, haciendo a = n - 17,b = 25, tenemos que:


   
n 17 25 (n 17) 1
+ =0
25 25
   
n 17 41 n
+ =0
25 25
   
41 n n 17
=
25 25
   
41 n n 17
n 12 + = n 12
25 25
   
41 n n 17
n 12 + n 12
25 25
= .
3 3

Tomando maximo entero:


   
41 n n 17
n 12 + 25 n 12 25
= ,
3 3
36 2. Valor Absoluto y Maximo Entero

efectuando el lado izquierdo

   
41 n 41 n
n 12 + 25 n 12 + 25
= ; pues n 12 Z
3 3

 
24n 259 24n 259 24n 259
25 25 3
=

=
=
3 3 25

   
24n 259 259
 
3 8n + 3 8n 87

=
25 = 25 = 25
     
8n + 13 100 8n + 13 8n + 13
= = 4 = 4,
25 25 25

 
n 17
n 12
 
8n + 13 25
luego 4 =

.

25 3

 
n 17
n 12
 
8n + 13 25


= 4.

25 3

2
CAPITULO 3
DESIGUALDAD DE
CAUCHY-SCHWARZ

Teorema 3.1 (Desigualdad de Cauchy-Schwarz)


Sean a1 , . . . , an , b1 , . . . , bn , n
umeros reales, entonces

(a1 b1 + a2 b2 + + an bn )2 (a21 + a22 + + a2n )(b21 + b22 + + b2n ).

La igualdad ocurre si y solo si las n-uplas (a1 , a2 , . . . , an ) y (b1 , b2 , . . . , bn ) son


colineales.

 Primera Prueba. Definimos el polinomio cuadratico

f (x) = (a1 x b1 )2 + (a2 x b2 )2 + + (an x bn )2 ; x R.

Vemos que f (x) 0, para todo x R.


Efectuando

f (x) = (a21 x2 2a1 b1 x + b21 ) + (a22 x2 2a2 b2 x + b22 ) +


+(a2n x2 2an bn x + b2n ) 0
f (x) = (a21 + a22 + + a2n )x2 2(a1 b1 + a2 b2 + + an bn )x
+(b21 + b22 + + b2n ) 0
f (x) = Ax2 2Bx + C 0,

37
38 3. Desigualdad de Cauchy-Schwarz

donde

a21 + a22 + + a2n = A,


(a1 b1 + a2 b2 + + an bn ) = B,
b21 + b22 + + b2n = C.

Luego Ax2 2Bx + C 0, para todo x R, y esto ocurre si 0. (:


Discriminante.)
En efecto

= (2B)2 4AC 0
4B 2 4AC 0
B 2 AC

Reemplazando, tenemos que

(a1 b1 + a2 b2 + + an bn )2 (a21 + a22 + + a2n )(b21 + b22 + + b2n ).

La igualdad ocurre cuando f (x) presenta races reales iguales.

(a1 x b1 ) = 0 (a2 x b2 ) = 0 (an x bn ) = 0


a1 x = b1 a2 x = b2 an x = bn

de donde (a1 , a2 , . . . , an ) y (b1 , b2 , . . . , bn ) son colineales o proporcionales. 2

n
! n
! n
!2
X X X
 Segunda Prueba. Haciendo M = a2i b2i ; N = ai bi , en-
i=1 i=1 i=1
tonces tenemos que
3.1. El Lema de Titu 39

n
! n
! n
!2
X X X
M N = a2i b2i ai bi
i=1 i=1 i=1
n
X X n
X X
= a2i b2i + a2i b2j a2i b2i 2 ai bj aj bi
i=1 i6=j i=1 1i<jn
X X
= a2i b2j 2 ai bj aj bi
i6=j 1i<jn
X  X
= a2i b2j + a2j b2i 2 ai bj aj bi
1i<jn 1i<jn
X 
= a2i b2j 2ai bj aj bi + a2j b2i
1i<jn
X
= (ai bj aj bi )2 0.
1i<jn

Entonces
M N 0
M N. [9]
2

3.1. El Lema de Titu


Sean a1 , a2 , . . . , an n
umeros reales arbitrarios y x1 , x2 , . . . , xn n
umeros reales po-
sitivos, se tiene la desigualdad

a21 a22 a2n (a1 + a2 + + an )2


+ ++ .
x1 x2 xn x1 + x2 + + xn

 Primera Prueba. Aplicando Cauchy-Schwarz

s s 2
 
a21 a22 a2n a21 a2n
+ ++ (x1 + x2 + + xn ) x1 + + xn
x1 x2 xn x1 xn
 
a21 a22 a2
+ ++ n (x1 + x2 + + xn ) (a1 + a2 + + an )2 .
x1 x2 xn
40 3. Desigualdad de Cauchy-Schwarz

a21 a22 a2n (a1 + a2 + + an )2


De donde tenemos que + ++ . 2
x1 x2 xn x1 + x2 + + xn
Como vemos, es una aplicacion de la desigualdad de Cauchy-Schwarz y por ello
algunos afirman que simplemente es la desigualdad de Cauchy-Schwarz.

 Segunda Prueba. (Por induccion).


Veamos que la induccion se reduce al caso n = 2.

a21 a22 (a1 + a2 )2


+
x1 x2 x1 + x2
 
a1 x2 + a2 x1 (x1 + x2 ) x1 x2 a21 + a22 + 2a1 a2
2 2

a21 x2 x1 + a21 x22 + a22 x21 + a22 x1 x2 a21 x1 x2 + a22 x1 x2 + 2a1 a2 x1 x2


a21 x22 + a22 x21 2a1 a2 x1 x2 0
(a1 x2 a2 x1 )2 0,

a1 a2
y la igualdad se tiene si y solo si = .
x1 x2
Aplicando el resultado dos veces se tiene que

a21 a22 a23 (a1 + a2 )2 a23


+ + +
x1 x2 x3 x1 + x2 x3
(a1 + a2 + a3 )2
.
x1 + x2 + x3
Supongamos que se cumple para n

a21 a22 a2n (a1 + a2 + + an )2


+ ++ .
x1 x2 xn x1 + x2 + + xn
Veamos para n + 1

 
a21 a22 a2n a2n+1 (a1 + a2 + + an )2 a2n+1
+ ++ + +
x1 x2 xn xn+1 x1 + x2 + + xn xn+1
(a1 + a2 + + an+1 )2
. [11]
x1 + x2 + + xn+1

2
3.2. Desigualdad de Sch
ur 41

3.2. Desigualdad de Sch


ur
Si x, y, z son n
umeros reales positivos y n un entero positivo, entonces

xn (x y)(x z) + y n (y z)(y x) + z n (z x)(z y) 0.

 Prueba. Como el primer miembro es simetrico, entonces podemos asumir un


orden.
Sea x y z, entonces:

x z y z, x y 0
(x y)(x z) (x y)(y z), xn y n
xn (x y)(x z) y n (x y)(y z). (3.1)

Tambien

(z x)(z y) 0, pues x z, y z
n
z (z x)(z y) 0. (3.2)

Sumando (3.1) y (3.2)

xn (x y)(x z) + z n (z x)(z y) y n (x y)(y z)


xn (x y)(x z) y n (x y)(y z) + z n (z x)(z y) 0
xn (x y)(x z) + y n (y x)(y z) + z n (z x)(z y) 0.

La igualdad ocurre si y solo si x = y = z. 2

3.3. Dos sustituciones muy u


tiles
Si en las inecuaciones tenemos la condicion abc = 1, es conocida la sustitucion
x y z
a= , b= , c= ,
y z x
que hace que el problema se convierta en otro mas facil; aunque esto no es siempre
para todo problema.
42 3. Desigualdad de Cauchy-Schwarz

Veamos otras sustituciones que son muy u tiles en la resolucion de problemas de


desigualdades en los torneos internacionales.
Si tenemos las condiciones x, y, z > 0 y xyz = x + y + z + 2, nos preguntamos,
cual sera la sustitucion?
De la condicion

xyz = x + y + z + 2

tenemos

xyz + xy + xz + yz + x + y + z + 1 = xy + xz + yz + 2(x + y + z) + 3
(x + 1)(y + 1)(z + 1) = (xy + x + y + 1) + (xz + x + z + 1)
+(yz + y + z + 1)
(x + 1)(y + 1)(z + 1) = (x + 1)(y + 1) + (x + 1)(z + 1)
+(y + 1)(z + 1)

1 1 1
1= + + .
z+1 y+1 x+1
1 1 1
Haciendo a = , b= , c= , se tiene a + b + c = 1 y
x+1 y+1 z+1
1a b+c c+a a+b
x= = , y= , z= .
a a b c

Ahora veamos otra condicion.


Para x, y, z > 0 y xy + yz + zx + 2xyz = 1 la sustitucion es

a b c
x= , y= , z= ,
b+c c+a a+b

pues xy + yz + zx + 2xyz = 1 equivale a

1 1 1 1
= + + + 2,
xyz x y z

es decir, en la primera sustitucion solo ha sido cambiada por la inversa de x, y, z.


[11]
3.4. Aplicaciones 43

3.4. Aplicaciones
umeros reales tales que a2 + b2 = 1; halle la variacion de 2a + b.
1. Sean a, b n

Soluci
on. Como nos piden la variacion de 2a+b que es equivalente a escribir
2a+1b; entonces escogemos los pares (2, 1) y (a, b) para aplicar la desigualdad
de Cauchy-Schwarz.
En efecto, tenemos:

(2 a + 1 b)2 (22 + 12 )(a2 + b2 )


(2a + b)2 (5)(1)
(2a + b)2 5

5 2a + b 5.


2a + b [ 5, 5].

2. Sean x 1, y 1. Pruebe que


 2  
x1 y1 1 1
+ (x + y 2) 2
+ 2 .
x y x y
 
1 1
 Prueba. Consideremos los pares ( x 1, y 1), , y aplicando
x y
la desigualdad de Cauchy-Schwarz tenemos

 2  !
1 1   1 2  1 2
2 2
x1 + y1 x1 + y1 +
x y x y
 2  
x1 y1 1 1
+ (x 1 + y 1) +
x y x2 y 2
 2  
x1 y1 1 1
+ (x + y 2) + .
x y x2 y 2

La igualdad ocurre si y solo si x = y. 2


44 3. Desigualdad de Cauchy-Schwarz

3. Pruebe que

(a2 + ab + b2 )2 (a2 + 2b2 )(2a2 + b2 ),

para todo a, b n
umeros reales.

 Prueba. Tomamos las ternas

(a, a, b), (a, b, b)

para aplicar la desigualdad de Cauchy-Schwarz.


En efecto

(a a + a b + b b)2 (a2 + a2 + b2 )(a2 + b2 + b2 )

Luego tenemos

(a2 + ab + b2 )2 (2a2 + b2 )(a2 + 2b2 ).

La igualdad ocurre si y solo si a = b. 2

4. Sean a, b, c n
umeros reales, tales que no son nulos simultaneamente, halle el
valor maximo de

|a + 3b + 9c|
f (a, b, c) = .
a2 + b2 + c2

Soluci
on. Sean las ternas (a, b, c) y (1, 3, 9), entonces

(a 1 + b 3 + c 9)2 (a2 + b2 + c2 )(12 + 32 + 92 )


(a + 3b + 9c)2 91(a2 + b2 + c2 )
p p
(a + 3b + 9c)2 91 (a2 + b2 + c2 )
|a + 3b + 9c|
91,
a2 + b2 + c2
3.4. Aplicaciones 45


como f (a, b, c) 0 entonces 0 f (a, b, c) 91.

El maximo valor de f es 91, y esto ocurre si y solo si (a, b, c) = (1, 3, 9).
2

5. Sean a, b n
umeros reales, halle la variacion de

f () = (a sen + b cos ).

Soluci
on. Sean los pares (a, b), (sen , cos ). Utilizando Cauchy-Schwarz
tenemos:

(a sen + b cos )2 (a2 + b2 )(sen 2 + cos 2 )


(a sen + b cos )2 (a2 + b2 )(1)
(a sen + b cos )2 a2 + b2

a2 + b2 a sen + b cos a2 + b2 .


a2 + b2 f () a2 + b2 .

6. Sean a, b, c numeros reales tal que a + 2b + 3c = 14, halle la variacion de


a2 + b2 + c2 .

Soluci
on. Escogemos las ternas (1, 2, 3) y (a, b, c) para utilizar la desigual-
dad de Cauchy-Schwarz.
En efecto

(1 a + 2 b + 3 c)2 (12 + 22 + 32 )(a2 + b2 + c2 )


142 14(a2 + b2 + c2 )
14 a2 + b2 + c2

a2 + b2 + c2 14.

La igualdad ocurre si y solo si (a, b, c) = (1, 2, 3). 2


46 3. Desigualdad de Cauchy-Schwarz

7. Pruebe que para todo x, y, z n


umeros reales positivos

 1/2  1/2  1/2


x+y x+z y+z
+ + 61/2 .
x+y+z x+y+z x+y+z

 Prueba. Tomamos las ternas

 1/2  1/2  1/2 !


x+y x+z y+z
, , ; (1, 1, 1),
x+y+z x+y+z x+y+z

aplicando Cauchy-Schwarz:

 1/2  1/2  1/2 !2


x+y x+z y+z
1+ 1+ 1
x+y+z x+y+z x+y+z
 
x+y x+z y+z
+ + (12 + 12 + 12 )
x+y+z x+y+z x+y+z
 
2(x + y + z)
= (3)
x+y+z
= 6.

 1/2  1/2  1/2


x+y x+z y+z
+ + 61/2 .
x+y+z x+y+z x+y+z

La igualdad ocurre si y solo si x = y = z. 2

8. Pruebe que

 
x2 y2 z2
x+y+z 2 + + ,
y+z x+z x+y

para todo x, y, z n
umeros reales positivos.
3.4. Aplicaciones 47

 Prueba. Como

 2
2 x y z
(x + y + z) = y+z+ x+z+ x+y
y+z x+z x+y
 2 
x y2 z2
+ + (y + z + x + z + x + y)
y+z x+z x+y
 2 
2 x y2 z2
(x + y + z) 2 (x + y + z) + + ;
y+z x+z x+y

cancelando x + y + z > 0

 
x2 y2 z2
x+y+z 2 + + .
y+z x+z x+y

La igualdad ocurre si y solo si x = y = z. 2

1 1 1
9. Sean x, y, z > 1 tales que + + = 2, pruebe que
x y z
p
x+y+z x1+ y 1 + z 1.

 Prueba. Para aplicar Cauchy-Schwarz, tomamos las ternas

 
x1 y1 z1 
, , y x, y, z ;
x y z

en efecto tenemos que

 2
x1 y1 z1
x+
y+ z
x y z
 
x1 y1 z1
+ + (x + y + z)
x y z
48 3. Desigualdad de Cauchy-Schwarz

 2  
p 1 1 1
x1+ y1+ z1 1 +1 +1
x y z
(x + y + z)
  
1 1 1
= 3 + + (x + y + z)
x y z
= (3 2) (x + y + z)
= (x + y + z).

 p 2
Entonces x1+ y1+ z1 x + y + z.
p
x 1 + y 1 + z 1 x + y + z.

3
La igualdad ocurre si y solo si x = y = z = . 2
2

10. (China 1984/1985). Sean a1 , a2 , . . . , an > 0, pruebe que

a21 a22 a2 a2
+ + + n1 + n a1 + a2 + + an .
a2 a3 an a1

 Prueba. Aplicando el Lema de Titu.

a21 a22 a2 a2 (a1 + a2 + + an1 + an )2


+ + + n1 + n
a2 a3 an a1 a2 + a3 + + an + a1
= a1 + a2 + + an .

La igualdad ocurre si y solo si a1 = a2 = = an . 2

11. (Desigualdad de Nesbit) Pruebe que

a b c 3
+ + ,
b+c a+c a+b 2

para todo a, b, c n
umeros reales positivos.
3.4. Aplicaciones 49

 Prueba.

a b c a2 b2 c2
+ + = + +
b+c a+c a+b ab + ac ba + bc ca + cb
(a + b + c)2

(ab + ac) + (ba + bc) + (ca + cb)
(a + b + c)2
= ,
2(ab + bc + ca)

basta demostrar que

(a + b + c)2 3
.
2(ab + bc + ca) 2

Como

a2 + b2 + c2 ab + ac + bc
a2 + b2 + c2 + 2(ab + ac + bc) 3(ab + ac + bc)
(a + b + c)2 3
.
2(ab + bc + ca) 2

La igualdad ocurre si y solo si a = b = c. 2

12. Pruebe que para todo a, b, c n


umeros reales positivos

a3 b3 c3 a+b+c
2 2
+ 2 2
+ 2 2
.
a + ab + b b + bc + c c + ca + a 3

 Prueba. El lado izquierdo de la desigualdad se puede escribir como

a4 b4 c4
+ + ,
a3 + a2 b + ab2 b3 + b2 c + bc2 c3 + c2 a + ca2
50 3. Desigualdad de Cauchy-Schwarz

luego, aplicando el Lema de Titu, tenemos:

a4 b4 c4
+ +
a3 + a2 b + ab2 b3 + b2 c + bc2 c3 + c2 a + ca2
(a2 + b2 + c2 )2
3
a + b3 + c3 + ab(a + b) + bc(b + c) + ca(c + a)
(a2 + b2 + c2 )2
=
(a + b + c)(a2 + b2 + c2 )
a2 + b2 + c2
= ,
a+b+c

entonces

a3 b3 c3 a2 + b2 + c2
+ + .
a2 + ab + b2 b2 + bc + c2 c2 + ca + a2 a+b+c

Pero

a2 + b2 + c2 ab + ac + bc
2(a2 + b2 + c2 ) 2(ab + ac + bc)
3(a2 + b2 + c2 ) a2 + b2 + c2 + 2(ab + ac + bc)
3(a2 + b2 + c2 ) (a + b + c)2
a2 + b2 + c2 a+b+c
.
a+b+c 3

De donde

a3 b3 c3 a+b+c
2 2
+ 2 2
+ 2 2
.
a + ab + b b + bc + c c + ca + a 3

La igualdad ocurre si y solo si a = b = c. 2

13. Sean a, b, c n
umeros reales positivos. Pruebe que

a3 + b3 + c3 + 3abc ab(a + b) + bc(b + c) + ca(c + a).


3.4. Aplicaciones 51

 Prueba. La desigualdad es equivalente a

a3 + b3 + c3 + 3abc a2 b ab2 b2 c bc2 c2 a + ca2 0


(a3 a2 (b + c) + abc) + (b3 b2 (a + c) + abc) + (c3 c2 (b + a) + abc) 0
a (a2 (b + c)a + bc) + b (b2 (a + c)b + ac) + c (c2 (b + a)c + ab) 0
a(a b)(a c) + b(b a)(b c) + c(c a)(c b) 0,

y esta es la desigualdad de Sch


ur para n = 1.
La igualdad ocurre si y solo si a = b = c. 2

14. Pruebe la desigualdad de Nesbit utilizando las sustituciones u


tiles

a b c 3
+ + ,
b+c a+c a+b 2

para todo a, b, c n
umeros reales positivos.

a b c
 Prueba. Haciendo x = , y= , z= .
b+c c+a a+b
Entonces es suficiente probar que si x, y, z > 0 con xy + yz + zx + 2xyz = 1,
entonces x + y + z 3/2.
3
Supongamos que x + y + z < .
2
Como

(x + y + z)2 3
xy + yz + zx xy + yz + zx <
3 4
 3
x+y+z 1
xyz 2xyz <
3 4

de donde

3 1
1 = (xy + yz + zx) + 2xyz < + = 1.
4 4
52 3. Desigualdad de Cauchy-Schwarz

Lo cual es una contradiccion, entonces

x + y + z 3/2.

15. Pruebe que si x, y, z son n


umeros reales positivos y xyz = x+y +z +2 entonces

xy + yz + zx 2(x + y + z).

 Prueba. De x, y, z > 0 y xyz = x + y + z + 2, tenemos la sustitucion

b+c c+a a+b


x= , y= , z= .
a b c

Entonces sera suficiente demostrar

        
b+c c+a c+a a+b a+b b+c
+ +
a b b c c a
 
b+c c+a a+b
2 + +
a b c
a3 + b3 + c3 + 3abc ab(a + b) + bc(b + c) + ca(c + a)
a3 + b3 + c3 + 3abc a2 (b + c) + b2 (a + c) + c2 (a + b)
(a3 a2 (b + c) + abc) + (b3 b2 (a + c) + abc) + (c3 c2 (a + b) + abc) 0
a(a b)(a c) + b(b a)(b c) + c(c a)(c b) 0.

Esta u
ltima desigualdad es verdadera, pues es la desigualdad de Sch
ur.
La igualdad ocurre si y solo si x = y = z = 2. 2

16. Pruebe que para todo a, b, c n


umeros reales positivos:

(b + c a)2 (c + a b)2 (a + b c)2 3


2 2
+ 2 2
+ 2 2
.
(b + c) + a (c + a) + b (a + b) + c 5
3.4. Aplicaciones 53

 Prueba. En cada fraccion del primer miembro dividimos por a2 , b2 , c2


respectivamente y se tiene

 2  2  2
b+c c+a a+b
1 1 1
a b c 3
 2 + 2 + 2 .
b+c c+a a+b 5
+1 +1 +1
a b c

Utilizando la sustitucion

b+c c+a a+b


= x, = y, = z,
a b c

tenemos la desigualdad equivalente

(x 1)2 (y 1)2 (y 1)2 3


2
+ 2 + 2
x +1 y +1 y +1 5

con xyz = x + y + z + 2, x + y + z 6.
Aplicando el Lema de Titu

(x 1)2 (y 1)2 (y 1)2 (x + y + z 3)2


+ + .
x2 + 1 y2 + 1 y2 + 1 x2 + y 2 + z 2 + 3

Luego sera suficiente probar

(x + y + z 3)2 3
.
x2 + y 2 + z 2 + 3 5
5(x + y + z 3)2 3(x2 + y 2 + z 2 ) + 9 (3.3)

Haciendo

x + y + z = s x2 + y 2 + z 2 = (x + y + z)2 2(xy + xz + yz)


= s2 2(xy + xz + yz).
54 3. Desigualdad de Cauchy-Schwarz

Reemplazando en (3.3)

5(s 3)2 3(s2 2(xy + xz + yz)) + 9

efectuando

5s2 30s + 45 3s2 6(xy + xz + yz) + 9


2s2 30s + 36 6(xy + xz + yz)
s2 15s + 18 3(xy + xz + yz). (3.4)

Pero de la aplicacion 15

xy + yz + zx 2(x + y + z) = 2s
xy + yz + zx 2s
3(xy + yz + zx) 6s
6s 3(xy + yz + zx). (3.5)

De (3.4) y (3.5) es suficiente probar

s2 15s + 18 6s
s2 9s + 18 0
(s 3)(s 6) 0,

es verdadero, pues s 6.
La igualdad ocurre si y solo si a = b = c. 2
CAPITULO 4
DESIGUALDAD DE LA MEDIA

ARITMETICA
- MEDIA GEOMETRICA

En las olimpiadas de matematicas, en lo que corresponde al captulo de de-


sigualdades, una de las propiedades muy utilizadas es la Media Aritmetica - Media
Geometrica; en tal sentido, mostraremos una serie de ejercicios que nos permitan
profundizar.
Antes de enunciar los teoremas, definiremos la Media Aritmetica, Media Ge-
ometrica y Media Armonica de los n umeros x1 , x2 , . . . , xn reales positivos, de la
siguiente manera:

x1 + x2 + + xn
MA = ,
n

MG = n x1 x2 xn ,
n
MH = .
1 1 1
+ ++
x1 x2 xn

Teorema 4.1
Sean x1 , x2 , . . . , xn n
umeros reales positivos entonces su Media Aritmetica es mayor
o igual que su Media Geometrica (MA MG).
La igualdad ocurre si y solo si x1 = x2 = = xn .

 Prueba. Sea Pn = {MA MG para n n


umeros}; probemos por induccion
matematica de la siguiente manera

55
56 4. Desigualdad de la Media Aritmetica - Media Geometrica

(i) Probaremos que se cumple para dos n


umeros, es decir P2 es verdadero.

(ii) Tomando la hipotesis que Pn es verdadero, probaremos que Pn1 es verdadero.

(iii) As mismo probaremos que si Pn es verdadero entonces P2n es verdadero.

Cuando (i), (ii) y (iii) se verifican, entonces Pn con n 2 es verdadero.


Veamos

(i) Como x1 , x2 R+ , entonces x1 , x2 R+ , luego ( x1 x2 )2 0.
Efectuando

x1 2 x1 x2 + x2 0

x1 + x2 2 x1 x2
x1 + x2
x1 x2 .
2

La igualdad ocurre si y solo si x1 = x2 .


Vemos que P2 es verdadero.

(ii) Sea g = n1 x1 x2 xn1 , entonces g n1 = x1 x2 xn1 .
Como Pn es verdadero, entonces

x1 + x2 + + xn1 + g
n x1 x2 xn1 g
n
x1 + x2 + + xn1 + g p
n g n1 g
n

x1 + x2 + + xn1 + g n n g n = ng
x1 + x2 + + xn1 g(n 1)
x1 + x2 + + xn1
g.
n1

De donde

x1 + x2 + + xn1
n1
x1 x2 xn1 .
n1

Vemos que Pn1 es verdadero.


57

(iii) Sean x1 , x2 , . . . , x2n n


umeros reales positivos, entonces

x1 + x2 + + x2n = (x1 + x2 ) + (x3 + x4 ) + + (x2n1 + x2n )



2( x1 x2 + x3 x4 + + x2n1 x2n ) (4.1)

Como Pn es verdadero, entonces

q
x3 x4 + + x2n1 x2n
x1 x2 +
n x1 x2 x3 x4 x2n1 x2n
n q

x1 x2 + x3 x4 + + x2n1 x2n n n x1 x2 x3 x4 x2n1 x2n

2 x1 x2 + x3 x4 + + x2n1 x2n 2n 2n x1 x2 x3 x4 x2n1 x2n . (4.2)

De (4.1) y (4.2) tenemos que


x1 + x2 + + x2n 2n 2n x1 x2 x2n
x1 + x2 + + x2n
2n x1 x2 x2n ,
2n

de donde P2n es verdadero.

Colorario 4.1 (Desigualdad de la Media Geometrica - Media Armonica)


Sean x1 , x2 , . . . , xn n
umeros reales positivos, entonces su Media Geometrica es
mayor o igual que su Media Armonica (MG MH).
La igualdad ocurre si y solo si x1 = x2 = = xn .

1 1 1
 Prueba. Aplicando el teorema 4.1 a los n
umeros , ,..., reales positivos,
x1 x2 xn
tenemos:

1 1 1 r
+ ++
x1 x2 xn 1 1 1
n ,
n x1 x2 xn
58 4. Desigualdad de la Media Aritmetica - Media Geometrica

1 1 1 n
+ ++
x1 x2 xn n
x1 x2 xn
n
n x1 x2 xn ,
1 1 1
+ ++
x1 x2 xn

la igualdad ocurre si y solo si x1 = x2 = = xn . 2

Colorario 4.2 (Desigualdad de la Media Aritmetica - Media Armonica)


Sean x1 , x2 , . . . , xn n
umeros reales positivos, entonces
x1 + x2 + + xn n
.
n 1 1 1
+ ++
x1 x2 xn

 Prueba. La demostracion es inmediata, pues por transitividad, se tendra, ya


que

MA MG MG MH MA MH.

Es importante observar que de MA MH


x1 + x2 + + xn n
,
n 1 1 1
+ ++
x1 x2 xn
se tiene
 
1 1 1
(x1 + x2 + + xn ) + ++ n2 .
x1 x2 xn

4.1. Aplicaciones
1. Sean a, b, c n
umeros reales positivos. Pruebe que

a b c
+ + 3.
b c a
4.1. Aplicaciones 59

 Prueba. Como a, b, c R+ , entonces podemos utilizar MA - MG.


En efecto

a b c r
+ +
b c a 3 abc
3 b c a
a b c
+ + 3 31
b c a
a b c
+ + 3.
b c a

La igualdad ocurre si y solo si a = b = c.


2

2. Sean a, b, c n
umeros reales positivos. Pruebe que

a4 b4 c4
+ + 3abc.
b c a

a4 b4 c4
 Prueba. Utilizando MA - MG, a los n
umeros , , .
b c a
En efecto

a4 b4 c4 r
+ + 4 4
b c a 3 a4 b b c
3 c a
4 4 4
a b c
+ + 3abc.
b c a

La igualdad ocurre si y solo si a = b = c. 2

3. Sabiendo que a, b, c, d son n


umeros reales positivos con abcd = 1, pruebe que

a2 + b2 + c2 + d2 + ab + ac + ad + bc + bd + cd 10.
60 4. Desigualdad de la Media Aritmetica - Media Geometrica

 Prueba. Utilizando MA MG se tiene:

a2 + b2 + c2 + d2 + ab + ac + ad + bc + bd + cd p
10 (abcd)4
10
2 2 2 2

a + b + c + d + ab + ac + ad + bc + bd + cd 10 10 1

a2 + b2 + c2 + d2 + ab + ac + ad + bc + bd + cd 10.

La igualdad ocurre si y solo si a = b = c = d = 1. 2

4. (Australia 2000). Sea a un n


umero real no nulo, b un n
umero real, pruebe que

1 b
a2 + b2 + + 3.
a2 a

 Prueba. Agrupando convenientemente

1 b
a2 + b2 + 2 +
a a
   
2 1 b 2 3
= b + 2+ + a + 2
4a a 4a
 2  
1 3
= b+ + a2 + 2
2a 4a

y ademas
r
3 3 2 3
a2 + 2 2 a2 2 = = 3
4a 4a 2
3
a2 + 2 3
4a
 2    2
1 2 3 1
b+ + a + 2 b+ + 3 3.
2a 4a 2a

1 b
a2 + b2 + 2
+ 3.
a a
1
La igualdad ocurre si y solo si b = . 2
2a
4.1. Aplicaciones 61

5. Pruebe que

a2 + 2b 3(ab)2/3

para todo a, b n
umeros reales positivos.

 Prueba. Observamos que a2 +2b = a2 +b+b, luego aplicamos MA MG


umeros a2 , b, b. En efecto tenemos:
a los n

a2 + b + b
3
a2 b b
3
3
a2 + 2b 3 a2 b2
a2 + 2b 3(ab)2/3 .

La igualdad ocurre si y solo si a2 = b. 2

6. Halle el menor valor de

4
f (x) = x2 + , x > 0.
x

Soluci
on. Vemos que

4 1 1 1 1
f (x) = x2 + = x2 + + + +
x x x x x

1 1 1 1
umeros x2 , , , , , veamos:
entonces aplicamos MA MG a los n
x x x x
1 1 1 1
x2 + + + + r
x x x x 1 1 1 1
5 x2
5 x x x x
4
x2 + 5(1)
x

entonces f (x) 5.
Por lo tanto, el mnimo de f es 5 y ocurre si y solo si x = 1. 2
62 4. Desigualdad de la Media Aritmetica - Media Geometrica

7. Pruebe que
r r r
2+1 3 3+1 n n+1
M =1+ + ++ < n + 1.
2 3 n

r
k k+1
 Prueba. El termino general de la suma del primer miembro es
k
y vemos que

(k 1) sumandos
r v k+1 z }| {
k+1 uk + 1 +1+1++1
u
k
= t
k 1| 1{z 1} < k
k k k
(k 1) factores

r k+1
k+1 +k1 1
k
< k = 1 + 2.
k k k

En efecto
     
1 1 1
M < 1+ 1+ 2 + 1+ 2 ++ 1+ 2
2 3 n
1 1 1
= n+ 2 + 2 ++ 2
2 3 n
1 1 1
< n+ + ++
12 23 (n 1) n
     
1 1 1 1 1 1
= n+ + ++
1 2 2 3 n1 n
1
= n + 1 < n + 1. [10, pag 17]
n
2

8. Pruebe que
r r
n
n
n n
n
n
1+ + 1 < 2,
n n

siendo n entero positivo.


4.1. Aplicaciones 63

 Prueba.

r v  
n
n
u 1 + + (n 1)
n
n
n u n
n n
n
n
1+ = t 1+
n 1| 1{z 1} < =1+ 2 .
n n n n
(n 1) factores

Similarmente
r
n n n n
n
1 <1 2 ,
n n

entonces
r r
n n
n
n n
n n
n n
n
1+ + 1 < 1 + 2 + 1 2 = 2.
n n n n

r r
n
n
n n
n
n
1+ + 1 < 2. [10, pag 17]
n n
2

9. Sean a, b, c n
umeros reales positivos con a + b + c = 3, halle el maximo valor
de
p
3
p
3
p
3
f (a, b, c) = a(b + 2c) + b(c + 2a) + c(a + 2b).

 Prueba. Como f (a, b, c) es una expresion simetrica para a, b, c entonces


el maximo ocurre cuando a = b = c y como a + b + c = 3

a = b = c = 1

3a = 3b = 3c = 3


b + 2c = c + 2a = a + 2b = 3.

Luego aplicando MA MG, de la siguiente manera:


 
p
3 1 p 3 1 3a + (b + 2c) + 3
a(b + 2c) = 3a(b + 2c) 3
3
9 3
9 3
64 4. Desigualdad de la Media Aritmetica - Media Geometrica

 
p 1 3a + b + 2c + 3
a(b + 2c)
3
; similarmente
3
9 3
 
p 1 3b + c + 2a + 3
3
b(c + 2a) ;
3
9 3
 
p 1 3c + a + 2b + 3
3
c(a + 2b) .
3
9 3

Sumando miembro a miembro tenemos

 
p3
p p 1 6(a + b + c) + 9
a(b + 2c) + 3 b(c + 2a) + 3 c(a + 2b) ,
| {z } 3
9 3
f (a,b,c)

1
3
f (a, b, c)
3
9 = 3 3.
9


max f = 3 3 3. [10, pag 16] 2

10. (APMO 1998). Sean a, b, c n


umeros reales positivos. Pruebe que
 
x  y z 2(x + y + z)
1+ 1+ 1+ 2+ .
y z x 3 xyz

     
x y z 2x y 2y z
 Prueba. Observar que 3 + + = + + + +
  y z x y z z x
2z x
+ .
x y
Utilizando MA MG, tenemos que
r
2x y x x y x x y
+ = + + 33 , entonces
y z y y z y y z
s
2x y x2
+ 33 , similarmente
y z yz
4.1. Aplicaciones 65

r
2
2y z 3 y
+ 3 ;
z x xz
s
2z x z2
+ 33
x y xy

de donde tenemos que:

  s r s
x y z 3 x2 3 y2 3 z2
3 + + 3 +3 +3
y z x yz xz xy
 
x y z
= 3 3 xyz
+ 3 xyz
+
3 xyz

x y z x+y+z
+ + ; similarmente
y z x 3 xyz

x z y x+y+z
+ + ,
z y x 3 xyz

Sumando miembro a miembro tenemos

x y z x z y 2(x + y + z)
+ + + + + ,
y z x z y x 3 xyz

sumando 2 en ambos miembros

x y z x y x z y z x y z 2(x + y + z)
1+ + + + + + + 2+ ,
y z x y z y x z x y z x 3 xyz
| {z }
1

de donde
 
x  y z 2(x + y + z)
1+ 1+ 1+ 2+ .
y z x 3 xyz

La igualdad ocurre si y solo si x = y = z. 2


66 4. Desigualdad de la Media Aritmetica - Media Geometrica

11. Sean a, b, c lados de un triangulo con permetro 3. Pruebe que

1 1 1 9
+ + . (4.3)
a+bc b+ca c+ab ab + bc + ca

 Prueba. Haciendo

x= a + b c; y = b + c a; z = c + a b,

x2 + z 2 x2 + y 2
entonces x2 + y 2 + z 2 = a + b + c = 3 y a = , b = ,
2 2
y2 + z2
c= , de donde se deduce que:
2
9 + x2 y 2 + y 2z 2 + z 2 x2
ab + ac + bc = .
4

Luego la desigualdad (4.3) es equivalente a

1 1 1 36
+ +
x y z 9 + x y + y 2z 2 + z 2 x2
2 2
p
(yz + xz + xy) (9 + (xy)2 + (yz)2 + (zx)2 ) 36xyz = 36 (xyz)2 .

Haciendo xy = m, xz = n, yz = p, tenemos


(m + n + p)(9 + m2 + n2 + p2 ) 36 mnp.

Pero aplicando MA MG, tenemos que


m + n + p 3 3 mnp (4.4)
9 sumandos
z }| {
m2 + n2 + p2 + 1 + 1 + + 1 p
12 m2 n2 p2
12 p
m2 + n2 + p2 + 9 12 12 (mnp)2

m2 + n2 + p2 + 9 12 6 mnp. (4.5)
4.1. Aplicaciones 67

Multiplicando miembro a miembro (4.4) y (4.5)


 
(m + n + p)(m2 + n2 + p2 + 9) 3 3 mnp 12 6 mnp

= 36 mnp,

de donde obtenemos:


(m + n + p)(m2 + n2 + p2 + 9) 36 mnp.

La igualdad ocurre si y solo si a = b = c = 1. 2

12. Pruebe que para x, y n umeros reales no negativos, con x y, y n entero


positivo, se cumple que

n1
n(x y)(xy) 2 xn y n .

 Prueba. Hay dos casos:

(i) Si x = y, se cumple la igualdad.


(ii) Si x 6= y

n1 xn y n
n(xy) 2 ; x>y
xy
n1
n(xy) 2 xn1 + xn2 y + xn3 y 2 + + xy n2 + y n1.

Aplicando MA MG

xn1 + xn2 y + + y n1 p
> n (xn1 ) (xn2 y) (xn3 y 2 ) (y n1)
n
q
n (n1)n (n1)n n1
= x 2 y 2 = (xy) 2

Por lo tanto de (i) y (ii) tenemos


n1
xn1 + xn2 y + + y n1 n(xy) 2 .

2
68 4. Desigualdad de la Media Aritmetica - Media Geometrica
CAPITULO 5
DESIGUALDAD DE BERNOULLI Y LA
MEDIA POTENCIAL

La desigualdad de Bernoulli es muy importante, puesto que es muy utilizada en


el analisis matematico y en otras ramas de la matematica.

Teorema 5.1
Si x 1 y n entero positivo, entonces

(1 + x)n 1 + nx.

 Prueba. La demostracion la haremos por induccion.

Si n = 1: 1 + x 1 + x, es verdadera.

Supongamos que se cumpla para n = k, es decir:

(1 + x)k 1 + kx (hipotesis inductiva).

Veremos que se cumple para n = k + 1.


Multiplicando por (1 + x) en la desigualdad anterior

(1 + x)k (1 + x) (1 + kx)(1 + x)
(1 + x)k+1 1 + (k + 1)x + kx2 1 + (k + 1)x

69
70 5. Desigualdad de Bernoulli y la Media Potencial

de donde

(1 + x)k+1 1 + (k + 1)x.

(1 + x)n 1 + nx.

2
Teorema 5.2
Si x 1 y 0 < < 1, entonces

(1 + x) 1 + x.

 Prueba. Si x = 1, la desigualdad se verifica trivialmente.


m
Veamos para x > 1. Sea racional, = , m, n Z+ con 1 m < n.
n
Como 1 + x > 0, tenemos

m p p
(1 + x) = (1 + x) n = n
(1 + x)m = n
(1 + x)m 1nm
s
= (1 + x) (1 + x) (1 + x) 1| 1{z 1}
n | {z }
m nm

m nm
z }| { z }| {
(1 + x) + (1 + x) + + (1 + x) + 1 + 1 + + 1

m + (n m)
m(1 + x) + n m n + mx m
= = = 1 + x = 1 + x,
n n n

de donde se tiene

(1 + x) 1 + x, Q, 0 < < 1.

Ahora veremos para Q . (Q conjunto de los n umeros irracionales.)


Sea (qk )kN = (q1 , q2 , q3 , . . . , qk , . . .) una sucesion de n
umeros racionales tal que
= lm qk y 0 < qk < 1.
k
En efecto

(1 + x)qk 1 + qk x; x 1, k = 1, 2, 3, . . .
71

luego

(1 + x) = lm (1 + x)qk lm (1 + qk x) = 1 + x.
k k

Para completar la prueba, veamos que 0 < < 1 y x 6= 0, se tiene

(1 + x) < 1 + x;

tomemos un numeroh racional iq tal que < q < 1.


q
Y como (1 + x) = (1 + x) q se cumple, entonces

(1 + x) q 1 + x; pues 0 < < 1
q q
 q

(1 + x) 1 + x < 1 + q x; o sea
q q
(1 + x) < 1 + x.

Luego la desigualdad ocurre si y solo si x = 0. 2

Teorema 5.3
Si x 1 y ( < 0 > 1), se tiene

(1 + x) 1 + x.

 Prueba. Si 1 + x < 0, la desigualdad es evidente, pues el primer miembro es


no negativo.
Si 1 + x 0, es decir x 1, consideremos dos casos:

Sea > 1, entonces por el teorema 5.2

1 1
(1 + x) 1 + x = 1 + x,

de donde

1 + x (1 + x) (1 + x) 1 + x.
72 5. Desigualdad de Bernoulli y la Media Potencial

Sea < 0, tomemos un entero n positivo tal que n < 1, luego por el teorema
5.2
 

(1 + x) n 1+ x
n
1
(1 + x) n 1 + n x.
1 x
n

La igualdad solo es posible si x = 0. 2

5.1. Media Potencial


Definici
on 5.1 : Dados x1 , x2 , . . . , xn n
umeros reales positivos, el n
umero
  1
x1 + x2 + + xn
M =
n

umeros x1 , x2 , , xn . En particular
es la media potencial de grado de los n
x1 + x2 + + xn
M1 = es la media aritmetica,
n
 2 1
x1 + x22 + + x2n 2
M2 = es la media cuadratica,
n
 1 
1 1
x1 + x1 2 + + xn
M1 = es la media armonica.
n

Teorema 5.4
Si x1 , x2 , . . . , xn n
umeros reales positivos y < 0 < , entonces

M MG M .

 Prueba. Como MG MA, entonces


p x1 + x2 + + xn
n
x1 x2 xn .
n
1 1
De < 0 < , tenemos
< 0, entonces elevando a la potencia
5.1. Media Potencial 73


 1

x 1 + x 2
1 + + xn
( x1 x2 xn )
n
n
 1
x1 + x2 + + xn
n
x1 x2 xn = M ,
n

de donde M MG.
As mismo
q
x1 + x2 + + xn
x1 x2 xn
n
; > 0,
n
1
elevando a la potencia

q  1 ! 1
n x1 + x2
++ xn
x1 x2 xn
n
! 1
x1 + x2 + + xn
n
x1 x2 xn = M ,
n

de donde MG M .

M MG M .

2
Teorema 5.5
Si x1 , x2 , . . . , xn son n
umeros reales positivos y < , se tiene

M M .

La igualdad ocurre si y solo si x1 = x2 = = xn .

 Prueba. El teorema 5.4 demuestra cuando y tienen diferentes signos. Ahora


veamos cuando tienen el mismo signo.
Supongamos que 0 < < y haciendo
  1
x1 + x2 + + xn
M = = q,
n
74 5. Desigualdad de Bernoulli y la Media Potencial

entonces
      1
x1 x2 xn
+ ++
M M q q q
= = .
M q n

Tomando
     
x1 x2 xn
d1 = ; d2 = ; ; dn = ,
q q q

obtenemos

1
M d1 + d2 + + dn

= .
q n

Pero

      1
x1 x2 xn
  1
q + ++
d1 + d2 + + dn q q
=



n n

  1
1 x1 + x2 + + xn 1
= = q = 1,
q n q

d1 + d2 + + dn
resulta = 1, es decir
n
d1 + d2 + + dn = n.

Haciendo

d1 = 1 + k1 ; d2 = 1 + k2 ; . . . ; dn = 1 + kn ,

entonces k1 + k2 + + kn = 0.

Como > 1, pues < , se tiene

5.1. Media Potencial 75


1 = (1 + k1 ) 1+ k1


2 = (1 + k2 ) 1 + k2

.. ..
. .

n = (1 + kn ) 1 + kn .

Sumando miembro a miembro, tenemos

1 + 2 + + n n + (k1 + k2 + + kn )
| {z }
0

1 + 2 + + n

n

+ 2 + + n
1 1
n

1
+ 2 + + n

1 1,
n

de donde

M
1 M q M M .
q
La igualdad M = M se cumple si y solo si

k1 = k2 = = kn = 0, luego
d1 = d2 = = dn = 1, y por consiguiente
x1 = x2 = = xn .


Falta demostrar el caso < < 0, pero de < < 0 se tiene 0 < < 1, y se

repite el caso anterior.

M M .

2
76 5. Desigualdad de Bernoulli y la Media Potencial

5.2. Aplicaciones
1. Demuestre que si x3 + y 3 + z 3 = 2187, siendo x, y, z n
umeros reales positivos,
entonces

x + y + z 27.

 Prueba. Como M1 M3 , entonces:


 1/3
x+y+z x3 + y 3 + z 3

3 3
(x + y + z)3 x3 + y 3 + z 3

27 3
(x + y + z) 9(x + y 3 + z 3 )
3 3

(x + y + z)3 32 37 = 39
x + y + z 33 .

Por lo tanto x + y + z 27.


La igualdad ocurre si y solo si x = y = z = 9. 2

2. Si f (x, y, z) = x3 +y 3 +z 3 , con x, y, z n
umeros reales positivos y x+y+z = 3
3,
entonces el mnimo valor de f es:

Solucion. Como x, y, z > 0 y tenemos que buscar una relacion entre x+y+z
y x3 + y 3 + z 3 , entonces aplicamos la Media Potencial, veamos:
 1/3
x+y+z x3 + y 3 + z 3

3 3
(x + y + z)3 x3 + y 3 + z 3

27 3
3 3
( 3)
x3 + y 3 + z 3
9
1
x3 + y 3 + z 3
3

3
1 3
Por lo tanto el mnimo de f es y ocurre si y solo si x = y = z = . 2
3 3
5.2. Aplicaciones 77

3. Sean x, y, z reales no negativos, pruebe que

x5 + y 5 + z 5 5(x + y + z) 12.

 Prueba. Como en la desigualdad aparecen suma y suma de quintas,


entonces podemos aplicar la desigualdad de Bernoulli o la media potencial;
veamos utilizando la desigualdad de Bernoulli:

(1 + a)5 1 + 5a; a 1.

Haciendo 1 + a = x a = x 1, entonces

x5 1 + 5(x 1)
x5 5x 4; x 0; similarmente
y 5 5y 4
z 5 5z 4

sumando miembro a miembro y efectuando tenemos

x5 + y 5 + z 5 5x + 5y + 5z 12.

x5 + y 5 + z 5 5(x + y + z) 12.

La igualdad ocurre si y solo si x = y = z = 1. 2


78 5. Desigualdad de Bernoulli y la Media Potencial
CAPITULO 6
DESIGUALDAD DE
REORDENAMIENTOS

Teorema 6.1 (Teorema de Abel)


Sean (x1 , x2 , . . . , xn ), (y1 , y2 , . . . , yn ), dos n-uplas de n
umeros reales y denotamos

ck = y1 + y2 + + yk ; (k = 1, 2, . . . n),

entonces

x1 y1 + x2 y2 + + xn yn = (x1 x2 )c1 + (x2 x3 )c2 + (xn1 xn )cn1 + xn cn . (6.1)

 Prueba. Efectuando el segundo miembro de (6.1)

(x1 x2 )c1 + (x2 x3 )c2 + (xn1 xn )cn1 + xn cn = c1 x1 + (c2 c1 )x2 +


+(cn cn1 )xn
= y1 x1 + y2 x2 + + yn xn .
2
Teorema 6.2 (Desigualdad de Abel)
Sean x1 , x2 , . . . , xn , y y1 y2 yn 0 n umeros reales y denotemos
Xk
sk = xi con M = max {s1 , s2 , . . . , sn } y m = mn {s1 , s2 , . . . , sn }, entonces
i=1

my1 x1 y1 + x2 y2 + + xn yn My1 .

79
80 6. Desigualdad de Reordenamientos

 Prueba. Como y1 y2 yn 0, entonces aplicando el teorema de Abel


tenemos
n
X n
X
xi yi = (yi yi+1 )si ; yn+1 = 0
i=1 i=1
Xn n
X
(yi yi+1 )m = m (yi yi+1 ) = my1 ,
i=1 i=1

de donde
n
X
xi yi my1 .
i=1

Similarmente
n
X n
X
xi yi = (yi yi+1 )si ; yn+1 = 0
i=1 i=1
n
X n
X
(yi yi+1 )M = M (yi yi+1 ) = My1 ,
i=1 i=1

de donde
n
X
xi yi My1 .
i=1

n
X
my1 xi yi My1 .
i=1

2
Aplicaci on 6.1
Sean a1 , a2 , , an y b1 b2 bn 0 n
umeros reales positivos tales que

a1 a2 ak b1 b2 bk , k {1, 2, . . . , n}.

Pruebe que

a1 + a2 + + an b1 + b2 + + bn .
81

 Prueba. Aplicando el teorema de Abel

n
X n
X n
X  
ai
ai bi = bi 1
i=1 i=1 i=1
b i
   
a1 a1 a2
= (b1 b2 ) 1 + (b2 b3 ) + 2 +
b1 b1 b2
n1
! n
!
X ai X ai
+(bn1 bn ) n + 1 + bn n .
b
i=1 i
b
i=1 i

k
X ai
Analizando cada suma ; k = 1, 2, . . . , n, mediante la propiedad MA MG
i=1
bi
r
a1 a2 ak a1 a2 ak
+ ++ k k
k,
b1 b2 bk b1 b2 bk

luego
k
!
X ai
k 0.
i=1
bi

n
X n
X
ai bi 0.
i=1 i=1

2
Teorema 6.3 (Desigualdad de Reordenamientos)
Sean (a1 , a2 , . . . , an ) y (b1 , b2 , . . . , bn ) dos sucesiones crecientes de n
umeros reales y
sea (bi1 , bi2 , . . . , bin ) una permutacion de (b1 , b2 , . . . , bn ), entonces

a1 b1 + a2 b2 + + an bn a1 bi1 + a2 bi2 + + an bin .

Si (a1 , a2 , . . . , an ) es creciente y (b1 , b2 , . . . , bn ) es decreciente, entonces

a1 b1 + a2 b2 + + an bn a1 bi1 + a2 bi2 + + an bin .

 Prueba. Veamos el caso

a1 a2 an y b1 b2 bn .
82 6. Desigualdad de Reordenamientos

Aplicando el teorema de Abel

n
X n
X n
X
ak bk ak bik = ak (bk bik )
k=1 i=1 k=1

= (a1 a2 ) (b1 bi1 ) + (a2 a3 ) (b1 + b2 bi1 bi2 ) +


| {z } | {z }
(+) (+)
n1 n1
!
X X
+ (an1 an ) bk bik
k=1 k=1
| {z }
(+)
n n
!
X X
+ an bk bik 0,
k=1 k=1
| {z }
(+)

pues para cada k = 1, 2, . . . , n, tenemos que


k
X k
X
bj bij ,
j=1 j=1

ya que b1 b2 bn .
La segunda parte del teorema se prueba similarmente. 2

Aplicaci
on 6.2
Pruebe que
a b c 3
+ +
b+c c+a a+b 2
para todo a, b, c n
umeros reales positivos.

 Prueba. Esta desigualdad ya fue probada por el Lema de Titu y aplicando


MA MG, ahora veamos aplicando reordenamientos.
En efecto, por la simetra del primer miembro de la desigualdad, podemos asumir
un orden, como a b c, entonces a + b a + c b + c y
1 1 1
.
b+c a+c a+b
83

Luego

a b c b c a
+ + + +
b+c c+a a+b b+c c+a a+b
a b c c a b
+ + + + ,
b+c c+a a+b b+c c+a a+b
sumando miembro a miembro, tenemos:
 
a b c b+c c+a a+b
2 + + + + = 3,
b+c c+a a+b b+c c+a a+b

de donde

a b c 3
+ + .
b+c c+a a+b 2
2

Colorario 6.1 Para toda permutacion (a1 , a2 , . . . , an ) de (a1 , a2 , . . . , an ), se tiene

a21 + a22 + + a2n a1 a1 + a2 a2 + + an an .

Colorario 6.2 Para toda permutacion (a1 , a2 , . . . , an ) de (a1 , a2 , . . . , an ), se tiene

a1 a2 an
+ ++ n.
a1 a2 an

Teorema 6.4 (Desigualdad de Chebyshev)


Sean a1 a2 an y b1 b2 bn , entonces

  
a1 b1 + a2 b2 + + an bn a1 + a2 + + an b1 + b2 + + bn
.
n n n

Si una de las sucesiones es creciente y la otra, decreciente, el sentido de la de-


sigualdad cambia

  
a1 b1 + a2 b2 + + an bn a1 + a2 + + an b1 + b2 + + bn
.
n n n
84 6. Desigualdad de Reordenamientos

 Prueba. Veamos el caso cuando las dos sucesiones son crecientes. En efecto,
por la desigualdad de reordenamientos, tenemos

a1 b1 + a2 b2 + + an bn = a1 b1 + a2 b2 + + an bn ,
a1 b1 + a2 b2 + + an bn a1 b2 + a2 b3 + + an b1 ,
a1 b1 + a2 b2 + + an bn a1 b3 + a2 b4 + + an b2 ,
.. .. ..
. . .
a1 b1 + a2 b2 + + an bn a1 bn + a2 b1 + + an bn1 .

Sumando miembro a miembro, tenemos

n(a1 b1 + a2 b2 + + an bn ) (a1 + a2 + + an )(b1 + b2 + + bn ).

El otro caso se prueba de manera similar. 2


Aplicaci
on 6.3
Pruebe que

a3 b3 c3 (a + b + c)3
+ +
x y z 3(x + y + z)

umeros reales positivos con a b c


para todo a, b, c, x, y, z n y z y x.

 Prueba. Sean las ternas


 
a2 b2 c2
, , ; (a, b, c),
x y z

a2 b2 c2
como a b c y z y x, entonces .
x y z
Aplicando el teorema de Chebyshev
2
a2 b2 c2 a b2 c2
a+ b+ c  
x y z x + y + z a+b+c


3 3 3
  
a3 b3 c3 a2 b2 c2 a+b+c
+ + + + .
x y z x y z 3
85

Luego es suficiente demostrar


 2  
a b2 c2 a+b+c (a + b + c)3
+ + .
x y z 3 3(x + y + z)

Pero sabemos por el lema de Titu

a2 b2 c2 (a + b + c)2
+ + .
x y z x+y+z
 
a+b+c
Multiplicando por , tenemos
3
 2  
a b2 c2 a+b+c (a + b + c)3
+ + .
x y z 3 3(x + y + z)
2
Aplicaci
on 6.4
Pruebe que
x y z 3
+ +
y+z z+x x+y 2
umeros reales positivos tales que xyz = 1 y 1.
para todo , x, y, z n

 Prueba. Por la simetra, podemos asumir el orden x y z, entonces


1 1
z+xy+z
z+x y+z
1 1
x+y x+z ,
x+y x+z
1 1 1
de donde .
x+y x+z y+z
z y x
Luego y ademas
x+y x+z y+z
x1 y 1 z 1 .

Utilizando Chebyshev

 
x y z 1 1  x y z
+ + x + y 1 + z 1 + + .
y+z z+x x+y 3 y+z z+x x+y
86 6. Desigualdad de Reordenamientos

Pero sabemos que

x1 + y 1 + z 1 p
3 (xyz)1 = 1
3
x y z 3
+ +
y+z z+x x+y 2

x y z 3
+ + .
y+z z+x x+y 2
2
Aplicaci
on 6.5
Pruebe que

a3 b3 c3 d3 1
+ + +
b+c+d a+c+d a+b+d a+b+c 3
para todo a, b, c, d reales positivos con ab + bc + cd + da = 1.

 Prueba. Por la simetra del primer miembro de la desigualdad, supongamos


sin perdida de generalidad que a b c d, y haciendo

m = b + c + d,
n = a + c + d,
p = a + b + d,
q = a+b+c
1 1 1 1
mnpq ,
m n p q

luego, aplicando Chebyshev, tenemos:

a3 b3 c3 d3
+ + +
m n p q
 
1 3 3 3 3 1 1 1 1
(a + b + c + d ) + + +
4 m n p q
 
1 2 2 2 2 1 1 1 1
(a + b + c + d ) (a + b + c + d) + + +
16 m n p q
87

como m + n + p + q = 3(a + b + c + d), entonces

a3 b3 c3 d3
+ + +
m n p q
  
1 2 m + n + p + q 1 1 1 1
(a + b2 + c2 + d2 ) + + +
16 3 m n p q
  
1 2 2 2 2 1 1 1 1 1
= (a + b + c + d ) (m + n + p + q) + + +
16 3 m n p q
1 2 1
(a + b2 + c2 + d2 ) (16)
16 3
1 2
= (a + b2 + c2 + d2 ) .
3

Nos falta acotar a2 + b2 + c2 + d2 y como tenemos la relacion ab + bc + cd + da = 1,


entonces tomamos las cuaternas (a, b, c, d) y (b, c, d, a) para aplicar Cauchy-Schwarz;
en efecto:

(a2 + b2 + c2 + d2 ) (b2 + c2 + d2 + a2 ) (ab + bc + cd + da)2


2
(a2 + b2 + c2 + d2 ) (1)2
a2 + b2 + c2 + d2 1.

a3 b3 c3 d3 1
+ + + .
m n p q 3
2
88 6. Desigualdad de Reordenamientos
CAPITULO 7
DESIGUALDAD CON FUNCIONES
CONVEXAS

Las funciones convexas cumplen un rol muy importante en la matematica, espe-


cialmente en la lnea de optimizacion, ya que en estos tiempos se estan estudiando
modelos matematicos en ingeniera, economa, etc.
En este captulo estudiaremos estas funciones para obtener una desigualdad muy
importante llamada la desigualdad de Jensen, la cual se ha utilizado en muchas
olimpiadas matematicas internacionales.

7.1. Funci
on convexa
Definicion 7.1: Sea f una funcion real de variable real, definida sobre [a, b] R. f
es llamada una funcion convexa sobre [a, b] si y solo si para cada x, y [a, b] y para
todo 0 t 1, se tiene

f (tx + (1 t)y) tf (x) + (1 t)f (y).

A continuacion utilizaremos el concepto de primera y segunda derivada (se puede


cualquier libro de Analisis Matematico).

Teorema 7.1
Si f es una funcion real definida sobre [a, b] R y f (x) > 0 para todo x ha, bi,
entonces f es una funcion convexa sobre [a, b].

89
90 7. Desigualdad con Funciones Convexas

 Prueba. Debemos probar que para todo x [a, b] y para todo t [0, 1], se
cumple

f (tx + (1 t)y) tf (x) + (1 t)f (y). (7.1)

Supongamos que t y y son constantes; definimos

g(x) = tf (x) + (1 t)f (y) f (tx + (1 t)y).

Derivando con respecto a x

g (x) = tf (x) tf (tx + (1 t)y) = t(f (x) f (tx + (1 t)y)).

Como f (x) > 0, x [a, b], entonces f es creciente en [a, b], de donde tenemos
que

g (x) 0 si x y; y

g (x) 0 si x y,

el mnimo de la funcion es g(y), evaluando en (7.1) tenemos g(y) = 0.


Luego

g(x) g(y), x [a; b],

de donde

g(x) 0 x [a; b].

Reemplazando

tf (x) + (1 t)f (y) f (tx + (1 t)y) 0.

f (tx + (1 t)y) tf (x) + (1 t)f (y), x [a; b], t [0; 1]. [9]

2
7.1. Funcion convexa 91

Ejemplo 7.1
La funcion f (x) = x , con x n
umero real, es convexa.

4 3 2 1 1 2 3 4
1

pues f (x) = x > 0, para todo x n


umero real.
Teorema 7.2
Si f es convexa sobre [a, b], entonces para cada x, y [a, b] se tiene
 
x+y 1
f (f (x) + f (y)) .
2 2

1
 Prueba. Es suficiente tomar t = en la definicion. 2
2
Definicion 7.2 : Una funcion f real definida sobre [a, b] R es llamada funcion
concava sobre [a, b] si y solo si para cada x, y [a, b] y para todo 0 t 1 se tiene

f (tx + (1 t)y) tf (x) + (1 t)f (y).

Teorema 7.3 (Desigualdad de Jensen) n


X
Si f es convexa sobre [a, b], entonces para cada t1 , t2 , . . . , tn [0, 1] con ti = 1 y
i=1
para cada x1 , x2 , . . . , xn [a, b], se tiene

f (t1 x1 + t2 x2 + + tn xn ) t1 f (x1 ) + t2 f (x2 ) + + tn f (xn ).


92 7. Desigualdad con Funciones Convexas

 Prueba. Demostraremos por induccion. Para n = 2 es el teorema anterior.


Supongamos que se cumpla para (n 1), veamos para n.
Como

h i
t1 t2 tn
t1 x1 + t2 x2 + + tn1 xn1 + tn xn = (1 tn ) x
1tn 1
+ x
1tn 2
++ x
1tn1 n1

+tn xn ,

entonces

   
t1 tn1
f (t1 x1 + t2 x2 + + tn xn ) = f (1 tn ) x1 + + xn1 + tn xn
1 tn 1 tn
 
t1 tn1
(1 tn )f x1 + + xn1
1 tn 1 tn
+tn f (xn ), f convexa
 
t1 tn1
(1 tn ) f (x1 ) + + f (xn1 )
1 tn 1 tn
+tn f (xn ),
= t1 f (x1 ) + t2 f (x2 ) + + tn f (xn ),

de donde se tiene

f (t1 x1 + t2 x2 + + tn xn ) t1 f (x1 ) + t2 f (x2 ) + + tn f (xn ).

2
1
En particular, si t1 = t2 = = tn = , se tiene
n
 
x1 + x2 + + xn 1
f (f (x1 ) + f (x2 ) + + f (xn )) .
n n
Similarmente si f es concava entonces

f (t1 x1 + t2 x2 + + tn xn ) t1 f (x1 ) + t2 f (x2 ) + + tn f (xn ).

Teorema 7.4
Si f es una funcion real definida sobre [a, b] R, y f (x) < 0 para todo x ha, bi,
entonces f es una funcion concava sobre [a, b].
7.1. Funcion convexa 93

 Prueba. La prueba es similar al teorema 7.1. 2


Aplicaci
on 7.1 n
X
Pruebe que dados x1 , x2 , . . . , xn , t1 , t2 , . . . , tn n
umeros reales positivos con ti = 1,
i=1
entonces

xt11 xt22 xtnn t1 x1 + t2 x2 + + tn xn .

 Prueba. Sabemos que f (x) = x , x R, es convexa y ademas


ti
xtii = ln xi = ti ln xi ,

entonces

xt11 xt22 xtnn = t1 ln x1 t2 ln x2 tn ln xn


= t1 ln x1 +t2 ln x2 ++tn ln xn

aplicando la Desigualdad de Jensen

t1 ln x1 +t2 ln x2 ++tn ln xn t1 ln x1 + t2 ln x2 + + tn ln xn
= t1 x1 + t2 x2 + + tn xn .
2
Aplicacion 7.2
1 1
umeros reales positivos, que satisfacen + = 1, entonces
Sean x, y, a, b n
a b
1 1
xy xa + y b.
a b

 Prueba. De la aplicacion anterior se tiene


1  1b 1 1
xy = (xa ) a y b xa + y b .
a b
2
Aplicaci on 7.3
Si x1 , x2 , . . . , xn son n
umeros reales positivos. Pruebe que
1 1 1 n
+ ++ .
1 + r1 1 + r2 1 + rn n
r1 r2 rn + 1
94 7. Desigualdad con Funciones Convexas

 Prueba. Por la forma de cada fraccion del primer miembro, definimos la funcion
1
f (x) = ; x R+ .
1 + x
Veamos que es convexa.
Derivando, tenemos
x
f (x) = y
(1 + x )2
x (x 1)
f (x) = > 0, x > 0.
(1 + x )3
Por lo tanto f es convexa. Luego, podemos aplicar la Desigualdad de Jensen

 
x1 + x2 + + xn 1
f (f (x1 ) + f (x2 ) + + f (xn ))
n n
 
1 1 1 1 1
x1 +x2 ++xn + ++
1+ n n 1 + x1 1 + x2 1 + xn

haciendo xi = ri , i = 1, 2, . . . , n, tenemos
n 1 1 1
+ ++ .
n
r1 r2 rn + 1 1 + r1 1 + r2 1 + rn
2
Aplicacion 7.4
Utilizando la desigualdad de Jensen, pruebe que
x1 + x2 + + xn
n x1 x2 xn ,
n
siendo x1 , x2 , , xn n
umeros reales positivos.

 Prueba. Definamos f (x) = ln x, x > 0.


Derivando
1
f (x) = ,
x
1
f (x) = < 0,
x2
7.1. Funcion convexa 95

entonces f es concava. Luego


 
1 x1 + x2 + + xn
(ln x1 + ln x2 + + ln xn ) ln
n n
 
1 x1 + x2 + + xn
ln (x1 x2 xn ) ln
n n
 
x1 + x 2 + + xn
ln n x1 x2 xn ln
n
x1 + x2 + + xn
n x1 x2 xn .
n
2
Aplicaci
on 7.5
(IMO 2001). Sean a, b, c n
umeros reales positivos, pruebe que
a b c
+ + 1.
a2 + 8bc b2 + 8ca c2 + 8ab

1
 Prueba. Definimos la funcion f (x) = , x > 0. La funcion es convexa, y sin
x
perdida de generalidad asumimos que a + b + c = 1, luego

af (a2 + 8bc) + bf (b2 + 8ca) + cf (c2 + 8ab)


f a(a2 + 8bc) + b(b2 + 8ca) + c(c2 + 8ab) ,


| {z }
x

luego basta demostrar que


1
f (x) = 1
x
x1
a3 + b3 + c3 + 24abc 1
a3 + b3 + c3 + 24abc (a + b + c)3
a3 + b3 + c3 + 24abc a3 + b3 + c3 + 3(a + b)(b + c)(c + a)
(a + b)(b + c)(c + a) 8abc.

La igualdad ocurre si y solo si a = b = c. 2


96 7. Desigualdad con Funciones Convexas
CAPITULO 8

ESPACIO METRICO

Una de las operaciones principales del analisis es el paso al lmite. Esta operacion
descansa sobre el hecho de que en la recta numerica esta definida la distancia entre
dos puntos.
Es impresionante ver que muchos resultados principales del analisis no tienen
nada que ver con la naturaleza algebraica del conjunto de los n umeros reales, es
decir, solo se apoya en las propiedades de distancia y con ello llegamos al concepto
de Espacio Metrico, que es uno de los conceptos mas importantes de la matematica
moderna.

Definicion 8.1 : Un espacio metrico es un par (X, d) compuesto de un conjunto


(espacio) X 6= de elementos (puntos) y de una distancia, es decir una funcion

d : X X R
(x, y) 7 d(x, y)

no negativa, que verifica las tres condiciones siguientes:

(1) d(x, y) = 0 x = y.

(2) d(x, y) = d(y, x), (axioma de simetra.)

(3) d(x, z) d(x, y) + d(y, z), (axioma triangular.)

Muchas veces lo denotaremos simplemente como X al espacio metrico (X, d),


por comodidad en la notacion.

97
98 8. Espacio Metrico

Mencionaremos algunos ejemplos que desempe


nan un papel importante en el
analisis.

1. Sea X 6= un conjunto arbitrario con


(
0, x = y
d(x, y) =
6 y
1, x =

(X, d) es un espacio metrico llamado de puntos aislados.

2. El conjunto de los n
umeros reales con la distancia

d(x, y) = |x y|

forma el espacio metrico (R, d).


Veamos que cumple los axiomas correspondientes.

 Prueba.

(1)
d(x, y) = 0 |x y| = 0
xy =0
x = y.

(2) d(x, y) = |x y| = |y x| = d(x, y).


(3) d(x, z) = |x z| = |(x y) + (y z)| |x y| + |y z| = d(x, y) + d(y, z)
de donde

d(x, z) d(x, y) + d(y, z).

3. El conjunto

Rn = {x = (x1 , x2 , . . . , xn )/ x1 , x2 , . . . , xn R}
99

con la distancia
v
u n
uX
d(x, y) = t (yk xk )2
k=1

es el espacio metrico (Rn , d).


Para verificar que es espacio metrico, veamos que se cumple el axioma trian-
gular (3), ya que (1) y (2) son directos.
Sean

x = (x1 , x2 , . . . , xn ), y = (y1 , y2 , . . . , yn ), z = (z1 , z2 , . . . , zn ),

entonces para que se cumpla

d(x, z) d(x, y) + d(y, z)

se tiene que cumplir la desigualdad


v v v
u n u n u n
uX uX uX
t (zk xk )2 t (yk xk )2 + t (zk yk )2 . (8.1)
k=1 k=1 k=1

Si tomamos yk xk = ak y zk yk = bk zk xk = ak + bk , entonces (8.1)


tiene la forma
v v v
u n u n u n
uX uX uX
t (ak + bk )2 t ak + t
2
b2k . (8.2)
k=1 k=1 k=1

Esta desigualdad se deduce de la desigualdad de Cauchy-Schwarz

n
!2 n n
X X X
ak bk a2k b2k . (8.3)
k=1 k=1 k=1
100 8. Espacio Metrico

En efecto:

n
X n
X n
X n
X
2
(ak + bk ) = a2k +2 ak bk + b2k
k=1 k=1 k=1 k=1
v
n u n n n
X uX X X
2
ak + 2 t 2
ak 2
bk + b2k
k=1 k=1 k=1 k=1
v v 2
u n u n
uX uX
= t a2k + t b2k ,
k=1 k=1

de donde
v v v
u n u n u n
uX uX uX
t (ak + bk )2 t a2k + t b2k .
k=1 k=1 k=1

4. Considerando el mismo conjunto Rn , pero definiendo la distancia

n
X
d1 (x, y) = |yk xk |.
k=1

Tambien es un espacio metrico (Rn , d1 ).

5. Considerando nuevamente Rn , y definiendo una nueva distancia

d2 (x, y) = max |yk xk |.


1kn

Tambien es un espacio metrico (Rn , d2 ).

6. L2 = {x = (x1 , x2 , . . . , xn , . . .)} tal que xk R para todo k N tal que


X
x2k < y con distancia
k=1
v
u
uX
d(x, y) = t (yk xk )2 .
k=1
101

La funcion d(x, y) as definida tiene sentido para x, y L2 ya que la serie


v
u
uX X X
t (yk xk )2 converge siempre que x2k < y yk2 < , esto ocurre
k=1 k=1 k=1
2
pues (xk yk ) 2(x2k + yk2 ).
Al mismo tiempo vemos que si (x1 , x2 , . . . , xn , . . .), (y1 , y2, . . . , yn , . . .) L2 ,
tambien (x1 + y1 , x2 + y2 , . . . , xn + yn , . . .) L2 .

7. El conjunto Rn , con la distancia

n
!1/p
X
dp (x, y) = |yk xk |p ,
k=1

donde p es un n umero fijo arbitrario mayor que 1 (p > 1), representa el espacio
n
metrico (R , dp ). Para comprobar que es espacio metrico, veamos que cumple
los tres axiomas.
(1) y (2) son obvias, veamos el axioma triangular (3), es decir, se debe cumplir

n
!1/p n
!1/p n
!1/p
X X X
|zk xk |p |zk yk |p + |yk xk |p .
k=1 k=1 k=1

Haciendo

zk yk = ak y yk xk = bk zk xk = ak + bk ,

tenemos

n
!1/p n
!1/p n
!1/p
X X X
|ak + bk |p |ak |p + |bk |p .
k=1 k=1 k=1

A continuacion estudiaremos esta desigualdad, llamada desigualdad de Minkon-


ski; previamente veremos la desigualdad de Holder.
102 8. Espacio Metrico

8.1. Desigualdad de H
older
1
Sean x1 , x2 , . . . , xn , y1 , y2 , . . . , yn n
umeros reales positivos y a, b > 0 tal que +
a
1
= 1, entonces
b
n n
!1/a n
!1/b
X X X
xi yi xai yib .
i=1 i=1 i=1

n
X n
X
 Prueba. Supongamos que xai = yib = 1, usando el hecho que
i=1 i=1
1 1
xi yi xai + yib (ejercicio demostrado anteriormente),
a b

entonces
n
X n n
1X a 1X b 1 1
xi yi x + y = + = 1. (8.4)
i=1
a i=1 i b i=1 i a b

n
X n
X
Ahora supongamos que xai =M y yib = N.
i=1 i=1
xi yi
Sea xi = 1/a , yi = 1/b , entonces
M N
Xn

n
xai
X a i=1 M
x i = = = 1,
i=1
M M
n
X
n
yib
X b i=1 N
yi = = = 1.
i=1
N N

Luego
n
X n
X Xn
xi yi 1
x iy i = = 1/a xi yi. (8.5)
i=1 i=1
M 1/a N 1/b M N 1/b i=1
8.2. Desigualdad de Minkonski 103

n
X
De (8.4) se tiene que xi yi 1, entonces de (8.5) tenemos
i=1
X n
1
xi yi 1
M 1/a N 1/b i=1
n
X
xi yi M 1/a N 1/b .
i=1

Reemplazando

n n
!1/a n
!1/b
X X X
xi yi xai yib .
i=1 i=1 i=1

En particular si a = b = 2, tenemos la desigualdad de Cauchy-Schwarz.

8.2. Desigualdad de Minkonski


Sean a1 , a2 , . . . , an , b1 , b2 , . . . , bn n
umeros positivos y p > 1, entonces

n
!1/p n
!1/p n
!1/p
X X X
(ak + bk )p (ak )p + (bk )p .
k=1 k=1 k=1

 Prueba. Como

(ak + bk )p = (ak + bk )(ak + bk )p1


= ak (ak + bk )p1 + bk (ak + bk )p1 ,

entonces
n
X n
X n
X
(ak + bk )p = ak (ak + bk )p1 + bk (ak + bk )p1 , (8.6)
k=1 k=1 k=1

aplicando la desigualdad de Holder en cada termino del segundo miembro, sa-


1 1
biendo que p > 1 y + = 1
p q
104 8. Espacio Metrico

n n
!1/p n
!1/q
X X X
ak (ak + bk )p1 (ak )p (ak + bk )q(p1) ,
k=1 k=1 k=1
n n
!1/p n
!1/q
X X X
bk (ak + bk )p1 (bk )p (ak + bk )q(p1) ,
k=1 k=1 k=1

ademas q(p 1) = p y reemplazando en (8.6)

n n
!1/p n
!1/q
X X X
(ak + bk )p (ak )p (ak + bk )p
k=1 k=1 k=1
n
!1/p n
!1/q
X X
+ (bk )p (ak + bk )p
k=1 k=1

n n
!1/q n !1/p n
!1/p
X X X X
(ak + bk )p (ak + bk )p (ak )p + (bk )p
k=1 k=1 k=1 k=1

n
!11/q n
!1/p n
!1/p
X X X
(ak + bk )p (ak )p + (bk )p
k=1 k=1 k=1
n
!1/p n
!1/p n
!1/p
X X X
(ak + bk )p (ak )p + (bk )p .
k=1 k=1 k=1

2
Teorema 8.1
Dados los puntos A, B, C del plano, se tiene que

AB AC + CB,

la igualdad ocurre si C AB.


Aplicaci on 8.1
Sean a, b, c los lados de un triangulo. Pruebe que
a b c
+ + < 2.
b+c c+a a+b
8.2. Desigualdad de Minkonski 105

 Prueba. Como a < b + c, b < c + a, c < a + b,

a b c 2a 2b 2c
+ + = + +
b+c c+a a+b 2b + 2c 2c + 2a 2a + 2b
2a 2b 2c
= + +
(b + c) +(b + c) (c + a) +(c + a) (a + b) +(a + b)
| {z } | {z } | {z }
2a 2b 2c
< + +
a+b+c a+b+c a+b+c
2(a + b + c)
= = 2,
a+b+c

de donde

a b c
+ + < 2.
b+c c+a a+b
2
Aplicaci
on 8.2
Pruebe que
p p
2|x + y + z| x2 + y 2 + y 2 + z 2 + x2 + z 2 ,

para todo x, y, z n
umeros reales.

 Prueba. Escribiendo adecuadamente el punto

(x + y + z, x + y + z) = (x, y) + (y, z) + (z, x).

Pero

d(x + y + z, x + y + z) = d((x, y) + (y, z) + (z, x))


d(x, y) + d(y, z) + d(z, x),

de donde
p p
2|x + y + z| x2 + y 2 + y 2 + z 2 + x2 + z 2 .

2
106 8. Espacio Metrico

Aplicaci
on 8.3
Pruebe que

x4/3 y 4/3
M = +
x4/3 + (x2 + y 2)1/3 (x + z)2/3 y 4/3 + (y 2 + z 2 )1/3 (y + z)2/3
z 4/3
+ 1
z 4/3 + (z 2 + x2 )1/3 (x + y)2/3

para todo x, y, z n
umeros reales positivos.

 Prueba. Haciendo x = a3 , y = b3 , z = c3 , por la desigualdad de Holder tenemos

1/3 1/3
(x2 + y 2 ) (x + z)2/3 = ((a2 )3 + (b2 )3 ) ((c2 )3/2 + (a2 )3/2 )2/3
a2 c2 + b2 a2 = (xy)2/3 + (xz)2/3
1/3
x4/3 + (x2 + y 2 ) (x + z)2/3 x4/3 + (xy)2/3 + (xz)2/3
x4/3 x4/3

x4/3 + (x2 + y 2 )1/3 (x + z)2/3 x4/3 + (xy)2/3 + (xz)2/3

x2/3
= 2/3
x + y 2/3 + z 2/3

similarmente

y 4/3 y 2/3
,
y 4/3 + (y 2 + z 2 )1/3 (y + z)2/3 x2/3 + y 2/3 + z 2/3

z 4/3 z 2/3
,
z 4/3 + (z 2 + x2 )1/3 (z + x)2/3 x2/3 + y 2/3 + z 2/3

sumando miembro a miembro las tres desigualdades, en el segundo miembro se


obtiene el valor de 1, de donde

M 1.

2
8.2. Desigualdad de Minkonski 107

Aplicaci on 8.4
Sean a, b, c, d n
umeros reales. Determine el mnimo de

p p
F = (a + 1)2 + 2(b 2)2 + (c + 3)2 + (b + 1)2 + 2(c 2)2 + (d + 3)2
p p
+ (c + 1)2 + 2(d 2)2 + (a + 3)2 + (d + 1)2 + 2(a 2)2 + (b + 3)2 .

 Prueba. Sea S = a + b + c + d y aplicando la desigualdad de Minkowski:

p p
S1 = (a + 1)2 + 2(b 2)2 + (c + 3)2 + (b + 1)2 + 2(c 2)2 + (d + 3)2
p
(a + 1 + b + 1)2 + 2(b 2 + c 2)2 + (c + 3 + d + 3)2
p
= (a + b + 2)2 + 2(b + c 4)2 + (c + d + 6)2
p p
S2 = (c + 1)2 + 2(d 2)2 + (a + 3)2 + (d + 1)2 + 2(a 2)2 + (b + 3)2
p
(c + d + 2)2 + 2(a + d 4)2 + (a + b + 6)2
p
S1 + S2 (a + b + 2)2 + 2(b + c 4)2 + (c + d + 6)2
p
+ (c + d + 2)2 + 2(a + d 4)2 + (a + b + 6)2
(aplicando nuevamente Minkowski)
p
F (a + b + c + d + 4)2 + 2(a + b + c + d 8)2 + (a + b + c + d + 12)2
p
= (s + 4)2 + 2(s 8)2 + (s + 12)2

Efectuando dentro del radical



F s2 + 8s + 16 + 2s2 32s + 128 + s2 + 24s + 144

F 4s2 + 288 288 = 12 2

luego el mnimo valor de F es 12 2 y esto ocurre cuando a = b = c = d = 0. 2
108 8. Espacio Metrico
CAPITULO 9

METODO DE LA SUMA DE
CUADRADOS

Definicion 9.1 : El polinomio f (a1 , a2 , . . . , an ) es homogeneo si todos sus terminos


tienen el mismo grado absoluto. Mencionaremos algunos ejemplos:

1. f (a1 , a2 ) = a21 + a22 + 5a1 a2 , es homogeneo de segundo grado.


2. f (a1 , a2 , a3 ) = a31 + a22 a3 + a1 a2 a3 , es homogeneo de tercer grado.

Sea el polinomio homogeneo f (a1 , a2 , . . . , an ), entonces f (a1 , a2 , . . . , an ) 0,


para todo a1 , a2 , . . . , an reales, significa que la funcion puede expresarse como una
suma f (a1 , a2 , . . . , an ) = p21 + p22 + + p2n , donde pk es una funcion real.
Veamos la desigualdad

A B,

donde A, B son expresiones de variables a, b, c para explicar la idea central.


Si queremos demostrar A B, entonces veremos la diferencia

A B = f (a, b, c) = Sa (b c)2 + Sb (c a)2 + Sc (a b)2 . (9.1)

Si Sa , Sb , Sc 0, entonces

f (a, b, c) = Sa (b c)2 + Sb (c a)2 + Sc (a b)2 0;

esto significa que A B es una desigualdad verdadera.

109
110 9. Metodo de la Suma de Cuadrados

Teorema 9.1
Si a, b, c, Sa , Sb , Sc satisfacen las condiciones

1. Sa + Sb 0, Sb + Sc 0, Sc + Sa 0, y

2. (a b c a b c) y Sb 0,

entonces

Sa (b c)2 + Sb (c a)2 + Sc (a b)2 0 (9.2)

 Prueba. Sin perdida de generalidad supongamos que a b c, luego

Sa (b c)2 + Sb (c a)2 + Sc (a b)2


= Sa (b c)2 + Sb (c b + b a)2 + Sc (a b)2
= Sa (b c)2 + Sb [(c b)2 + (b a)2 + 2(c b)(b a)] + Sc (a b)2
= Sa (b c)2 + Sb (c b)2 + Sb (b a)2 + 2(c b)(b a)Sb + Sc (a b)2
= (Sa + Sb )(b c)2 + (Sb + Sc )(a b)2 + 2 (c b)(b a) Sb 0.
| {z } | {z } | {z } |{z}
0 0 0 0

La igualdad ocurre si y solo si

(Sa + Sb ) (b c)2 = 0 (Sb + Sc ) (a b)2 = 0 2(c b)(b a)Sb = 0,

de donde se deduce que

(a = b = c) (a = b Sa = Sb = 0) (b = c Sb = Sc = 0)
(Sa = Sb = Sc = 0).

2
Teorema 9.2
Si a, b, c, Sa , Sb , Sc son n
umeros reales que satisfacen las condiciones:
(a b c a b c), Sa 0, Sc 0, Sa + 2Sb 0, Sc + 2Sb 0, entonces

Sa (b c)2 + Sb (c a)2 + Sc (a b)2 0 (9.3)


111

 Prueba. Sin perdida de generalidad supongamos que a b c, entonces


Sa 0, Sc 0, Sa + 2Sb 0, Sc + 2Sb 0.

1. Si Sb 0, entonces (9.3) siempre es verdadero.


2. Si Sb < 0, entonces

Sa (b c)2 + Sb (c a)2 + Sc (a b)2


= Sa (b c)2 + Sb (c b + b a)2 + Sc (a b)2
= (Sb + Sa ) (c b)2 + (Sb + Sc ) (b a)2 + 2Sb (c b)(b a)
(Sb + Sa ) (c b)2 + (Sb + Sc ) (b a)2 + Sb [(c b)2 + (b a)2 ]
= (Sa + 2Sb ) (c b)2 + (Sc + 2Sb ) (b a)2 0.

Analice cuando se da la igualdad. 2


Teorema 9.3
Si a, b, c, Sa , Sb , Sc son n
umeros reales que satisfacen las condiciones:
(a b c a b c), Sa 0, Sb 0 y b2 Sc + c2 Sb 0, entonces

Sa (b c)2 + Sb (c a)2 + Sc (a b)2 0 (9.4)

 Prueba. Sin perdida de generalidad supongamos que a b c, y de Sa 0,


Sb 0 y b2 Sc + c2 Sb 0, entonces

Sa (b c)2 + Sb (c a)2 + Sc (a b)2


"  2 #
ca
= Sa (b c)2 + (b a)2 Sb + Sc ;
ba
 2  c 2
ca
como , entonces
ba b
   
2 2 2 2 2 c 2
Sa (b c) + Sb (c a) + Sc (a b) Sa (b c) + (b a) Sb + Sc
b
 
2
 2 2
 ba 2
= Sa (b c) + Sb c + Sc b 0.
| {z } | {z } b
0 0

Analice cuando se da la igualdad. 2


112 9. Metodo de la Suma de Cuadrados

Teorema 9.4
Si a, b, c, Sa , Sb , Sc son n
umeros reales que satisfacen las condiciones

1. Sa + Sb 0 S a + Sc 0 Sb + Sc 0, y

2. Sa Sb + Sb Sc + Sc Sa 0,

entonces

Sa (b c)2 + Sb (c a)2 + Sc (a b)2 0 (9.5)

 Prueba. Sin perdida de generalidad supongamos que Sb + Sc 0.


Haciendo u = b a, v = c b, tenemos

Sa (b c)2 + Sb (c a)2 + Sc (a b)2


= Sa (b c)2 + Sb (c b + b a)2 + Sc (a b)2
= Sa (b c)2 + Sb (c b)2 + Sb (b a)2 + +2Sb (c b)(b a) + Sc (a b)2
= (Sa + Sb )(c b)2 + (Sb + Sc )(b a)2 + 2Sb (c b)(b a)
= (Sa + Sb )v 2 + (Sb + Sc )u2 + 2Sb u v

 2
Sb Sa Sb + Sb Sc + Sc Sa 2
= (Sb + Sc ) u + v +
v 0.

Sb + Sc Sb + Sc
| {z }
0

2
Aplicaci
on 9.1
Pruebe que
   
a b c b c a 1 1 1
3 mn + + , + + (a + b + c) + + ,
b c a a b c a b c

siendo a, b, clos lados de un triangulo.

 Prueba. Sin perdida de generalidad supongamos que


 
a b c b c a a b c
mn + + , + + = + + ,
b c a a b c b c a
113

entonces la desigualdad que vamos a demostrar es equivalente a


   
a b c 1 1 1
3 + + (a + b + c) + +
b c a a b c

3(a2 c + b2 a + c2 b) (a + b + c)(ab + ac + bc)

3(a2 c + b2 a + c2 b) a2 b + a2 c + ab2 + b2 c + ac2 + bc2 + 3abc

2(a2 c + b2 a + c2 b) a2 b + b2 c + ac2 + 3abc

multiplicando por 2 miembro a miembro

4a2 c + 4b2 a + 4c2 b 2a2 b + 2b2 c + 2ac2 + 6abc


2 2 2 2 2 2
a
| c 2abc
{z + b }c + |b a 2abc
{z + c a} + |c b 2abc
{z + a }b
+3a2 c + 3b2 a + 3c2 b 3a2 b 3b2 c 3c2 a 0
 
c(a b)2 + a(b c)2 + b(c a)2 + 3 a2 c + b2 a + c2 b a2 b b2 c c2 a 0
c(a b)2 + a(b c)2 + b(c a)2 + 3(a b)(b c)(c a) 0
| {z }
c(a b) + a(b c) + b(c a) + (a b) + (b c) + (c a)3 0
2 2 2 3 3

(pues (a b) + (b c) + (c a) = 0)
(a b)2 (c| + {z
a }b) + (b c)2 (a b }c) + (c a)2 (b| + {z
| +{z c a}) 0
Sc Sa Sb
2 2 2
Sa (b c) + Sb (c a) + Sc (a b) 0 (9.6)

y como

Sa + Sb = 2b > 0,

Sa + Sc = 2a > 0,

Sb + Sc = 2c > 0

entonces para que se cumpla (9.6) por el teorema 9.4 basta probar que

Sa Sb + Sa Sc + Sb Sc 0.
114 9. Metodo de la Suma de Cuadrados

Reemplazando, tenemos la desigualdad equivalente

(a + b c)(b + c a) + (b + c a)(c + a b) + (c + a b)(a + b c)


= (b + a c)(b + c a) + (c + b a)(c + a b) + (a + c b)(a + b c)
= b2 (c a)2 + c2 (a b)2 + a2 (b c)2
= 2ab + 2ac + 2bc (a2 + b2 + c2 )
= 4ab (a2 + b2 + c2 + 2ab 2ac 2ac 2bc)

= 4ab (a + b c)2 = (2 ab + a + b c)(2 ab a b + c)
  
2 2 2  2
= a + b ( c) ( c) a b

 
= a + b + c a + b c c + a b c + b a > 0,
| {z } | {z } | {z }
+ + +

(pues a, b, c son lados de un triangulo)

entonces Sa Sb + Sb Sc + Sc Sa > 0; luego aplicando teorema 9.4, concluimos que

(c + a b)(a b)2 + (a + b c)(b c)2 + (b + c a)(c a)2 0.

La igualdad ocurre si y solo si el triangulo es equilatero. 2

Aplicaci on 9.2
Sean a, b, c reales positivos tales que ab + bc + ca = 1. Pruebe que

1 + a2 b2 1 + b2 c2 1 + c2 a2 5
2
+ 2
+ 2
.
(a + b) (b + c) (c + a) 2

 Prueba. Veamos

1 + a2 b2 12 + a2 b2 (ab + bc + ca)2 + a2 b2
= =
(a + b)2 (a + b)2 (a + b)2
2a2 b2 + 2ab(bc + ca) + (bc + ca)2
=
(a + b)2
2ab(ab + ac + bc) + (bc + ca)2 2ab(ab + ac + bc)
= 2
= + c2 ,
(a + b) (a + b)2
115

entonces

1 + a2 b2 2ab(ab + ac + bc)
2
= + c2 ; analogamente
(a + b) (a + b)2
1 + b2 c2 2bc(ab + ac + bc)
2
= + a2 ;
(b + c) (b + c)2
1 + c2 a2 2ca(ab + ac + bc)
2
= + b2 ;
(c + a) (c + a)2

sumando miembro a miembro, tenemos

 
1 + a2 b2 1 + b2 c2 1 + c2 a2 2ab 2bc 2ac
+ + = (ab + ac + bc) + +
(a + b)2 (b + c)2 (c + a)2 (a + b)2 (b + c)2 (a + c)2
+a2 + b2 + c2 ,

luego basta demostrar

 
2ab 2bc 2ac 5
(ab + ac + bc) 2
+ 2
+ 2
+ a2 + b2 + c2
(a + b) (b + c) (a + c) 2
 
4ab 4bc 4ac
(ab + ac + bc) 2
+ 2
+ + 2(a2 + b2 + c2 )
(a + b) (b + c) (a + c)2
5 = 5(ab + bc + ac)
 
4ab 4bc 4ac
(ab + ac + bc) + + 3
(a + b)2 (b + c)2 (a + c)2
+2(a2 + b2 + c2 ab ac bc) 0
     
4ab 4bc 4ac
(ab + ac + bc) 1 + 1 + 1
(a + b)2 (b + c)2 (a + c)2
+2(a2 + b2 + c2 ab ac bc) 0
116 9. Metodo de la Suma de Cuadrados

 
(a b)2 (b c)2 (a c)2
(ab + ac + bc) + +
(a + b)2 (b + c)2 (a + c)2
+(a b)2 + (b c)2 + (c a)2 0
   
(ab + ac + bc) 2 (ab + ac + bc)
1 (a b) + 1 (b c)2
(a + b)2 (b + c)2
 
(ab + ac + bc)
+ 1 (c a)2 0.
(a + c)2

Haciendo

ab + ac + bc ab + ac + bc ab + ac + bc
Sa = 1 ; S b = 1 ; S c = 1 .
(b + c)2 (a + c)2 (a + b)2

Supongamos sin perdida de generalidad a b c entonces Sa Sb Sc , luego


tenemos

ab + ac + bc a2 + (a + c)(c b)
Sb = 1 = 0 Sa Sb 0 y
(c + a)2 (a + c)2

   
2 2 2 ab + bc + ca 2 ab + bc + ca
b Sc + c Sb = b 1 +c 1
(a + b)2 (c + a)2
 2   2 
2 a + (a + b)(b c) 2 a + (a + c)(c b)
= b +c
(a + b)2 (c + a)2
 2 2
  2 2

b c c b
= a2 + + (c b)
(a + b)2 (c + a)2 (a + c) (a + b)
   
2 b2 c2 2 ab + bc + ca
= a + + (c b) > 0,
(a + b)2 (c + a)2 (a + c)(a + b)

luego por el teorema 9.3 queda demostrado.


3
La igualdad ocurre si y solo si a = b = c = . [10] 2
3
CAPITULO 10

DESIGUALDADES SIMETRICAS Y
CICLICAS

Definicion 10.1 : Sean a1 , a2 , . . . , an n


umeros reales, los coeficientes c0 , c1 , . . . , cn
del polinomio

n
X
H(x) = (x + a1 )(x + a2 ) (x + an ) = ck xnk
k=0

se denominan funciones polinomiales simetricas elementales de a1 , a2 , . . . , an , es


decir,

c0 = 1
n
X
c1 = a1 + a2 + + an = ai
i=1
X
c2 = a1 a2 + a1 a3 + + an1 an = ai aj
1i<jn

..
.

cn = a1 a2 an .

117
118 10. Desigualdades Simetricas y Cclicas

Ejemplo 10.1
Si H(x) = (x + a)(x + b)(x + c) = x3 + (a + b + c)x2 + (ab + ac + bc)x + abc, entonces

c0 = 1
c1 = a + b + c
c2 = ab + ac + bc
c3 = abc.

En base a estos polinomios simetricos elementales, daremos una definicion.

Definici
on 10.2: Dados los polinomios simetricos elementales c0 , c1 , . . . , cn , defini-
mos

1 k!(n k)!
Pk = 
n ck = ck ; k = 0, 1, 2, . . . , n.
k
n!

Ejemplo 10.2

P 0 = c0 = 1
1 a1 + a2 + + an
P1 = c1 =
n n
2 2
P2 = c2 = (a1 a2 + a1 a3 + + an1 an ).
n(n 1) n(n 1)

Teorema 10.1 (Desigualdad de Newton)


umeros reales positivos y k {1, 2, . . . , (n 1)} entonces, se
Sean a1 , a2 , . . . , an n
cumple que

Pk1 Pk+1 Pk2 .

La igualdad ocurre si y solo si a1 = a2 = = an .


Nota 10.1
Antes de la demostracion del teorema, veamos un resultado muy importante que
usaremos en la demostracion de la desigualdad de Newton.
Sea

n
X Xn  
nk n
H(x) = (x + a1 )(x + a2 ) (x + an ) = ck x = Pk xnk .
k=0 k=0
k
119

Derivando tenemos:
n1
X  
n

H (x) = (n k) Pk xnk1 .
k=0
k

Definimos

n1
X   n1 
X 
1 nk n nk1 n1
Q(x) = H (x) = Pk x = Pk xnk1 .
n k=0
n k k=0
k

Si los valores reales ak , k = 1, 2, . . . , n son elementos del intervalo [, ], entonces


el polinomio H(x) tiene n races reales en [, ] y por el teorema de Rolle (cons ultese
cualquier libro de analisis matematico) entonces H (x) tiene (n 1) races reales en
[, ] y denotemos como y1 , y2 , . . . , yn1 ; luego
1
Q(x) = H (x) = (x + y1 )(x + y1 ) (x + yn1 ).
n
Igualando coeficientes de Q(x), obtenemos:

Pk (a1 , a2 , . . . , an ) = Pk (y1, y2 , . . . , yn1)

para todo k = 0, 1, 2, . . . , (n 1).

 Prueba. (de la desigualdad de Newton.) Consideremos la hipotesis inductiva


Tn :

Pj1 (a1 , a2 , . . . , an ) Pj+1 (a1 , a2 , . . . , an ) Pj2 (a1 , a2 , . . . , an ),

donde j = 2, 3, . . . , (n 2).
Para j = 1 y n = 2, tenemos:
 2
a1 + a2
P0 P2 P12 1 a1 a2
2
4a1 a2 a21 + a22 + 2a1 a2
(a1 a2 )2 0

como vemos se cumple para n = 2.


120 10. Desigualdades Simetricas y Cclicas

Para j = 1 y n = 3.

P0 (a1 , a2 , a3 ) P2 (a1 , a2 , a3 ) P12 (a1 , a2 , a3 )


 2
2 a1 + a2 + a3
(1) (a1 a2 + a1 a3 + a2 a3 )
3(2) 3
3(a1 a2 + a1 a3 + a2 a3 ) a21 + a22 + a23 + 2(a1 a2 + a1 a3 + a2 a3 )
a21 + a22 + a23 a1 a2 + a1 a3 + a2 a3

esta es una desigualdad verdadera.


Para j = 2 y n = 3.

P1 (a1 , a2 , a3 ) P3 (a1 , a2 , a3 ) P22 (a1 , a2 , a3 )


   2
a1 + a2 + a3 a1 a2 + a1 a3 + a2 a3
(a1 a2 a3 )
3 3

Si a1 a2 a3 = 0, se cumple trivialmente.
Si a1 a2 a3 6= 0, entonces dividiendo por (a1 a2 a3 )2 , tenemos
   2
1 1 1 1 1 1 1 1
+ + + +
3 a1 a2 a2 a3 a3 a1 9 a1 a2 a3
   2
1 1 1 1 1 1
3 + + + +
a1 a2 a2 a3 a3 a1 a1 a2 a3
 2  2  2
1 1 1 1 1 1
+ + + + .
a1 a2 a3 a1 a2 a1 a3 a2 a3

Ahora veamos para j = n 1, es decir probaremos que


2
Pn2 (a1 , a2 , . . . , an ) Pn (a1 , a2 , . . . , an ) Pn1 (a1 , a2 , . . . , an )
" #
2 X
(a1 abj abk an ) a1 a2 an
n(n 1) 1j<kn

n
!2
1X
a1 a2 abj an
n j=1

donde los smbolos abj y abk se omiten.


Si a1 a2 an = 0, se verifica la desigualdad.
121

Si a1 a2 an 6= 0, entonces dividiendo por (a1 a2 an )2 , se tiene


n
!2
1 X 1 1X 1
n

2
aj ak n j=1 aj
1j<kn

y esta desigualdad es equivalente a:


     
1 1 1 1 1 1 1 1 1
P0 , ,..., P2 , ,..., P12 , ,..., . (10.1)
a1 a2 an a1 a2 an a1 a2 an

Teniendo en cuenta la nota anterior, la desigualdad (10.1) es verdadera y con


esto se completa la prueba. 2
Teorema 10.2 (Desigualdad de Mac Laurin)
Si a1 , a2 , . . . , an son n
umeros reales positivos, entonces

P1 (P2 )1/2 (Pk )1/k (Pn )1/n .

La igualdad ocurre si y solo si a1 = a2 = = an .


 Prueba. Como a1 , a2 , . . . , an > 0, entonces podemos aplicar la desigualdad de
Newton para 1 k < n, en efecto:

k1
Y 2(k1)
Pkk1 k
Pk+1 Pj2j = P0 P12 P24 P36 Pk1 Pkk1 Pk+1
k

j=0

= (P0 P2 )(P1 P3 )2 (P2 P4 )3 (Pk1 Pk+1 )2k


P12 P24 P36 Pk2k
k
Y
= Pj2j
j=1
k1
Y k1
Y
Pkk1 k
Pk+1 Pj2j Pj2j Pk2k .
j=0 j=1

Pkk1 k
Pk+1 Pk2k
k
Pk+1 Pkk+1
1/k+1 1/k
Pk+1 Pk .
La igualdad ocurre si y solo si se cumple a1 = a2 = = an . 2
122 10. Desigualdades Simetricas y Cclicas

Aplicaci on 10.1
Sean a, b, c, d n
umeros reales positivos, pruebe que

27
(ab + ac + ad + bc + bd + cd)3 (abc + abd + acd + bcd)2 .
2

 Prueba. Utilizando la desigualdad de Mac Laurin:

1/2 1/3
P2 P3 P23 P32

!3 !2
1 1
4 (ab + ac + ad + bc + bd + cd)
 4 (abc + abd + acd + bcd)

2 3
1 1
3
(ab + ac + ad + bc + bd + cd)3 2 (abc + abd + acd + bcd)2
6 4
27
(ab + ac + ad + bc + bd + cd)3 (abc + abd + acd + bcd)2 .
2

Aplicacion 10.2
Dados a, b, c, d n
umeros reales positivos, pruebe que
   2
1 1 1 1 1 1 3 1 1 1 1
+ + + + + + + + .
ab ac ad bc bd cd 8 a b c d

 Prueba. Multiplicando por (abcd)2 tenemos la desigualdad equivalente:

   2
1 2 1 1 1 1 1 3 2 1 1 1 1
(abcd) + + + + + (abcd) + + +
ab ac ad bc bd cd 8 a b c d
   2
cd + bd + bc + ad + ac + ab bcd + acd + abd + abc
abcd
6 4
P4 P2 (P3 )2 ,

esta desigualdad es verdadera, pues es la desigualdad de Newton. 2


10.1. Polinomios simetricos y cclicos 123

10.1. Polinomios sim


etricos y cclicos
Para mayor comprension consideremos un polinomio P (x, y, z) de variables x, y, z;
luego, para definir introducimos dos notaciones muy importantes como:
X X
y .
cc sim

Definici
on 10.3 :
X
P (x, y, z) = P (x, y, z) + P (y, z, x) + P (z, x, y)
cc
X
P (x, y, z) = P (x, y, z) + P (x, z, y) + P (y, z, x)
sim
+P (y, x, z) + P (z, x, y) + P (z, y, x).

Ejemplo 10.3
X
x2 y = x2 y + y 2z + z 2 x
cc
X
x2 y = x2 y + x2 z + y 2 x + y 2 z + z 2 x + z 2 y
sim
X
x3 = x3 + y 3 + z 3
cc
X
x3 = 2 (x3 + y 3 + z 3 )
sim
X
xyz = xyz + yzx + zxy = 3xyz
cc
X
xyz = xyz + xzy + yxz + yzx + zxy + zyx = 6xyz.
sim

A continuacion enunciaremos el teorema de Muirhead para tres variables para


mayor entendimiento.
124 10. Desigualdades Simetricas y Cclicas

Teorema 10.3 (Teorema de Muirhead)


Sean a1 , a2 , a3 , b1 , b2 , b3 n
umeros reales no negativos tales que

a1 a2 a3 , b1 b2 b3 , a1 b1 , a1 + a2 b1 + b2 , a1 + a2 + a3 = b1 + b2 + b3 .

Sean x, y, z n
umeros reales no negativos, entonces
X X
xa1 y a2 z a3 xb1 y b2 z b3 .
sim sim

 Prueba. De las hipotesis se observa que hay dos casos para analizar:
Caso 1:

b1 a2 a1 + b1 a1 + a2
a1 a1 + a2 b1

y de a1 b1 se tiene que a1 max {a1 + a2 b1 , b1 }.


Ademas max {a1 , a2 } = a1 max {a1 + a2 b1 , b1 }; tambien de a1 + a2 b1
b1 + a3 b1 = a3 y a1 + a2 b1 b2 b3 , tenemos max {a1 + a2 b1 , a3 }
max {b2 , b3 }.
En efecto
X X
xa1 y a2 z a3 = (xa1 y a2 z a3 + xa2 y a1 z a3 )
sim cc
X
= z a3 (xa1 y a2 + xa2 y a1 )
cc
X 
z a3 xa1 +a2 b1 y b1 + xb1 y a1 +a2 b1 ,
cc
pues

xa1 y a2 + xa2 y a1 xa1 +a2 b1 y b1 + xb1 y a1 +a2 b1


xa1 y a2 + xa2 y a1 xa1 +a2 b1 y b1 xb1 y a1 +a2 b1 0

xa2 y a2 xa1 a2 + y a1 a2 xa1 b1 y b1a2 xb1 a2 y a1 b1 0
 
xa2 y a2 xb1 a2 y b1 a2 xa1 b1 y a1 b1 0
10.1. Polinomios simetricos y cclicos 125

por que para cualquier orden x y o y x, el producto siempre es no negativo.


Luego

X X 
xa1 y a2 z a3 z a3 xa1 +a2 b1 y b1 + xb1 y a1 +a2 b1
sim cc
X 
= xb1 y a1 +a2 b1 z a3 + xb1 y a3 z a1 +a2 b1
cc
X 
= xb1 y a1 +a2 b1 z a3 + y a3 z a1 +a2 b1
cc
X 
xb1 y b2 z b3 + y b3 z b2
cc
X 
= xb1 y b2 z b3 + xb1 y b3 z b2
cc
X
= xb1 y b2 z b3
sim

Caso 2: Si b1 a2

3b1 b1 + b2 + b3 = a1 + a2 + a3 b1 + a2 + a3
3b1 b1 + a2 + a3
2b1 a2 + a3
b1 a2 + a3 b1

ademas a1 a2 b1 a1 a2 + a3 b1 .
Luego tenemos que max {a2 , a3 } max {b1 , a2 + a3 b1 } y max {a1 , a2 + a3 b1 }
max {b2 , b3 }.
En efecto:

X X
xa1 y a2 z a3 = (xa1 y a2 z a3 + xa1 z a2 y a3 )
sim cc
X
= xa1 (y a2 z a3 + z a2 y a3 )
cc
126 10. Desigualdades Simetricas y Cclicas

X 
xa1 y b1 z a2 +a3 b1 + y a2 +a3 b1 z b1
cc
X 
= xa1 y b1 z a2 +a3 b1 + xa1 y a2 +a3 b1 z b1
cc
X 
= xa1 y b1 z a2 +a3 b1 + xa2 +a3 b1 y b1 z a1
cc
X 
= y b1 xa1 z a2 +a3 b1 + xa2 +a3 b1 z a1
cc
X 
y b1 xb2 z b3 + xb3 z b2
cc
X
= xb1 y b2 z b3
sim
2
Aplicaci
on 10.3
Pruebe para todo a, b, c n
umeros reales positivos la desigualdad
a b c 3
+ + .
b+c a+c a+b 2
 Prueba. La desigualdad a demostrar es equivalente a

2 [a(a + c)(a + b) + b(b + c)(a + b) + c(b + c)(a + c)] 3(b + c)(a + c)(a + b),

efectuando

2 [a(a2 + (b + c)a + bc) + b(b2 + (a + c)b + ac) + c(c2 + (a + b)c + ab)]


3 [(b + c)(a2 + (b + c)a + bc)]
2(a3 + b3 + c3 ) + 2 (a2 (b + c) + b2 (a + c) + c2 (a + b)) + 6abc
3 [a2 (b + c) + b2 (a + c) + c2 (a + b) + 2abc]
2(a3 + b3 + c3 ) a2 (b + c) + b2 (a + c) + c2 (a + b)
X X
a3 a2 b.
sim sim
2
10.1. Polinomios simetricos y cclicos 127

Aplicaci
on 10.4
(IMO 1995). Sean a, b, c n
umeros reales positivos tales que abc = 1, pruebe que
1 1 1 3
+ 3 + 3 .
a3 (b + c) b (c + a) c (a + b) 2

 Prueba. Como los denominadores del primer miembro son de grado 4 y abc = 1,
entonces la desigualdad es equivalente a
1 1 1 3
+ 3 + 3 ,
a3 (b + c) b (c + a) c (a + b) 2(abc)4/3

es conveniente eliminar el exponente 4/3, para ello hacemos

a = x3 , b = y 3 , c = z 3 , con x, y, z > 0,

luego tenemos:

1 1 1 3 3x5 y 5 z 5
+ + =
x9 (y 3 + z 3 ) y 9(z 3 + x3 ) z 9 (x3 + y 3) 2(xyz)4 2(x9 y 9 z 9 )
2 [y 9z 9 (z 3 + x3 )(x3 + y 3 ) + x9 z 9 (y 3 + z 3 )(x3 + y 3) + x9 z 9 (y 3 + z 3 )(z 3 + x3 )]
3 [x5 y 5z 5 (y 3 + z 3 )(z 3 + x3 )(x3 + y 3 )] ,

efectuando

X X X X
x12 y 12 + 2 x12 y 9 z 3 + x9 y 9 z 6 3 x11 y 8z 5 + 6x8 y 8 z 8
sim sim sim sim

X X X X
x12 y 12 x11 y 8 z 5 + 2 x12 y 9z 3 x11 y 8z 5
sim sim sim sim

X X
+ x9 y 9 z 6 x8 y 8 z 8 0.
sim sim

Para enunciar en forma general el teorema de Muirhead, es necesaria la siguiente


definicion.
128 10. Desigualdades Simetricas y Cclicas

on 10.4 : Sean = (1 , 2 , . . . , n ), = (1 , 2 , . . . , n ) elementos en Rn ,


Definici
de componentes no negativas; diremos que > si y solo si

(i) 1 2 n y 1 2 n .

(ii) 1 + 2 + + k 1 + 2 + + k ; k = 1, 2, . . . , (n 1).

(iii) 1 + 2 + + n = 1 + 2 + + n .

Teorema 10.4 (Generalizado de Muirhead)


Si > y x1 , x2 , . . . , xn numeros reales no negativos, entonces
X X
x1 1 x2 2 xnn x1 1 x2 2 xnn .
sim sim
La igualdad ocurre si y solo si = o x1 = x2 = = xn .
Aplicaci on 10.5
Sean x1 , x2 , x3 , x4 n
umeros reales positivos, pruebe que

3 3
(x1 + x32 + x33 + x34 ) x1 x4 x2 x3 + x2 x4 x1 x3 + x3 x4 x1 x2
2

+x2 x3 x1 x4 + x1 x2 x3 x4 + x1 x3 x2 x4 .
X X
 Prueba. Como x31 = x31 (x02 x03 x04 ), entonces tenemos 6 permutaciones
sim sim
para (x2 , x3 , x4 ) y para x1 x2 x3 x4 , tenemos 2 permutaciones para x1 x2 y 2 per-

mutaciones para x3 x4 , en total 4; luego, la desigualdad se puede escribir equiva-
lentemente como


6(x31 + x32 + x33 + x34 ) 4[x1 x4 x2 x3 + x2 x4 x1 x3 + x3 x4 x1 x2

+x2 x3 x1 x4 + x1 x2 x3 x4 + x1 x3 x2 x4 ].
X X
x31 x1 x2 x3 x4 ,
sim sim

como = (3, 0, 0, 0) y = (1, 1, 1/2, 1/2) entonces esta u


ltima desigualdad es
verdadera.
La igualdad ocurre si y solo si x1 = x2 = x3 = x4 . 2
CAPITULO 11
PROBLEMAS

1. Sean x, y n
umeros reales positivos tales que a y b son las medias aritmetica y
geometrica respectivamente, pruebe que

(1 + b)2 (1 + x)(1 + y) (1 + a)2 .

2. Pruebe que

    2
1 1 2
1+ 1+ 1+
x y x+y

para todo x, y n
umeros reales positivos.

3. Sean x, y, z n
umeros reales positivos, pruebe que al menos una de las siguientes
desigualdades es verdadera:

xy 0
yz 0
zx 0.

4. Si a2 + b2 = 1, halle la variacion de 2a + 3b.

5. Sabiendo que 2a + 3b + 4c = 29, halle el menor valor de a2 + b2 + c2 .

129
130 11. Problemas

6. Pruebe que
p p
m2 + y 2 + n2 + x2 (m + n)2 + (x + y)2

para todo x, y, m, n n
umeros reales.
7. Si a, b, c son n
umeros reales positivos que satisfacen abc = 1, pruebe que

1 + ab 1 + bc 1 + ac
+ + 3.
1+a 1+b 1+c

8. Sean a, b, c n
umeros reales positivos, pruebe que

a b c 3
+ + .
b+c c+a a+b 2

(Desigualdad de Nesbit).
9. Pruebe que si x, y, z son n
umeros reales positivos y x + y + z = 1, entonces
   
1 1 1
1+ 1+ 1+ 64.
x y z

umeros reales positivos tales que a2 + b2 = (c2 + d2 )3 , pruebe


10. Sean a, b, c, d n
que

c3 d 3
+ 1.
a b

11. Sean a, b, c n
umeros reales positivos, pruebe que

a b c
+ + 1.
b + 2c c + 2a a + 2b

12. Pruebe que

x2 y2
+ 8,
y1 x1

para todo x, y > 1.


131

13. Sean a, b, c n
umeros reales positivos tales que a + b + c = 2, pruebe que

a+b b+c c+a 9


+ + .
2a + bc 2b + ac 2c + ab 4

umeros reales tales que a2 + b2 = 1, pruebe que


14. Si a, b son n

2 2
a b + ab2 .
2

15. Sean x, y n
umeros reales positivos, pruebe que

x y
+ 1.
x + 3y y + 3x

umeros reales positivos tales que ab + bc + ca 3abc, pruebe que


16. Sean a, b, c n

a + b + c a3 + b3 + c3 .

umeros reales positivos tales que a2 + b2 + c2 = 3, pruebe que


17. Sean a, b, c n

1 1 1
+ + 1.
1 + 2ab 1 + 2bc 1 + 2ca

umeros reales positivos tales que a2 + b2 + c2 = 3, pruebe que


18. Sean a, b, c n

1 1 1 3
+ + .
1 + ab 1 + bc 1 + ca 2

19. Pruebe que



(a + b + c)4 8 a2 + b2 + c2 (ab + ac + bc).

para todo a, b, c n
umeros reales.
20. Dados a, b, c n
umeros reales positivos con abc = 8, pruebe que

1 1 1
+ + 1.
1+a3 1+b3 1 + c3
132 11. Problemas

21. Sean a, b n
umeros reales positivos, pruebe que

1 1 1
2
+ 2
.
(1 + a) (1 + b) 1 + ab

22. Sean a, b, c, d n
umeros reales positivos tales que abcd = 1, pruebe que

1 1 1 1
2
+ 2
+ 2
+ 1.
(1 + a) (1 + b) (1 + c) (1 + d)2

23. Sean a, b, c n
umeros reales positivos, pruebe que

2 (a3 + b3 + c3 ) + (3a + 1) (b2 + c2 ) + (3b + 1) (a2 + c2 ) + (3c + 1) (a2 + b2 )


2(ab + ac + bc) + 3(a + b)(a + c)(b + c)

24. (IMO 1995). Sean a, b, c n


umeros reales positivos tales que abc = 1, pruebe
que

1 1 1 3
+ 3 + 3 .
a3 (b + c) b (c + a) c (a + b) 2

25. Demuestre que


     
2 1 1 2 1 1 2 1 1
a + +b + +c + 2(a + b + c)
b c a c a b

para todo a, b, c n
umeros reales positivos.

26. Sean x, y, z n
umeros reales que pertenecen al intervalo [1, 1] tales que x +
y + z = 0, pruebe que
p p
1 + x + y 2 + 1 + y + z 2 + 1 + z + x2 3.

27. Sean a, b, c n
umeros reales, pruebe que

a+b+c+ a2 + b2 + c2 ab bc ca 3 max {a, b, c}.
133

28. Dados x, y, z n
umeros reales positivos tales que x + y + z = 1, pruebe que
r
x y z 3
+ + .
y+z z+x x+y 2

29. (IMO 1964). Sean a, b, c los lados de un triangulo, pruebe que

a2 (b + c a) + b2 (a + c b) + c2 (a + b c) 3abc.

umeros reales tales que a2 + b2 = 1, pruebe que


30. Dados a, b n

3 3
ab + max {a; b} .
4
31. Dados a, b, c, d n
umeros reales no negativos, pruebe que

a b c d
+ 2 + 2 + 2 2.
b2 2
+c +d 2 2
c +d +a2 2
d +a +b2 a + b2 + c2

umeros reales no negativos tales que a2 +b2 +c2 +d2 = 4, pruebe


32. Sean a, b, c, d n
que

a3 + b3 + c3 + d3 8.

33. Sean a, b, c los lados de un triangulo, pruebe que

a b c
+ + 1.
3a b + c 3b c + a 3c a + b

34. Pruebe que si a, b, c son n


umeros reales positivos tales que abc = 1,entonces

1 1 1 1 1 1
+ + + + .
1+a+b 1+b+c 1+c+a 2+a 2+b 2+c
umeros reales positivos que satisfacen abc 1,
35. (IMO 2005). Dados a, b, c n
pruebe que

a5 a2 b5 b2 c5 c2
+ + 0.
a5 + b2 + c2 b5 + c2 + a2 c5 + a2 + b2
134 11. Problemas

36. Sean a, b, c n
umeros reales tales que a + b + c = 3, pruebe que


a+ b+ c ab + bc + ca.

37. (APMO 1998). Sean x, y, z n


umeros reales positivos, pruebe que

 
x  y z 2(x + y + z)
1+ 1+ 1+ 2+ .
y z x 3 xyz

38. (USAMO 1998). Sean a, b, c n


umeros reales positivos, pruebe que

1 1 1 1
+ 3 + 3 .
a3 b3 3 3
+ + 3abc b + c + 3abc c + a + 3abc abc

39. Sean a, b, c reales positivos con suma 3, pruebe que

a b c 3
+ + .
1 + b2 1 + c2 1 + a2 2

40. Sean a, b, c, d n
umeros reales positivos, pruebe que

a3 b3 c3 d3 a+b+c+d
+ + + .
a2 + b2 b2 + c2 c2 + d2 d2 + a2 2

41. Sean a, b, c, d n
umeros reales positivos tales que abcd = 1, pruebe que

1 1 1
S = + +
1 + ab + bc + ca 1 + bc + cd + ab 1 + cd + da + ac
1
+ 1.
1 + da + ab + bd

42. Sean a, b, c n
umeros no negativos tales que a + b + c = 3, pruebe que

1 1 1
+ + 1.
2ab2 +1 2bc2 2
+ 1 2ca + 1
135

43. Sean a, b, c, x, y, z n
umeros reales no negativos tales que a + b + c = x + y + z,
pruebe que

ax(a + x) + by(b + y) + cz(c + z) 3(abc + xyz).

44. Sean a, b, c n
umeros reales positivos, pruebe que

a2 (b + c a) b2 (c + a b) c2 (a + b c) ab + bc + ca
+ + .
b+c c+a a+b 2

45. Si a, b, c reales positivos, pruebe que


s s s
a3 b3 c3
+ + 1.
a3 + (b + c)3 b3 + (c + a)3 c3 + (a + b)3

46. Si a, b, c, d son n
umeros reales no negativos, pruebe que

ab bc cd da
+ + + 0.
a + 2b + c b + 2c + d c + 2d + a d + 2a + b

47. Si a, b, c son n
umeros reales no negativos, entonces

  
a2 bc b + c + b2 ca c + a + c2 ab a + b 0.

48. Dados a, b, c n
umeros reales positivos, pruebe que

 2  2  2
a b c 3
+ + .
a+b b+c c+a 4

49. Sean a, b, c, d n
umeros reales positivos que satisfacen

1 1 1 1
+ + + = 1,
1+a 1+b 1+c 1+d

pruebe que abcd 81.


136 11. Problemas

50. Sean a, b, c los lados de un triangulo, pruebe que

a b c
+ + 1.
3a b + c 3b c + a 3c a + b

51. Dados a, b, c, d n
umeros reales positivos, pruebe que
21
a a 16

3
21 21 21 21 .
a3 + 63bcd a 16 + b 16 + c 16 + d 16

52. Dados a, b, c, d n
umeros reales positivos, pruebe que

a b c d
A=
3
+
3
+
3
+
3
1.
a3 + 63bcd b3 + 63acd c3 + 63bda d3 + 63abc

53. Sean a, b, c los lados de un triangulo, pruebe que


r
3 a3 + b3 + c3 + 3abc
max {a, b, c}.
2

54. Sean a, b, c n
umeros reales positivos, pruebe que

(a + b)2 (b + c)2 (c + a)2


S= + + > 2.
a2 + b2 + c2 + ab a2 + b2 + c2 + bc a2 + b2 + c2 + ca

umeros reales tales que a, b, c [0; 1], pruebe que


55. Dados a, b, c n

a b c
+ + + (1 a)(1 b)(1 c) 1.
b+c+1 c+a+1 a+b+1

56. Sean a, b, c n
umeros reales positivos, pruebe que

a b c 3
+ + .
2a + b + c 2b + c + a 2c + a + b 4

umeros reales tales que a2 + b2 + c2 = 1, pruebe que


57. Sean a, b, c n

1 ab + 1 bc + 1 ca 6.
137

58. Pruebe que

a4 + b4 + c4 + abc(a + b + c) ab(a2 + b2 ) + bc(b2 + c2 ) + ca(c2 + a2 ),

se cumple para todo a, b, c n


umeros reales.

59. Pruebe que

a3 + b3 + c3 + 4(a + b + c) + 9abc 8(ab + bc + ca),

se cumple para todo a, b, c n


umeros reales no negativos.

60. Pruebe que



p p p 3 2
M= a2 + (1 b)2 + b2 + (1 c)2 + c2 + (1 a)2 ,
2

se cumple para todo a, b, c n


umeros reales.

61. Pruebe que


s s s s
3 3 3
a b c d3
3
+ 3
+ 3
+ 1,
a + 15bcd b + 15acd c + 15abd d3 + 15abc

se cumple para todo a, b, c n


umeros reales positivos.

62. Sean a, b, c n
umeros reales no negativos tales que ab + bc + ca = 3, pruebe que

1 1 1
+ 2 + 2 1.
a2 +2 b +2 c +2

63. Sean a, b, c n
umeros reales positivos, pruebe que

a3 b3 c3 3 2  1
+ + a + b2 + c2 (ab + bc + ca).
a+b b+c c+a 4 4
138 11. Problemas

64. Sean a, b, c n
umeros reales positivos, pruebe que

ab bc ca 1
S= + + (a + b + c).
2c + a + b 2a + b + c 2b + c + a 4

65. Si a b c d e, entonces

(a + b + c + d + e)2 8(ac + bd + ce).

Cuando ocurre la igualdad?

umeros reales positivos tales que xyz xy + yz + zx.


66. Sean x, y, z n
Pruebe que:

xyz 3(x + y + z).

67. Sean x, y n
umeros reales positivos tales que x + y = 1, pruebe que

x y 2
+p .
1x2 1 y2 3

68. Sean a1 , a2 , . . . , an , b1 , b2 , . . . , bn n
umeros reales positivos, tales que a1 + a2 +
+ an = b1 + b2 + + bn . Pruebe que

a21 a22 a2n 1


+ + + (a1 + a2 + + an ).
a1 + b1 a2 + b2 an + bn 2

69. Sean a, b, c n
umeros reales positivos, pruebe que

2ab 2bc 2ca


+ + a + b + c.
a+b b+c c+a

70. (Rusia 1992). Sean x, y, z n


umeros reales positivos, pruebe que


x4 + y 4 + z 2 8xyz.
139

71. Sean a, b, c n
umeros reales positivos con a + b + c = 1, pruebe que
   
1 1 1
+1 +1 +1 64.
a b c

72. Sean a, b, c n
umeros reales, y

x= b2 bc + c2 ; y = c2 ca + a2 ; z = a2 ab + b2

pruebe que

xy + yz + zx a2 + b2 + c2 .

73. Sean x, y, z n
umeros reales positivos tales que x + y + z = 3, pruebe que

(1 x)2 (1 y)2 (1 z)2


+ + 0.
1 x4 1 y4 1 z4

74. Si a1 , a2 , . . . , an son n
umeros reales, pruebe que

n X
X n
i j cos (ai aj ) 0.
i=1 j=1

75. Sean a, b, c n
umeros reales, tales que

x= b2 + bc + c2 ; y = c2 + ca + a2 ; z = a2 + ab + b2

pruebe que

xy + yz + zx (a + b + c)2 .

76. Sean a, b, c, d, e, f n
umeros reales positivos, pruebe que

a b c d e f
+ + + + + 3.
b+c c+d d+e e+f f +a a+b
140 11. Problemas

77. Sean a, b, c, x, y, z, t, v, u n
umeros reales positivos, entonces

  
a3 + b3 + c3 x3 + y 3 + z 3 t3 + u3 + v 3 (axt + byu + czv)3 .

78. (IMO 2001). Pruebe para a, b, c n


umeros reales positivos

a b c
+ + 1.
a2 + 8bc 2
b + 8ca 2
c + 8ab

1 1 1
79. (APMO 2002). Sean a, b y c n
umeros reales positivos que satisfacen + + =
a b c
1, pruebe que


a + bc + b + ca + c + ab abc + a+ b+ c.

80. Sean a, b, c n
umeros reales positivos que satisfacen abc = 1, pruebe que

3 6
1+ .
a+b+c ab + ac + bc

81. Sean a, b, c, x, y, z n
umeros reales positivos tales que x + a = y + b = z + c = 1,
pruebe que
 
1 1 1
(abc + xyz) + + 3.
ay bz cx

82. Sean x, y, z n
umeros reales positivos, pruebe que

x+y+z xy + yz + zx
p p .
3 3 x2 + xy + y 2 + y 2 + yz + z 2 + z 2 + zx + x2

umeros reales tales que x2 + y 2 1 y z 2 + t2 1, pruebe que


83. Sean x, y, z, t n
p p
(x + z)2 + (y + t)2 + (x z)2 + (y t)2 2 2.

umeros reales tales que a2 +b2 +c2 8, pruebe que ab+bc+2ca


84. Dados a, b, c n
8.
141

85. Dados a, b, c n umeros reales no negativos tales que a + b + c = 1, pruebe que


256
a4 b + b4 c + c4 a .
3125
86. Si a, b, y c son los lados de un triangulo, pruebe que

a b c
+ + 3.
b+ca c+ab a+bc

umero entero 2, pruebe que


87. Sea n un n
!4
X  X
8 xi xj x2i + x2j xi .
1i<jn 1in

umeros reales positivos y S = a1 + a2 + + an , pruebe


88. Sean a1 , a2 , . . . , an n
que

S S S n2
+ ++ .
S a1 S a2 S an n1

89. Sean a, b, c n
umeros reales no negativos tales que a + b + c = 3, pruebe que

a 1 + b3 + b 1 + c3 + c 1 + a3 5.

90. Sean a, b, c, d n
umeros reales no negativos, pruebe que
    
2a 2b 2c 2d
1+ 1+ 1+ 1+ 9.
b+c c+d d+a a+b

91. Si a, b, c son n
umeros reales positivos, entonces
r r r
2a 2b 2c
+ + 3.
a+b b+c c+a

92. Si x1 , x2 , . . . , xn n
umeros reales positivos tales que

1 1 1
+ ++ = 1,
1 + x1 1 + x2 1 + xn
142 11. Problemas

pruebe que

x1 x2 xn (n 1)n .

umeros reales positivos tales que a2 + b2 + c2 = 3, pruebe que


93. Sean a, b, c n

ab bc ca
+ + 3.
c a b

94. Sean a, b, c n
umeros reales positivos, pruebe que

a+b+c
aa bb cc (abc) 3 .

95. Sean a, b, c n
umeros reales positivos, pruebe que

a b c c+a a+b b+c


+ + + + .
b c a c+b a+c b+a

96. Si a, b, c, d son n
umeros reales positivos, entonces

ab bc cd da
+ + + 0.
b+c c+d d+a a+b

97. Sean a, b, c n
umeros reales positivos, pruebe que

a3 b3 c3 3(ab + ac + bc)
2 2
+ 2 2
+ 2 2
.
b bc + b c ac + a a ab + b a+b+c

98. Si a, b, c, d son n
umeros reales positivos, entonces
 2  2  2  2
a b c d
+ + + 1.
a+b b+c c+d d+a

99. Si a, b, c son n
umeros reales no negativos, entonces

  
3 1 a + a2 1 b + b2 1 c + c2 1 + abc + a2 b2 c2 .
143

100. Sean a, b, c son n


umeros reales no negativos, pruebe que

a2 + b2 + c2 + 2abc + 1 2(ab + ac + bc).

umeros reales positivos tales que abc 1, pruebe que


101. Sean a, b, c n

a3 + b3 + c3 ab + ac + bc.

102. Sean a, b, c n
umeros reales positivos tales que a + b + c = 1, pruebe que


ab + c + bc + a + ca + b 1 + ab + bc + ca.

103. Sean a, b numeros reales positivos tales que a + b 1, halle el mnimo valor
de f (a, b) = ab + (ab)1 .

3
104. Sean a, b, c numeros reales positivos tales que a + b + c , halle el mnimo
r r r 2
1 1 1
de f (a, b, c) = a2 + 2 + b2 + 2 + c2 + 2 .
b c a

umeros reales tales que a 2, b 6 y c 12, halle el maximo


105. Sean a, b, c n
valor de


bc a 2 + ca 3 b 6 + ab 4 c 12
f (a, b, c) = .
abc

a3 b3 c3
106. Sean a, b, c n
umeros reales positivos, pruebe que + + ab + bc + ca.
b c a
107. Sean a, b, c, d n
umeros reales positivos, halle el mnimo valor de la funcion

a b c d
f (a, b, c, d) = + + +
b+c+d c+d+a d+a+b a+b+c
b+c+d c+d+a d+a+b a+b+c
+ + + + .
a b c d
144 11. Problemas

108. Halle el maximo valor de

a3 (b + c + d) + b3 (c + d + a) + c3 (d + a + b) + d3 (a + b + c)

donde a, b, c, d son n
umeros reales tales que la suma de cuadrados es 1.

109. Sean a, b, c n
umeros reales positivos, pruebe que

 
a+b+c
3 2  2 2
abc max a b , b c , c a .
3

110. Si a, b, c son n
umeros reales no negativos, pruebe que

 3
3 3 b+c
3
a +b +c +2 a 3abc.
2

111. Sean x1 , x2 , x3 , x4 , x5 n
umeros reales tales que x1 + x2 + x3 + x4 + x5 = 0,
pruebe que

|cos x1 | + |cos x2 | + |cos x3 | + |cos x4 | + |cos x5 | 1.

112. Sean a, b, c n
umeros reales diferentes, pruebe que

a2 b2 c2
+ + 2.
(b c)2 (c a)2 (a b)2

113. (IMO 1960). Resuelva

4x2
2 < 2x + 9.
1 1 + 2x

114. (IMO 1964). Si a, b, c son los lados de un triangulo, pruebe que

a2 (b + c a) + b2 (a + c b) + c2 (a + b c) 3abc.
145

115. (IMO 1975). Considere las sucesiones (x1 , x2 , . . . , xn , . . .) y (y1 , y2, . . . , yn , . . .)


tales que x1 x2 xn , y1 y2 yn , y (z1 , z2 , . . . , zn ) es una
permutacion de (y1 , y2 , . . . , yn ), pruebe que

(x1 y1 )2 + + (xn yn )2 (x1 z1 )2 + + (xn zn )2 .

116. (IMO 1978). Sean x1 , x2 , . . . , xn n


umeros enteros diferentes, pruebe que

x1 x2 xn 1 1 1
2
+ 2 ++ 2 + ++ .
1 2 n 1 2 n

117. (IMO 1997). Sean x1 , x2 , . . . , xn n


umeros reales que satisfacen

n+1
|x1 + x2 + + xn | = 1 y |xi | , para todo i = 1, 2, . . . , n,
2

pruebe que existe la permutacion (y1 , y2 , . . . , yn ) de (x1 , x2 , . . . , xn ) tal que

n+1
|y1 + 2y2 + + nyn | .
2

118. (IMO 2004). Sea n un n umero entero positivo 3 y sean t1 , t2 , . . . , tn n


umeros
reales positivos tales que
 
2 1 1 1
n + 1 > (t1 + t2 + + tn ) + ++ ,
t1 t2 tn

pruebe que ti , tj , tk son los lados de un triangulo para todo i, j, k con 1 i <
j < k n.

119. (IMO 2000). Sean a, b, c n


umeros reales positivos tales que abc = 1, pruebe
que
   
1 1 1
a1+ b1+ c1+ 1.
b c a
146 11. Problemas

120. Pruebe que

n
X nX
|i j| |xi xj | = |xi xj |
i,j=1
2 i,j

si x1 x2 xn .

121. Pruebe que

n
X n2 (n2 1)
(i j)2 = .
i,j=1
6

122. (IMO 2003). Sea n un n umero entero 2 y los n


umeros reales x1 , x2 , . . . , xn
con x1 x2 xn , pruebe que

n
!2 n
X 2 2 X
|xi xj | n 1 (xi xj )2 .
i,j=1
3 i,j=1

123. (IMO 2006). Determine el menor valor de M para que la desigualdad


ab(a2 b2 ) + bc(b2 c2 ) + ca(c2 a2 ) M a2 + b2 + c2 2

se verifique para cualesquiera a, b, c n


umeros reales.

124. (IMO 1983). Sean a, b, c los lados de un triangulo, pruebe que

a2 b(a b) + b2 c(b c) + c2 a(c a) 0.

125. Sean a, b, c, d, e n
umeros reales positivos tales que abcde = 1, pruebe que

a + abc b + bcd c + cde d + dea


+ + +
1 + ab + abcd 1 + bc + bcde 1 + cd + cdea 1 + de + deab
e + eab 10
+ .
1 + ea + eabc 3
147

126. Sean a, b, c que satisfacen a2 + b2 + c2 = 1, halle el maximo valor de

P = (a b)(a c)(b c)(a + b + c).

127. (IMO 2008).

(a) Demuestre que

x2 x2 x2
+ + 1 (11.1)
(x 1)2 (x 1)2 (x 1)2

para todos los n


umeros reales x, y, z, distintos de 1, con xyz = 1.
(b) Demuestre que existen infinitas ternas de numeros racionales x, y, z, dis-
tintos de 1, con xyz = 1 para los cuales la expresion (11.1) es una igual-
dad.
148 11. Problemas
CAPITULO 12
SOLUCIONES

1. Sean x, y n
umeros reales positivos tales que a y b son las medias aritmetica y
geometrica respectivamente, pruebe que

(1 + b)2 (1 + x)(1 + y) (1 + a)2 .

x+y
 Prueba. Sean a = , b = xy, y como se cumple:
2


2 xy x + y

1 + 2 xy + xy 1 + x + y + xy

(1 + xy)2 (1 + x)(1 + y)
(1 + b)2 (1 + x)(1 + y). (12.1)

Tambien se cumple

4xy (x + y)2
(x + y)2
xy
4
(x + y)2
1 + x + y + xy 1 + x + y +
4
149
150 12. Soluciones

 2
x+y
(1 + x)(1 + y) 1 +
2
(1 + x)(1 + y) (1 + a)2 (12.2)

Luego de (12.1) y (12.2) se tiene:

(1 + b)2 (1 + x)(1 + y) (1 + a)2 .

2. Pruebe que
    2
1 1 2
1+ 1+ 1+
x y x+y

para todo x, y n
umeros reales positivos.

 Prueba. La desigualdad es equivalente a:

1 1 1 4 4
1+ + + 1+ +
x y xy x + y (x + y)2
(x + y + 1)(x + y)2 4(x + y)xy + 4xy
(x + y + 1)(x + y)2 4xy(x + y + 1)
(x + y)2 4xy
(x y)2 0.
2

3. Sean x, y, z n
umeros reales positivos, pruebe que al menos una de las siguientes
desigualdades es verdadera:

xy 0
yz 0
zx 0.
151

 Prueba. Supongamos que no se cumple ninguna de las desigualdades,


esto es

xy < 0
yz < 0
zx < 0.

Luego tendramos que xy > 0, yz > 0, zx > 0, y multiplicando x2 y 2 z 2 >


0, lo cual es una contradiccion. Por lo tanto una de las desigualdades debe ser
verdadera. 2

4. Si a2 + b2 = 1, halle la variacion de 2a + 3b.

Soluci
on. Escogemos los pares (a, b) y (2, 3) para utilizar la desigualdad de
Cauchy-Schwarz.
En efecto

(2a + 3b)2 (a2 + b2 ) (22 + 32 )


(2a + 3b)2 (1)(13)
(2a + 3b)2 13


de donde 13 2a + 3b 13. 2

5. Sabiendo que 2a + 3b + 4c = 29, halle el menor valor de a2 + b2 + c2 .

Soluci
on. Tomamos las ternas (2, 3, 4) y (a, b, c), luego aplicamos Cauchy-
Schwarz:

(2a + 3b + 4c)2 (22 + 32 + 42 ) (a2 + b2 + c2 )


(29)2 (29) (a2 + b2 + c2 )
a2 + b2 + c2 29

El mnimo de a2 + b2 + c2 es 29 y ocurre si y solo si a = 2, b = 3, c = 4. 2


152 12. Soluciones

6. Pruebe que
p p
m2 + y 2 + n2 + x2 (m + n)2 + (x + y)2

para todo x, y, m, n n
umeros reales.

 Prueba. Aplicando Cauchy-Schwarz a (m, y) y (n, x), tenemos

(m2 + y 2)(n2 + x2 ) (mn + xy)2


p
(m2 + y 2 )(n2 + x2 ) (mn + xy)
p
(m2 + y 2) + (n2 + x2 ) + 2 (m2 + y 2 )(n2 + x2 ) (m2 + n2 + 2mn)

+(x2 + y 2 + 2xy)
p 2
m2 + y2 + n2 + x2 (m + n)2 + (x + y)2
p p
m2 + y 2 + n2 + x2 (m + n)2 + (x + y)2.

7. Si a, b, c son n
umeros reales positivos que satisfacen abc = 1, pruebe que

1 + ab 1 + bc 1 + ac
+ + 3.
1+a 1+b 1+c

 Prueba.

1 + ab c + abc c+1
= = ; similarmente
1+a c(1 + a) c(1 + a)
1 + bc a+1
=
1+b a(1 + b)
1 + ac b+1
=
1+c b(1 + c)
153

1
vemos que al multiplicar las tres u
ltimas fracciones obtenemos = 1, en-
abc
tonces aplicando MA MG, tenemos:

c+1 a+1 b+1


+ + r
c(1 + a) a(1 + b) b(1 + c) c+1 a+1 b+1
3

3 c(1 + a) a(1 + b) b(1 + c)
r
3 1
= =1
cab

entonces

c+1 a+1 b+1


+ + 3
c(1 + a) a(1 + b) b(1 + c)

1 + ab 1 + bc 1 + ac
+ + 3.
1+a 1+b 1+c
2

8. Sean a, b, c n
umeros reales positivos, pruebe que

a b c 3
+ + .
b+c c+a a+b 2

(Desigualdad de Nesbit).

 Prueba. La desigualdad es equivalente a:


     
a b c 3
+1 + +1 + +1 +3
b+c c+a a+b 2
a+b+c a+b+c a+b+c 9
+ +
b+c c+a a+b 2
 
1 1 1 9
(a + b + c) + +
b+c c+a a+b 2
154 12. Soluciones

 
1 1 1
2(a + b + c) + + 9
b+c c+a a+b
 
1 1 1
((b + c) + (c + a) + (a + b)) + + 9
b+c c+a a+b
(b + c) + (c + a) + (a + b) 3

3 1 1 1
+ +
b+c c+a a+b

ltima desigualdad es verdadera, pues MA MH.


esta u 2

9. Pruebe que si x, y, z son n


umeros reales positivos y x + y + z = 1, entonces
   
1 1 1
1+ 1+ 1+ 64.
x y z

 Prueba. La desigualdad es equivalente a:


   
x+1 y+1 z+1
64
x y z
(x + x + y + z)(y + x + y + z)(z + x + y + z) 64xyz
(2x + y + z)(x + 2y + z)(x + y + 2z) 64xyz

aplicando MA MG:

x+x+y+z
4xxyz
4
p
2x + y + z 4 4 x2 yz; similarmente
p
x + 2y + z 4 4 xy 2 z
p
x + y + 2z 4 4 xyz 2

multiplicando miembro a miembro:

(2x + y + z)(x + 2y + z)(x + y + 2z) 64xyz.

2
155

umeros reales positivos tales que a2 + b2 = (c2 + d2 )3 , pruebe


10. Sean a, b, c, d n
que

c3 d 3
+ 1.
a b

 Prueba. El primer miembro de la desigualdad

2 2
c3 d 3 c4 d4 (c2 ) (d2 )
+ = + = +
a b ac bd ac bd
luego, utilizando el Lema de Titu, tenemos

 2 !2 !2
2 2 2
c3 d 3 (c2 ) (d2 ) (c2 + d2 )
+ = +
a b ac bd ac + bd
4
(c2 + d2 )
=
(ac + bd)2
3
(c2 + d2 ) (c2 + d2 )
=
(ac + bd)2
(a2 + b2 ) (c2 + d2 )
=
(ac + bd)2

entonces
 2
c3 d 3 (a2 + b2 ) (c2 + d2 )
+ 1
a b (ac + bd)2

pues por Cauchy-Schwarz


 
a2 + b2 c2 + d2 (ac + bd)2 .

11. Sean a, b, c n
umeros reales positivos, pruebe que

a b c
+ + 1.
b + 2c c + 2a a + 2b
156 12. Soluciones

 Prueba. Vemos que

a b c a2 b2 c2
+ + = + +
b + 2c c + 2a a + 2b ab + 2ac bc + 2ab ac + 2bc

aplicando el Lema de Titu, tenemos

a2 b2 c2 (a + b + c)2
+ + ,
ab + 2ac bc + 2ab ac + 2bc 3(ab + ac + bc)

luego bastara demostrar que

(a + b + c)2
1
3(ab + ac + bc)
a2 + b2 + c2 + 2(ab + bc + ca) 3(ab + ac + bc)
a2 + b2 + c2 ab + ac + bc,

pero esta u
ltima desigualdad es verdadera.

a b c
+ + 1.
b + 2c c + 2a a + 2b
2

12. Pruebe que

x2 y2
+ 8,
y1 x1

para todo x, y > 1.

 Prueba. Como (x 2)2 0, para todo x real, en particular para x > 1,


efectuando

x2 4x + 4 0
x2 4(x 1)

x2 x1
x
2
x1
157

similarmente

y
2
y1

x y
como , R+ ; entonces aplicando MA MG
x1 y1
x2 y2 s
+
y1 x1 x2 y2

2 y1 x1
x2 y2 xy x y
+ 2 = 2
y1 x1 x1 y1 x1 y1
2(2)(2) = 8

x2 y2
+ 8.
y1 x1
2

13. Sean a, b, c n
umeros reales positivos tales que a + b + c = 2, pruebe que

a+b b+c c+a 9


+ + .
2a + bc 2b + ac 2c + ab 4

 Prueba. Veamos la fraccion

a+b a+b a+b a+b 1


= = 2 = = .
2a + bc (a + b + c)a + bc a + (b + c)a + bc (a + b)(a + c) a+c

Similarmente para las otras dos fracciones y la desigualdad se escribe as:

a+b b+c c+a 9


+ +
(a + b)(a + c) (b + a)(b + c) (c + a)(c + b) 4
1 1 1 9
+ + .
a+c b+a c+b 4
158 12. Soluciones

Pero
 
1 1 1
[(a + c) + (b + a) + (c + b)] + + 9
a+c b+a c+b
 
1 1 1
2(a + b + c) + + 9
a+c b+a c+b
 
1 1 1
4 + + 9
a+c b+a c+b
1 1 1 9
+ +
a+c b+a c+b 4

a+b b+c c+a 9


+ + .
2a + bc 2b + ac 2c + ab 4
2

umeros reales tales que a2 + b2 = 1, pruebe que


14. Si a, b son n

2 2
a b + ab2 .
2

 Prueba. 2
a b + ab2 = |ab(a + b)| = |ab||a + b|

pero


|a + b| 2 a2 + b2 = 2 1 = 2

|a + b| 2 (12.3)

ademas

a2 + b2 2|a||b|
1
|ab|
2
1
|ab| (12.4)
2
159

multiplicando miembro a miembro:



2
|ab||a + b|
2


2
a2 b + ab2 .
2
2

15. Sean x, y n
umeros reales positivos, pruebe que

x y
+ 1.
x + 3y y + 3x

 Prueba. Usando la identidad (a + b)2 = a2 + 2ab + b2 , en efecto

(a + b)2 = a2 + 2ab + b2

3
= a2 + ab + ab + b2 a2 + 3 a2 b4
(a + b)2 a2 + 3a2/3 b4/3

= a2/3 a4/3 + 3b4/3
p
(a + b) a2/3 (a4/3 + 3b4/3 ) = a1/3 a4/3 + 3b4/3

Haciendo a4/3 = x, b4/3 = y, tenemos:



3/4 3/4 1/4
x x3/4
x +y x x + 3y 3/4
x + 3y x + y 3/4

y y 3/4
similarmente 3/4
y + 3x x + y 3/4

sumando obtenemos

x y
+ 1.
x + 3y y + 3x
2
160 12. Soluciones

umeros reales positivos tales que ab + bc + ca 3abc, pruebe que


16. Sean a, b, c n

a + b + c a3 + b3 + c3 .

 Prueba. Como ab + bc + ca 3abc

ab + bc + ca
3
abc
1 1 1
+ + 3
a b c
 
1 1 1
Pero (a + b + c) + + 9
a b c

a+b+c3

ademas

3(a + b + c) (a + b + c)2
2
= a3/2 a1/2 + b3/2 b1/2 + c3/2 c1/2
(a3 + b3 + c3 ) (a1 + b1 + c1 )
(a3 + b3 + c3 ) 3


3(a + b + c) 3 a3 + b3 + c3

a + b + c a3 + b3 + c3 .

umeros reales positivos tales que a2 + b2 + c2 = 3, pruebe que


17. Sean a, b, c n

1 1 1
+ + 1.
1 + 2ab 1 + 2bc 1 + 2ca
161

 Prueba. Recordar que

a2 + b2 2ab
1 + a2 + b2 1 + 2ab
1 1

1 + 2ab 1 + a2 + b2

en efecto

1 1 1 12 12 12
+ + + +
1 + 2ab 1 + 2bc 1 + 2ca 1 + a2 + b2 1 + b2 + c2 1 + a2 + c2

aplicando el Lema de Titu

12 12 12 (1 + 1 + 1)2
+ +
1 + a2 + b2 1 + b2 + c2 1 + a2 + c2 3 + 2(a2 + b2 + c2 )
32
= =1
3 + 2(3)

1 1 1
+ + 1.
1 + 2ab 1 + 2bc 1 + 2ca
2

umeros reales positivos tales que a2 + b2 + c2 = 3, pruebe que


18. Sean a, b, c n

1 1 1 3
+ + .
1 + ab 1 + bc 1 + ca 2

 Prueba.

2
b2 + c}2 ab + ac + bc
|a + {z
3

3 ab + ac + bc
6 (1 + ab) + (1 + ac) + (1 + bc)
9 9

6 3 + ab + ac + bc
162 12. Soluciones

9 3

3 + ab + ac + bc 2

ademas sabemos que

 
1 1 1
[(1 + ab) + (1 + ac) + (1 + bc)] + + 9
1 + ab 1 + ac 1 + bc
1 1 1 9
+ +
1 + ab 1 + ac 1 + bc 3 + ab + ac + bc

1 1 1 3
+ + .
1 + ab 1 + bc 1 + ca 2
2

19. Pruebe que



(a + b + c)4 8 a2 + b2 + c2 (ab + ac + bc).

para todo a, b, c n
umeros reales.

 Prueba.

2
(a + b + c)4 = ((a + b + c)2 )
2
2
= a b2 + c}2 + 2(ab + ac + bc)
| + {z | {z }
x y
2
= (x + y)
4xy
= 4 (a2 + b2 + c2 ) (2)(ab + ac + bc)
= 8 (a2 + b2 + c2 ) (ab + ac + bc)


(a + b + c)4 8 a2 + b2 + c2 (ab + ac + bc).

2
163

20. Dados a, b, c n
umeros reales positivos con abc = 8, pruebe que

1 1 1
+ + 1.
1+a3 1+b3 1 + c3

 Prueba. La suma de cubos lo descomponemos para aplicar MA MG.


Veamos

1 + a3 = (1 + a)(1 a + a2 ),

entonces

(1 + a) + (1 a + a2 ) p
(1 + a)(1 a + a2 )
2
2 + a2
1 + a3
2
2 1
2
;
2+a 1 + a3

luego

1 1 1 2 2 2
+ + 2
+ 2
+
1+a3 1+b3 1+c 3 2+a 2+b 2 + c2

como vemos basta probar que

2 2 2
2
+ 2
+ 1
2+a 2+b 2 + c2
1 1 1 1
2
+ 2
+ 2

2+a 2+b 2+c 2
(2 + b2 ) (2 + c2 ) + (2 + a2 ) (2 + c2 ) + (2 + a2 ) (2 + b2 )
1
(2 + a2 ) (2 + b2 ) (2 + c2 )
2
164 12. Soluciones

12 + 4 (a2 + b2 + c2 ) + |b2 c2 + a{z


2 2
c + a2 b}2
 
1 2 2 2 2 2 2 2 2 2
 2 2 2
8 + 4 (a + b + c ) + 2 a b + a c + b c +a b c
2 | {z }
1
2 (a2 + b2 + c2 ) + 8 a2 b2 c2 ; abc = 8
2
2 (a2 + b2 + c2 ) + 8 32

a2 + b2 + c2 12.

Es verdadera pues

3
a2 + b2 + c2 3 a2 b2 c2 = 12.

21. Sean a, b n
umeros reales positivos, pruebe que

1 1 1
2
+ 2
.
(1 + a) (1 + b) 1 + ab

 Prueba.

1 1 1
2
+ 2
;
(1 + a) (1 + b) 1 + ab

[(1 + a)2 + (1 + b)2 ] (1 + ab) (1 + a)2 (1 + b)2

(2 + 2a + 2b + a2 + b2 ) (1 + ab) (1 + 2a + a2 )(1 + 2b + b2 )

a3 b + ab3 + 1 2ab + a2 b2

ab (a2 + b2 2ab) + (a2 b2 2ab + 1) 0

ab(a b)2 + (ab 1)2 0.

La igualdad ocurre si y solo si a = b = 1. 2


165

22. Sean a, b, c, d n
umeros reales positivos tales que abcd = 1, pruebe que

1 1 1 1
+ + + 1.
(1 + a)2 (1 + b)2 (1 + c)2 (1 + d)2

 Prueba. Aplicando el problema 21, tenemos

1 1 1 1 1 1
2
+ 2
+ 2
+ 2
+
(1 + a) (1 + b) (1 + c) (1 + d) 1 + ab 1 + cd
| {z } | {z }
1 ab
= +
1 + ab ab + abcd
1 ab
= + = 1.
1 + ab ab + 1

1 1 1 1
2
+ 2
+ 2
+ 1.
(1 + a) (1 + b) (1 + c) (1 + d)2

La igualdad ocurre si y solo si a = b = c = d = 1. 2

23. Sean a, b, c n
umeros reales positivos, pruebe que

2 (a3 + b3 + c3 ) + (3a + 1) (b2 + c2 ) + (3b + 1) (a2 + c2 ) + (3c + 1) (a2 + b2 )


2(ab + ac + bc) + 3(a + b)(a + c)(b + c)

 Prueba. (a b)2 0, para todo a, b reales, luego (a b)2 + (a + b)3


(a + b)3 ; efectuando

a2 2ab + b2 + a3 + b3 + 3a2 b + 3ab2 (a + b)3


a3 + b3 + a2 (3b + 1) + b2 (3a + 1) (a + b)3 + 2ab; similarmente
a3 + c3 + a2 (3c + 1) + c2 (3a + 1) (a + c)3 + 2ac
b3 + c3 + b2 (3c + 1) + c2 (3b + 1) (b + c)3 + 2bc
166 12. Soluciones

sumando miembro a miembro:

2 (a3 + b3 + c3 ) + (3a + 1) (b2 + c2 ) + (3b + 1) (a2 + c2 ) + (3c + 1) (a2 + b2 )


(a + b)3 + (a + c)3 + (b + c)3 +2ab + 2ac + 2bc
| {z }
3(a + b)(a + c)(b + c) + 2(ab + ac + bc).

24. (IMO 1995). Sean a, b, c n


umeros reales positivos tales que abc = 1, pruebe
que

1 1 1 3
+ 3 + 3 .
a3 (b+ c) b (c + a) c (a + b) 2

 Prueba. Como abc = 1, entonces la desigualdad es equivalente a:

abc abc abc 3


+ 3 + 3
a3 (b
+ c) b (c + a) c (a + b) 2
1 1 1
a2 2 2 3
+ b + c .
1 1 1 1 1 1 2
+ + +
b c b c b c
1 1 1
Haciendo = x, = y, = z, xyz = 1. Luego basta demostrar que
a b c
x2 y2 z2 3
+ + ,
y+z x+z x+y 2

aplicando el Lema de Titu:

x2 y2 z2 (x + y + z)2
+ +
y+z x+z x+y (y + z) + (x + z) + (x + y)
(x + y + z)2 x+y+z 3 3
= = 3 xyz = .
2(x + y + z) 2 2 | {z } 2
1

2
167

25. Demuestre que

     
2 1 1 2 1 1 2 1 1
a + +b + +c + 2(a + b + c)
b c a c a b

para todo a, b, c n
umeros reales positivos.

 Prueba. Como

a2 + b2 2ab
a2 ab + b2 ab; multiplicando por(a + b)
(a + b) (a2 ab + b2 ) ab(a + b)
a3 + b3 ab(a + b)
a3 b3
+ a+b
ab ab
a2 b2
+ a + b; similarmente
b a
a2 c2
+ a+c
c a
b2 c2
+ b+c
c b

sumando miembro a miembro y agrupando

     
2 1 1 2 1 1 2 1 1
a + +b + +c + 2(a + b + c)
b c a c a b

26. Sean x, y, z n
umeros reales que pertenecen al intervalo [1, 1] tales que x +
y + z = 0, pruebe que

p p
1 + x + y2 + 1 + y + z2 + 1 + z + x2 3.
168 12. Soluciones


 Prueba. Si ab 0, veamos que 1+a+ 1+b1+ 1 + a + b.
En efecto

1+a+ 1+b 1+ 1+a+b
p
2 + a + b + 2 (1 + a)(1 + b) 1 + 2 1 + a + b + 1 + a + b

1 + a + b + ab 1+a+b

1 + a + b + ab 1 + a + b

ab 0.

Para aplicar la afirmacion anterior, veamos el signo de x + y 2, y + z 2 , z + x2 .


Como x+y 2 +y +z 2 +z +x2 = x + y + z +x2 +y 2 +z 2 0, entonces escogemos
| {z }
0
dos de ellos del mismo signo, sin perdida de generalidad podemos escoger x+y 2
y y + z 2 . Luego tenemos que
p p
1 + x + y2 + 1 + y + z2 + 1 + z + x2
p
1+ 1 + x + y2 + y + z2 + 1 + z + x2
q
= 1+ 1 + x + y +y 2 + z 2 + 1 + z + x2
| {z }
p
= 1+ 1 z + z2 + y2 + 1 + z + x2
q q
2 2
= 1+ 1z+ z2 + y2 + 1 + z + x2
q 2
1+ 1 z + z2 + 1+z + (x + y)2
q 2
= 1+ 1 z + z2 + 1+z + (z)2
q 2
= 1+ 1 z + z2 + 1+z + z2
169

luego basta demostrar que


2
1 z + z2 + 1 + z + z2 4
p
1 z + z 2 + 1 + z + 2 (1 z + z 2 )(1 + z) + z 2 4
p
2z 2 + 2 (1 z + z 2 )(1 + z) 2
p
z 2 + (1 z + z 2 )(1 + z) 1
p 2
(1 z + z 2 )(1 + z) 1|
{zz};
(+)

elevando al cuadrado

1 + z 3 1 2z 2 + z 4
z 3 z 4 2z 2
z 4 z 3 2z 2 0
z 2 (z 2 z 2) 0
z 2 (z 2)(z + 1) 0
z 2 (2 z)(z + 1) 0

es verdadero pues |z| 1.


La igualdad ocurre si y solo si x = y = z = 0. 2

27. Sean a, b, c n
umeros reales, pruebe que

a+b+c+ a2 + b2 + c2 ab bc ca 3 max {a, b, c}. (12.5)

 Prueba. Supongamos que max {a, b, c} = a

a b; a c.

Luego (12.5) es equivalente a:



a + b + c + a2 + b2 + c2 ab bc ca 3a

a2 + b2 + c2 ab bc ca 2a
| {zb }c
(+)
170 12. Soluciones

elevando al cuadrado

a2 + b2 + c2 ab bc ca 4a2 + b2 + c2 4ab 4ac + 2bc


0 3a2 3ab 3ac + 3bc
a2 a(b + c) + bc 0
(a b)(a c) 0.
2

28. Dados x, y, z n
umeros reales positivos tales que x + y + z = 1, pruebe que
r
x y z 3
+ + .
y+z z+x x+y 2

x
 Prueba. Definimos la funcion f (x) = ; x < 1, entonces
1x
2x
f (x) =
2(1 x) 1 x

1 x(4 x)
f (x) = >0
4(1 x)3

luego f es convexa; entonces


 
x+y+z 1
f (f (x) + f (y) + f (z))
3 3
x+y+z  
3 1 x y z
r + +
x+y+z 3 1x 1y 1z
1
3

reemplazando x + y + z = 1, tenemos

1  
1 x y z
r 3 + +
1 3 1x 1y 1z
1
3
171

r
x y z 3
+ + .
y+z z+x x+y 2

1
La igualad ocurre si y solo si x = y = z = . 2
3

29. (IMO 1964). Sean a, b, c los lados de un triangulo, pruebe que

a2 (b + c a) + b2 (a + c b) + c2 (a + b c) 3abc.

 Prueba. Efectuando en el primer miembro

a2 (b + c) a3 + b2 (a + c) b3 + c2 (a + b) c3 3abc
a2 (b + c) + b2 (a + c) + c2 (a + b) a3 + b3 + c3 + 3abc
0 (a3 a2 (b + c) + abc) + (b3 b2 (a + c) + abc) + (c3 c2 (a + b) + abc)
0 a (a2 a(b + c) + bc) + b (b2 b(a + c) + ac) + c (c2 c(a + b) + ab)
0 a(a b)(a c) + b(b a)(b c) + c(c a)(c b)
a(a b)(a c) + b(b a)(b c) + c(c a)(c b) 0

es verdadera, pues es la desigualdad de Sch


ur. 2

umeros reales tales que a2 + b2 = 1, pruebe que


30. Dados a, b n

3 3
ab + max {a; b} . (12.6)
4

 Prueba. Sin perdida de generalidad, supongamos que

max {a; b} = a

entonces la desigualdad (12.6) es equivalente a:



3 3
ab + a .
4
172 12. Soluciones

Definamos la funcion

f (a, b) = ab + a = a(b + 1)
f 2 (a, b) = a2 (b + 1)2 = (1 b2 ) (1 + b)2
= (1 b)(1 + b)3
1
= (3 3b)(1 + b)3
3

aplicando MG MG
 4
1 3 1 3 3b + 1 + b + 1 + b + 1 + b
(3 3b)(1 + b)
3 3 4
 4
1 3 27
= =
3 2 16

27
f 2 (a, b) .
16

3 3
f (a, b) .
4
2

31. Dados a, b, c, d n
umeros reales no negativos, pruebe que

a b c d
+ 2 + 2 + 2 2.
b2 2
+c +d 2 2
c +d +a2 2
d +a +b2 a + b2 + c2

1+x
 Prueba. Recordar que x , entonces tenemos que
2

r b2 + c2 + d2
2
b +c +d 2 2 1+ a2 + b2 + c2 + d2
a2 =
a2 2 2a2
173

a 2a2
; similarmente
b + c2 + d 2
2 a2 + b2 + c2 + d2
b 2b2
2
c + d2 + a2
2 a + b2 + c2 + d2
c 2c2
2
d + a2 + b2
2 a + b2 + c2 + d2
d 2d2
2
a + b2 + c2
2 a + b2 + c2 + d2

sumando miembro a miembro se obtiene

a b c d
+ 2 + 2 + 2 2.
b2 2
+c +d 2 2
c +d +a2 2
d +a +b2 a + b2 + c2
2

umeros reales no negativos tales que a2 +b2 +c2 +d2 = 4, pruebe


32. Sean a, b, c, d n
que

a3 + b3 + c3 + d3 8.

 Prueba. Si utilizamos la desigualdad de la media potencial tendramos


 1/2  3 1/3
a2 + b2 + c2 + d2 a + b3 + c3 + d3

4 4
 1/2  3 1/3
4 a + b3 + c3 + d3

4 4

de donde a3 + b3 + c3 + d3 1
como vemos no hemos acotado superiormente, es decir esta propiedad no ayuda
resolver el problema; buscaremos otra opcion.
De la condicion a2 + b2 + c2 + d2 = 4, se deduce que

a2 4;
a 2; a 0
174 12. Soluciones

a3 2a2 ; similarmente

b3 2b2

c3 2c2

d3 2d2

sumando miembro a miembro:

a3 + b3 + c3 + d3 2 (a2 + b2 + c2 + d2 )

a3 + b3 + c3 + d3 2 (4)

de donde a3 + b3 + c3 + d3 8.
La igualdad ocurre si y solo si (a, b, c, d) = (2, 0, 0, 0), y sus respectivas per-
mutaciones. [8, pag 15] 2

33. Sean a, b, c los lados de un triangulo, pruebe que

a b c
+ + 1.
3a b + c 3b c + a 3c a + b

 Prueba. Multiplicando por 4 a los miembros de la desigualdad, tenemos


que

a b c
+ + 1
3a b + c 3b c + a 3c a + b
4a 4b 4c
+ + 4
3a b + c 3b c + a 3c a + b
     
4a 4b 4c
1 + 1 + 1 1
3a b + c 3b c + a 3c a + b
a+bc b+ca c+ab
+ + 1
3a b + c 3b c + a 3c a + b
175

En el primero miembro de la desigualdad generamos las condiciones para


aplicar el Lema de Titu

a+bc b+ca c+ab


+ +
3a b + c 3b c + a 3c a + b
(a + b c)2 (b + c a)2 (c + a b)2
= + +
(a + b c)(3a b + c) (b + c a)(3b c + a) (c + a b)(3c a + b)
(a + b c + b + c a + c + a b)2

(a + b c)(3a b + c) + (b + c a)(3b c + a) + (c + a b)(3c a + b)
| {z }
(a+b+c)2

(a + b + c)2
= = 1.
(a + b + c)2

34. Pruebe que si a, b, c son n


umeros reales positivos tales que abc = 1,entonces

1 1 1 1 1 1
+ + + + . (12.7)
1+a+b 1+b+c 1+c+a 2+a 2+b 2+c

 Prueba. Sean x = a + b + c, y = ab + bc + ca, como abc = 1, entonces

x=a+b+c3
x 3; y 3
x2 3y

pues (a + b + c)2 3(ab + ac + bc).


La desigualdad (12.7) es equivalente a:

(1 + b + c)(1 + c + a) + (1 + a + b)(1 + c + a) + (1 + a + b)(1 + b + c)


(1 + a + b)(1 + b + c)(1 + c + a)
(2 + b)(2 + c) + (2 + a)(2 + c) + (2 + a)(2 + b)

(2 + a)(2 + b)(2 + c)
176 12. Soluciones

(a + b + c)2 + 4(a + b + c) + (ab + ac + bc) + 3



(a + b + c)2 + 2(a + b + c) + (ab + ac + bc) + (a + b + c)(ab + ac + bc)
12 + 4(a + b + c) + (ab + ac + bc)

9 + 4(a + b + c) + 2(ab + ac + bc)

Reemplazando tenemos

x2 + 4x + y + 3 12 + 4x + y
2

x + 2x + y + xy 9 + 4x + 2y
x2 + 4x + y + 3 12 + 4x + y
2
1 1
x + 2x + y + xy 9 + 4x + 2y
2x xy + 3 3y
1
x2 + 2x + y + xy 9 + 4x + 2y
5x2 + 24x + y 2 + 3y + 27 6xy + 3x2 y + xy 2
5 1
5x2 + 24x + y 2 + 3y + 27 5xy + xy + x2 y + x2 y + x2 y + xy 2
3 3

esta desigualdad es verdadera, pues

5 1
5x2 x2 y; y 2 x2 y; 9x xy 2
3 3
15x 5xy; 3y xy; 27 x2 y.

umeros reales positivos que satisfacen abc 1,


35. (IMO 2005). Dados a, b, c n
pruebe que

a5 a2 b5 b2 c5 c2
+ + 0.
a5 + b2 + c2 b5 + c2 + a2 c5 + a2 + b2
177

 Prueba. La desigualdad es equivalente a:

     
a5 a2 b5 b2 c5 c2
1 + 5 1 + 5 1 3
a5 + b2 + c2 b + c2 + a2 c + a2 + b2
(a2 + b2 + c2 ) (a2 + b2 + c2 ) (a2 + b2 + c2 )
5 5 3
a5 + b2 + c2 b + c2 + a2 c + a2 + b2
(a2 + b2 + c2 ) (a2 + b2 + c2 ) (a2 + b2 + c2 )
+ 5 + 5 3
a5 + b2 + c2 b + c2 + a2 c + a2 + b2

La dificultad se presenta en los denominadores, por la potencia quinta, por ello


es conveniente deshacernos de dicha potencia, en efecto, aplicando Cauchy-
Schwarz.
2
(a5 + b2 + c2 ) (bc + b2 + c2 ) a5/2 b1/2 c1/2 + b2 + c2
2
= (abc)1/2 a2 + b2 + c2
2
(a2 + b2 + c2 ) ; pues abc 1

de donde

2
(a5 + b2 + c2 ) (bc + b2 + c2 ) (a2 + b2 + c2 )
bc + b2 + c2 a2 + b2 + c2

a2 + b2 + c2 a5 + b2 + c2
a2 + b2 + c2 bc + b2 + c2
5 2 ; similarmente
a + b2 + c2 a + b2 + c2
a2 + b2 + c2 ac + a2 + c2
2 ;
b5 + c2 + a2 a + b2 + c2
a2 + b2 + c2 ab + a2 + b2
2
c5 + a2 + b2 a + b2 + c2

sumando miembro a miembro tenemos:

a2 + b2 + c2 a2 + b2 + c2 a2 + b2 + c2
+ +
a5 + b2 + c2 b5 + c2 + a2 c5 + a2 + b2
bc + ac + ab + 2 (a2 + b2 + c2 ) 3 (a2 + b2 + c2 )
= 3.
a2 + b2 + c2 a2 + b2 + c2
178 12. Soluciones

Recordar que

ab + ac + bc a2 + b2 + c2 .

La igualdad ocurre si y solo si a = b = c. 2

36. Sean a, b, c n
umeros reales tales que a + b + c = 3, pruebe que


a+ b+ c ab + bc + ca.

 Prueba. Como 2(ab + bc + ca) = (a + b + c)2 (a2 + b2 + c2 ) entonces la


desigualdad es equivalente a:


a2 + b2 + c2 + 2( a + b + c) (a b + }c)2 = 9
| +{z
3

agrupando convenientemente en el primer miembro

  2  
a + 2 a + b + 2 b + c2 + 2 c 9.
2

Pero
p
a2 + 2 a = a2 + a + a 3 3 a2 a a = 3a

entonces

a2 + 2 a 3a; similarmente

b2 + 2 b 3b

c2 + 2 c 3c

sumando miembro a miembro, tenemos:



a2 + b2 + c2 + 2( a + b + c) 3(a + b + c)

a2 + b2 + c2 + 2( a + b + c) 9.
2
179

37. (APMO 1998). Sean x, y, z n


umeros reales positivos, pruebe que
 
x  y z 2(x + y + z)
1+ 1+ 1+ 2+ .
y z x 3 xyz

 Prueba. Efectuando en el primer miembro de la desigualdad

x y z x z y 2(x + y + z)
1+ + + + + + +12+
y z x z y x 3 xyz

simplificando

x y z x z y 2(x + y + z)
+ + + + +
y z x z y x 3 xyz

Aplicando MA MG convenientemente
s s
2
x y x x y x x3 3x
2 + = + + 33 =33 =
y z y y z yz xyz 3 xyz
x y 3x
2 + ; similarmente
y z 3 xyz
y z 3y
2 +
z x 3 xyz

z x 3z
2 +
x y 3 xyz

sumando las tres desigualdades:


   
x y z x y z
3 + + 3 + +
y z x 3 xyz 3 xyz 3 xyz

x y z x+y+z
+ + , similarmente
y z x 3 xyz

x z y x+y+z
+ + .
z y x 3 xyz

Sumando miembro a miembro obtenemos la desigualdad que queramos de-


mostrar. 2
180 12. Soluciones

38. (USAMO 1998). Sean a, b, c n


umeros reales positivos, pruebe que

1 1 1 1
+ 3 + 3 .
a3 b3 3 3
+ + 3abc b + c + 3abc c + a + 3abc abc

 Prueba. Sabemos que

a2 + b2 2ab, restando ab
a2 ab + b2 ab, multiplicando por (a + b)
(a + b) (a2 ab + b2 ) ab(a + b)
a3 + b3 ab(a + b), sumando abc
a3 + b3 + abc ab(a + b + c)
1 1

a3 3
+ b + abc ab(a + b + c)
abc abc

a3 3
+ b + abc ab(a + b + c)
abc c
, similarmente
a3 3
+ b + abc a+b+c
abc a

b3 + c3 + abc a+b+c
abc b
3 3

c + a + abc a+b+c

sumando miembro a miembro:

abc abc abc


+ 3 + 3 1
a3 + b + 3abc b + c + 3abc c + a3 + 3abc
3 3

de donde
1 1 1 1
+ 3 + 3 .
a3 b3 3 3
+ + 3abc b + c + 3abc c + a + 3abc abc

2
181

39. Sean a, b, c reales positivos con suma 3, pruebe que

a b c 3
+ + .
1 + b2 1 + c2 1 + a2 2

 Prueba. Antes de dar la prueba, notemos que la MA MG en algunos


casos no ayuda a solucionar el problema. Veamos:

a b c a b c
2
+ 2
+ 2
+ +
1+b 1+c 1+a 2b 2c 2a

a b c 3
Pero aplicando MA MG, tenemos + + , y no podemos
2b 2c 2a 2
a b c 3
garantizar que + + .
1 + b2 1 + c2 1 + a2 2
Por ello utilizaremos otro procedimiento, veamos:

a ab2 ab2 ab
=a a =a ,
1 + b2 1+b 2 2b 2
a ab
a , similarmente
1 + b2 2
b bc
b ,
1 + c2 2
c ca
c ,
1 + a2 2

sumando miembro a miembro, tenemos


 
a b c ab bc ac
2
+ 2
+ a+b+c + +
1+b 1+c 1 + a2 2 2 2
1
= 3 (ab + bc + ac)
2

luego es suficiente demostrar que

1 3
3 (ab + bc + ac) .
2 2
182 12. Soluciones

Sabemos que

(a + b + c)2 3(ab + bc + ac)


32 3(ab + bc + ac)
3 ab + bc + ac
1 3
(ab + bc + ac)
2 2
1 3 3
3 (ab + bc + ac) 3 = . [9, pag 27]
2 2 2
2

40. Sean a, b, c, d n
umeros reales positivos, pruebe que

a3 b3 c3 d3 a+b+c+d
2 2
+ 2 2
+ 2 2
+ 2 2
.
a +b b +c c +d d +a 2

 Prueba. La idea del problema anterior es interesante y la vamos a utilizar.


En efecto

a3 ab2
= a .
a2 + b2 a2 + b2

Pero

1 1
a2 + b2 2ab
a2
+b 2 2ab
2
ab ab2
2
a + b2 2ab

entonces

a3 ab2 b
2 2
=a 2 2
a
a +b a +b 2
183

a3 b
2 2
a , similarmente
a +b 2
3
b c
2 2
b
b +c 2
3
c d
2 2
c
c +d 2
3
d a
2 2
d
d +a 2

sumando miembro a miembro, tenemos

a3 b3 c3 d3 a+b+c+d
2 2
+ 2 2
+ 2 2
+ 2 2
. [9, pag 28]
a +b b +c c +d d +a 2
2

41. Sean a, b, c, d n
umeros reales positivos tales que abcd = 1, pruebe que

1 1 1
S = + +
1 + ab + bc + ca 1 + bc + cd + ab 1 + cd + da + ac
1
+ 1.
1 + da + ab + bd

 Prueba. Como necesitamos formar ab + bc + ca y aprovechando abcd = 1


partimos de

1 1 1 1 1 1
+ + + +
a b c ab ac bc

cd bd ad
= + +
abcd abcd abcd

= cd + bd + ad

= d( c + b + a)

bc + ac + ab d( a + b + c)

abc d d

a+ b+ c
bc + ac + ab
d
184 12. Soluciones


a+b+ c+ d
1 + bc + ac + ab
d

1 d
, similarmente
1 + bc + ac + ab a+ b+ c+ d

1 a

1 + bc + cd + db a+ b+ c+ d

1 b

1 + cd + da + ac a+ b+ c+ d

1 c

1 + da + ab + bd a+ b+ c+ d

sumando miembro a miembro, tenemos

S 1.

42. Sean a, b, c n
umeros no negativos tales que a + b + c = 3, pruebe que

1 1 1
+ + 1.
2ab2 +1 2bc2 2
+ 1 2ca + 1

 Prueba. Efectuando tenemos la desigualdad equivalente:

(2bc2 + 1) (2ca2 + 1) + (2ab2 + 1) (2ca2 + 1) + (2ab2 + 1) (2bc2 + 1)


(2ab2 + 1) (2bc2 + 1) (2ca2 + 1)
1 + a2 c + b2 a + bc2 4(abc)3 0

como

a2 c + b2 a + bc2 p
3 (abc)3
3
a c + b a + bc2 3abc
2 2
185

luego

1 + a2 c + b2 a + bc2 4(abc)3 1 + 3abc 4(abc)3

basta demostrar

1 + 3abc 4(abc)3 0. (12.8)

Para ello observe que

a+b+c
3 abc
3
1 abc; pues a + b + c = 3
abc 1.

En (12.8) factorizando el primer miembro vemos que

(1 abc)(2abc + 1)2 0.

Esta desigualdad es verdadera debido a que abc 1. 2

43. Sean a, b, c, x, y, z n
umeros reales no negativos tales que a + b + c = x + y + z,
pruebe que

ax(a + x) + by(b + y) + cz(c + z) 3(abc + xyz).

 Prueba. La desigualdad que vamos a demostrar es equivalente a:

a2 x + ax2 + b2 y + by 2 + c2 z + cz 2 3(abc + xyz)

aplicando la desigualdad de Cauchy-Schwarz a los n


umeros


a x, b y, c z, yz, zx, xy
186 12. Soluciones

en efecto se tiene:

(a2 x + b2 y + c2 z) (yz + zx + xy) (a xyz + b xyz + c xyz)2
(a2 x + b2 y + c2 z) (yz + zx + xy) xyz(a + b + c)2
(a2 x + b2 y + c2 z) (yz + zx + xy) xyz(x + y + z)2
xyz(3(xy + xz + yz))
a2 x + b2 y + c2 z 3xyz; similarmente
ax2 + by 2 + cz 2 3abc.

Sumando

ax(a + x) + by(b + y) + cz(c + z) 3(abc + xyz).

44. Sean a, b, c n
umeros reales positivos, pruebe que

a2 (b + c a) b2 (c + a b) c2 (a + b c) ab + bc + ca
+ + .
b+c c+a a+b 2

 Prueba. Efectuando en el primer miembro obtenemos una desigualdad


equivalente
 
2 2 2 a3 b3 c3 ab + bc + ca
a +b +c + +
b+c c+a a+b 2
 4 4 4

a b c ab + bc + ca
a2 + b2 + c2 + +
ab + ac bc + ba ac + bc 2

Aplicando el Lema de Titu:

2
a4 b4 c4 (a2 + b2 + c2 )
+ +
ab + ac bc + ba ac + bc 2(ab + ac + bc)
  2
a4 b4 c4 (a2 + b2 + c2 )
+ +
ab + ac bc + ba ac + bc 2(ab + ac + bc)
187

luego basta demostrar que


!
2
2 2 2 (a2 + b2 + c2 ) ab + bc + ca
a +b +c
2(ab + ac + bc) 2

En efecto equivale a:

2
2(ab + ac + bc) (a2 + b2 + c2 ) (a2 + b2 + c2 ) (ab + ac + bc)2

haciendo a2 + b2 + c2 = x, ab + ac + bc = y, tenemos la desigualdad equivalente

2yx x2 y 2

0 x2 2xy + y 2

(x y)2 0.

45. Si a, b, c reales positivos, pruebe que


s s s
3 3
a b c3
+ + 1.
a + (b + c)3
3 b + (c + a)3
3 c3 + (a + b)3

 Prueba. Veamos para x > 0

p (1 + x) + (1 x + x2 ) x2
1 + x3 = (1 + x) (1 x + x2 ) =1+
2 2

x2
1 + x3 1 + .
2
188 12. Soluciones

Como
s v
a3 u 1 1
= u  3 = s
a3 + (b + c)3 ut b+c 
b+c
3
1+ 1+
a a

1 1 a2
 2 =
1 b+c b2 + c2 a2 + b2 + c2
1+ 1+
2 a a2

(hemos aplicado media cuadratica media aritmetica)


luego
s
a3 a2
; en forma similar
a3 + (b + c)3 a2 + b2 + c2
s
b3 b2

b3 + (c + a)3 a2 + b2 + c2
s
c3 c2

c3 + (a + b)3 a2 + b2 + c2

sumando tenemos:
s s s
3 3
a b c3
+ + 1.
a + (b + c)3
3 b + (c + a)3
3 c3 + (a + b)3

La igualdad ocurre si y solo si a = b = c. 2

46. Si a, b, c, d son n
umeros reales no negativos, pruebe que

ab bc cd da
+ + + 0.
a + 2b + c b + 2c + d c + 2d + a d + 2a + b
189

1
 Prueba. Sumando a cada fraccion tenemos la desigualdad equivalente
2
a la anterior
     
ab 1 bc 1 cd 1
+ + + + +
a + 2b + c 2 b + 2c + d 2 c + 2d + a 2
 
da 1
+ + 2
d + 2a + b 2

efectuando

3a + c 3b + d 3c + a 3d + b
+ + + 4
a + 2b + c b + 2c + d c + 2d + a d + 2a + b

para aplicar el Lema de Titu, acondicionemos el numerador, en efecto

(3a + c)2 (3b + d)2 (3c + a)2


+ +
(3a + c)(a + 2b + c) (3b + d)(b + 2c + d) (3c + a)(c + 2d + a)
(3d + b)2
+
(3d + b)(d + 2a + b)
(3a+c+3b+d+3c+a+3d+b)2
(3a+c)(a+2b+c)+(3b+d)(b+2c+d)+(3c+a)(c+2d+a)+(3d+b)(d+2a+b)

16(a + b + c + d)2
= = 4.
4(a + b + c + d)2

47. Si a, b, c son n
umeros reales no negativos, entonces

  
a2 bc b + c + b2 ca c + a + c2 ab a + b 0.

 Prueba. La desigualdad que vamos a demostrar es equivalente a

(b + c) (a2 bc) (c + a) (b2 ca) (a + b) (c2 ab)


+ + 0 (12.9)
b+c c+a a+b

Haciendo m = (b + c) (a2 bc) ; n = (c + a) (b2 ca) ; p = (a + b) (c2 ab).


190 12. Soluciones

Efectuando y sumando obtenemos que m+n+p = 0, luego (12.9) es equivalente


a:

m n p
+ + 0, con m + n + p = 0.
b+c c+a a+b

Sin perdida de generalidad podemos considerar un orden a b c, de donde


se deduce que m 0 y p 0, y reemplazando n = (m + p), tenemos

m (m + p) p
+ 0
b+c c+a a+b
   
1 1 1 1
m +p 0
b+c c+a a+b c+a

1 1 1 1

m
|{z} + (p) 0
b+c c + a | {z } c+a b + a
(+) | {z } (+) | {z }
(+) (+)

48. Dados a, b, c n
umeros reales positivos, pruebe que
 2  2  2
a b c 3
+ + .
a+b b+c c+a 4

 Prueba. La desigualdad es equivalente a:


2 2 2
1 1 1 3
+ +
b 1+
c
1+
a 4
1+ b c
a

b c a
Haciendo = x, = y, = z, tenemos:
a b c
1 1 1 3
2
+ 2
+ 2
, con xyz = 1.
(1 + x) (1 + y) (1 + z) 4
191

Veamos que

1 1 1
2
+ 2
; x, y > 0.
(1 + x) (1 + y) 1 + xy

Aplicando el resultado del problema 21 y aprovechando que xyz = 1, tenemos:

1 1 1 z z
+ = =
(1 + x)2 (1 + y)2 1 + xy z + xyz z+1
1 1 1 z 1
2
+ 2
+ 2
+
(1 + x) (1 + y) (1 + z) z + 1 (1 + z)2
z2 + z + 1
=
(1 + z)2
(z 2 + 2z + 1) z
=
(1 + z)2
z
= 1 2
z + 2z + 1
1
= 1
1
z+ +2
z

acotando el segundo miembro:


1 1
z+ 2z+ +24
z z
1 1
0<
1 4
z+ +2
z
1 1
0>
1 4
z+ +2
z
1 3
1>1
1 4
z+ +2
z

1 1 1 3
2
+ 2
+ 2
.
(1 + x) (1 + y) (1 + z) 4
2
192 12. Soluciones

49. Sean a, b, c, d n
umeros reales positivos que satisfacen

1 1 1 1
+ + + = 1,
1+a 1+b 1+c 1+d

pruebe que abcd 81.

 Prueba. Haciendo

1 1x
= x a =
1+a x
1 1y
= y b =
1+b y
1 1z
= z c =
1+c z
1 1w
= w d =
1+d w
entonces tenemos que

x+y+z+w =1

y demostraremos que
    
1x 1y 1z 1w
81
x y z w
(y + z + w)(x + z + w)(x + y + w)(x + y + z) 81xyzw

como

y + z + w 3 3 yzw

x + z + w 3 3 xzw

x + y + w 3 3 xyw

x + y + z 3 3 xyz
p
(y + z + w)(x + z + w)(x + y + w)(x + y + z) 81 3 (xyzw)3
(y + z + w)(x + z + w)(x + y + w)(x + y + z) 81xyzw.
2
193

50. Sean a, b, c los lados de un triangulo, pruebe que

a b c
+ + 1.
3a b + c 3b c + a 3c a + b

 Prueba.

a b c
S = + +
3a b + c 3b c + a 3c a + b
4a 4b 4c
4S = + +
3a b + c 3b c + a 3c a + b
a+bc b+ca c+ab
4S = 1 + +1+ +1+
3a b + c 3b c + a 3c a + b
(a + b c)2 (b + c a)2
4S = 3 + +
(a + b c)(3a b + c) (b + c a)(3b c + a)
(c + a b)2
+
(c + a b)(3c a + b)

vemos que (a+bc)(3ab+c), (b+ca)(3bc+a), (c+ab)(3ca+b) 0,


pues a, b, c son los lados de un triangulo. Luego, aplicando el Lema de Titu,
tenemos

(a+bc+b+ca+c+ab)2
4S 3 + (a+bc)(3ab+c)+(b+ca)(3bc+a)+(c+ab)(3ca+b)

(a + b + c)2 (a + b + c)2
= 3+ = 3 +
a2 + b2 + c2 + 2(ab + ac + bc) (a + b + c)2

= 3.

Luego

4S 4

S 1.

2
194 12. Soluciones

51. Dados a, b, c, d n
umeros reales positivos, pruebe que
21
a a 16

3
21 21 21 21 .
3
a + 63bcd a 16 + b 16 + c 16 + d 16

 Prueba. Como

(x + y + z + t)3 = x3 + y 3 + z 3 + t3 + 3x2 (y + z + t)
+3y 2 (x + z + t) + 3z 2 (x + y + t)
+3t2 (x + y + z) + 6(xyz + xyt + xzt + yzt)

luego

(x + y + z + t)3 = x3 + y 3 + z 3 + t3 + 3x2 y + 3x2 z + 3x2 t


+3y 2 x + 3y 2 z + 3y 2 t + 3z 2 x + 3z 2 y + 3z 2 t
+3t2 x + 3t2 y + 3t2 z + 6xyz + 6xyt + 6xzt + 6yzt
p
x3 + 63 63 x45 y 48 z 48 t48
15 16 16 16
= x3 + 63x 21 y 21 z 21 t 21
q
3 15 16 16 16
(x + y + z + t) x3 + 63x 21 y 21 z 21 t 21 luego
r  
5 16 16 16 16
= 3 x 7 x 7 + 63y 21 z 21 t 21
r 
5 16 3 16 16 16
3
= x 21 x 21 + 63y 21 z 21 t 21

16 16 16 16
Haciendo x 21 = a, y 21 = b, z 21 = c, t 21 = d.
Tenemos
21 21 21 21 5
3
a 16 + b 16 + c 16 + d 16 a 16 a3 + 63bcd

de donde
21
a a 16

3
21 21 21 21 .
3
a + 63bcd a 16 + b 16 + c 16 + d 16
2
195

52. Dados a, b, c, d n
umeros reales positivos, pruebe que

a b c d
A=
3
+
3
+
3
+
3
1.
a3 + 63bcd b3 + 63acd c3 + 63bda d3 + 63abc

 Prueba. Del problema 51 tenemos que

21
a a 16

3
21 21 21 21
a3 + 63bcd a 16 + b 16 + c 16 + d 16
21
b b 16

3 3
21 21 21 21
b + 63acd a 16 + b 16 + c 16 + d 16
21
c c 16

3 3
21 21 21 21
c + 63adb a 16 + b 16 + c 16 + d 16
21
d d 16

3
21 21 21 21
3
d + 63abc a 16 + b 16 + c 16 + d 16

sumando obtenemos

A 1.

53. Sean a, b, c los lados de un triangulo, pruebe que


r
3 a3 + b3 + c3 + 3abc
max {a, b, c}.
2

 Prueba. Sin perdida de generalidad, sea a = max {a, b, c}, entonces debe-
mos probar
r
3 a3 + b3 + c3 + 3abc
a
2
196 12. Soluciones

En efecto tenemos

a3 + b3 + c3 + 3abc 2a3
b3 + c3 a3 + 3abc 0
b3 + c3 + (a)3 + 3abc 0, (Identidad de Gauss)
(b + c a)(b2 + c2 + a2 + bc ab ac) 0
1
(b + c a)(2b2 + 2c2 + 2a2 + 2bc 2ab 2ac) 0
2
1
c a})((a b)2 + (a c)2 + (b c)2 ) 0
(b| + {z
2
(+)

pues a, b, c son los lados de un triangulo, por lo tanto esta u


ltima desigualdad
es verdadera. 2

54. Sean a, b, c n
umeros reales positivos, pruebe que

(a + b)2 (b + c)2 (c + a)2


S= + + > 2.
a2 + b2 + c2 + ab a2 + b2 + c2 + bc a2 + b2 + c2 + ca

 Prueba. Vemos que

(a + b)2 (a + b)2
>
a2 + b2 + c2 + ab a2 + b2 + c2 + ab + ac + bc
2
(b + c) (b + c)2
>
a2 + b2 + c2 + bc a2 + b2 + c2 + ab + ac + bc
2
(c + a) (c + a)2
>
a2 + b2 + c2 + ca a2 + b2 + c2 + ab + ac + bc

sumando miembro a miembro se tiene:

2 (a2 + b2 + c2 + ab + ac + bc)
S> = 2.
a2 + b2 + c2 + ab + ac + bc
2
197

umeros reales tales que a, b, c [0; 1], pruebe que


55. Dados a, b, c n

a b c
+ + + (1 a)(1 b)(1 c) 1.
b+c+1 c+a+1 a+b+1

 Prueba. Por la simetra, sin perdida de generalidad podemos tomar a


b c. Primero veamos el caso cuando a 6= 1. Luego, como (1a), (1b), (1c)
umeros positivos, aplicamos MA MG, de la siguiente manera
son n

(b + c + 1) + (1 b) + (1 c) p
3 (b + c + 1)(1 b)(1 c)
3
(b + c + 1)(1 b)(1 c) 1
1
(1 b)(1 c)
b+c+1
1a
(1 a)(1 b)(1 c)
b+c+1
a 1
+ (1 a)(1 b)(1 c) , y ademas
b+c+1 b+c+1
b b

c+a+1 c+b+1
c c

a+b+1 c+b+1

sumando miembro a miembro

a b c
+ + + (1 a)(1 b)(1 c) 1.
b+c+1 c+a+1 a+b+1

En este caso la igualdad ocurre si y solo si a = b = c = 0.


Ahora veamos el caso cuando a = 1. Se tiene

a 1
= ,
b+c+1 b+c+1
b b
,
c+2 c+b+1
c c
,
b+2 c+b+1
198 12. Soluciones

sumando miembro a miembro, obtenemos

a b c
+ + 1.
b+c+1 c+a+1 a+b+1

En este caso la igualdad ocurre si y solo si (a, b, c) = (1, 1, 1), (a, b, c) = (1, 1, 0),
(a, b, c) = (1, 0, 0), o sus respectivas permutaciones. 2

56. Sean a, b, c n
umeros reales positivos, pruebe que

a b c 3
+ + .
2a + b + c 2b + c + a 2c + a + b 4

 Prueba. La desigualdad a demostrar es equivalente a


 
a+b+c a+b+c a+b+c 3
+ + 3
2a + b + c 2b + c + a 2c + a + b 4
a+b+c a+b+c a+b+c 9
+ +
2a + b + c 2b + c + a 2c + a + b 4
1 1 1
4(a + b + c) + + 9.
2a + b + c 2b + c + a 2c + a + b

Haciendo 2a + b + c = x, 2b + c + a = y, 2c + a + b = z, entonces 4(a + b + c) =


x + y + z, luego tenemos que la desigualdad anterior es equivalente a:
 
1 1 1
(x + y + z) + + 9,
x y z

pero esta desigualdad es verdadera. 2

umeros reales tales que a2 + b2 + c2 = 1, pruebe que


57. Sean a, b, c n


1 ab + 1 bc + 1 ca 6. (12.10)
199

 Prueba. Elevando al cuadrado tenemos el equivalente de la desigualdad


(12.10)

(1 ab) + (1 bc) + (1 ca)


hp p p i
+2 (1 ab)(1 bc) + (1 ab)(1 ca) + (1 bc)(1 ca) 6

usando el dato a2 + b2 + c2 = 1;

1 2
1 ab = [a + b2 + c2 + 1] ab
2
1 2 1
= (c + 1) + (a2 + b2 2ab)
2 2
1 2 1 1
= (c + 1) + (a b)2 (c2 + 1)
2 2 2
1 2
1 ab (c + 1) ; similarmente
2
1 2
1 bc (a + 1)
2
1 2
1 ca (b + 1)
2

multiplicando dos a dos tenemos:

1 2 1
(1 ab)(1 bc) (a + 1) (c2 + 1) (ac + 1)2
4 4
p 1
(1 ab)(1 bc) (ac + 1); similarmente
2
p 1
(1 ab)(1 ac) (bc + 1)
2
p 1
(1 bc)(1 ac) (ab + 1)
2
hp p p i
2 (1 ab)(1 bc) + (1 ab)(1 ca) + (1 bc)(1 ca)
ab + bc + ca + 3
200 12. Soluciones

(1 ab) + (1 bc) + (1 ca)


hp p p i
+2 (1 ab)(1 bc) + (1 ab)(1 ca) + (1 bc)(1 ca) 6.

58. Pruebe que

a4 + b4 + c4 + abc(a + b + c) ab(a2 + b2 ) + bc(b2 + c2 ) + ca(c2 + a2 ), (12.11)

se cumple para todo a, b, c n


umeros reales.

 Prueba. Sin perdida de generalidad, podemos suponer a b c, por la


simetra de las expresiones.
La desigualdad (12.11) es equivalente a:

a4 + b4 + c4 + abc(a + b + c) ab(a2 + b2 ) bc(b2 + c2 ) ca(c2 + a2 ) 0

a4 a3 (b + c) + a2 bc b3 (c + a) + b2 ca c3 (a + b) + c2 ab 0

a2 (a2 a(b + c) + bc) + b2 (b2 b(c + a) + bc) + c2 (c2 c(a + b) + ab) 0

a2 (a b)(a c) + b2 (b a)(b c) + c2 (c a)(c b) 0,

esta desigualdad es verdadera, pues es la desigualdad de Sch


ur. 2

59. Pruebe que

a3 + b3 + c3 + 4(a + b + c) + 9abc 8(ab + bc + ca),

se cumple para todo a, b, c n


umeros reales no negativos.
201

(ab + bc + ca)2
 Prueba. Aplicando MA MG a los n
umeros (a+b+c), ,
a+b+c
tenemos

(ab + bc + ca)2 s  
(a + b + c) + (ab + bc + ca)2
a+b+c (a + b + c)
2 a+b+c
4(ab + bc + ca)2
4(a + b + c) + 8(ab + bc + ca).
a+b+c

Luego basta demostrar que

4(ab + bc + ca)2
a3 + b3 + c3 + 4(a + b + c) + 9abc 4(a + b + c) +
a+b+c
4(ab + bc + ca)2
a3 + b3 + c3 + 9abc
a+b+c
a + ab + ac + ba + b + bc3 + ca3 + cb3 + c4 + 9abc(a + b + c)
4 3 3 3 4

4 (a2 b2 + b2 c2 + c2 a2 + 2abc(a + b + c))


a4 + b4 + c4 + abc(a + b + c) + ab(a2 + b2 ) + ac(a2 + c2 ) + bc(b2 + c2 )
4 (a2 b2 + b2 c2 + c2 a2 )

Por el problema anterior (problema 58), sabemos que

a4 + b4 + c4 + abc(a + b + c) ab(a2 + b2 ) + ac(a2 + c2 ) + bc(b2 + c2 )


a4 + b4 + c4 + abc(a + b + c) + ab(a2 + b2 ) + ac(a2 + c2 ) + bc(b2 + c2 )
2 [ab(a2 + b2 ) + ac(a2 + c2 ) + bc(b2 + c2 )] 4 [a2 b2 + b2 c2 + c2 a2 ]

La igualdad ocurre si y solo si

(a, b, c) = (2, 2, 2) (a, b, c) = (2, 2, 0) o sus respectivas permutaciones.

2
202 12. Soluciones

60. Pruebe que la desigualdad



p p p 3 2
M= a2 + (1 b)2 + b2 + (1 c)2 + c2 + (1 a)2 ,
2

se cumple para todo a, b, c n


umeros reales.

 Prueba. Aplicando la desigualdad de Minskowsky.


p p p
M = a2 + (1 b)2 + b2 + (1 c)2 + c2 + (1 a)2
p
(a + b + c)2 + (1 b + 1 c + 1 a)2
p
= (a + b + c)2 + (3 (a + b + c))2

Haciendo a + b + c = x.
Entonces
p
M x2 + (3 x)2 = 2x2 6x + 9
s  
2
9 9
= 2 x 3x + +
4 2
s  2
3 9
= 2 x +
2 2
r
9 3 2
=
2 2

3 2
M .
2
2

61. Pruebe que


s s s s
3 3 3
a b c d3
3
+ 3
+ 3
+ 1,
a + 15bcd b + 15acd c + 15abd d3 + 15abc

se cumple para todo a, b, c n


umeros reales positivos.
203

 Prueba. Efectuando

(m + n + p + q)2
= m2 + n2 + p2 + q 2 + 2(mn + mp + mq + np + nq + pq)
= m2 + n2 + p2 + q 2 + mn + mn + mp + mp + + pq + pq)
| {z }
p
m2 + 15 15 m6 n8 p8 q 8
= m2 + 15m6/15 n8/15 p8/15 q 8/15
m+n+p+q
p
m2 + 15m6/15 n8/15 p8/15 q 8/15
r  24 
6
= m 15 m + 15n
15 8/15 p 8/15 q 8/15

r 
1 8 3
= m5 m 15 + 15n8/15 p8/15 q 8/15

m+n+p+q
r 3
1 8
m5 m 15 + 15n8/15 p8/15 q 8/15

Haciendo

m8/15 = a; p8/15 = c
n8/15 = b; q 8/15 = d.

Tenemos:


a15/8 + b15/8 + c15/8 + d15/8 a3/8 a3 + 15bcd
a3/2 a15/8
15/8
a3 + 15bcd a + b15/8 + c15/8 + d15/8
204 12. Soluciones

r
a3 a15/8
; similarmente
a3 + 15bcd a15/8 + b15/8 + c15/8 + d15/8
r
b3 b15/8

b3 + 15acd a15/8 + b15/8 + c15/8 + d15/8
r
c3 c15/8

c3 + 15abd a15/8 + b15/8 + c15/8 + d15/8
r
d3 d15/8

d3 + 15abc a15/8 + b15/8 + c15/8 + d15/8

sumando miembro a miembro:


s s s s
a3 b3 c3 d3
+ + + 1,
a3 + 15bcd b3 + 15acd c3 + 15abd d3 + 15abc
2

62. Sean a, b, c n
umeros reales no negativos tales que ab + bc + ca = 3, pruebe que

1 1 1
+ 2 + 2 1.
a2 +2 b +2 c +2

 Prueba. La desigualdad que demostraremos es equivalente a:

(b2 + 2) (c2 + 2) + (a2 + 2) (c2 + 2) + (a2 + 2) (b2 + 2)


(a2 + 2) (b2 + 2) (c2 + 2)
a2 b2 c2 + a2 b2 + a2 c2 + b2 c2 4 0

Haciendo ab = x, ac = y, bc = z, tenemos que demostrar

xyz + x2 + y 2 + z 2 4 0, sabiendo que x + y + z = 3


x2 + y 2 + 2yz + z 2 + xyz 2yz 4 0
x2 + (y + z)2 + yz(x 2) 4 0
 
2 2 2x
x + (y + z) 4yz 40
4
205

Como:

   
2 2 2x 2 2 2 2x
x + (3 x) 4yz 4 x + (3 x) (y + z) 4
4 4
 
2 2 2 2x
= x + (3 x) (3 x) 4
4

basta demostrar que


 
2x
x2 + (3 x)2 (3 x)2 4 0
4
 
2 2 x2
x + (x 3) + (x 3)2 4 0
4
x3 3x + 2 0
(x 1)2 (x + 2) 0

La igualdad ocurre si y solo si a = b = c = 1. 2

63. Sean a, b, c n
umeros reales positivos, pruebe que

a3 b3 c3 3 2  1
+ + a + b2 + c2 (ab + bc + ca).
a+b b+c c+a 4 4

 Prueba. Aplicando MA MG:

a3 a(a + b) r
+ a3 a(a + b)
a+b 4
2 a+b 4
a3 a2 + ab
+ a2
a+b 4
a3 3 2 1
a ab; similarmente
a+b 4 4
b3 3 2 1
b bc
b+c 4 4
c3 3 2 1
c ca
c+a 4 4
206 12. Soluciones

sumando tenemos:

a3 b3 c3 3 2  1
+ + a + b2 + c2 (ab + bc + ca).
a+b b+c c+a 4 4
2

64. Sean a, b, c n
umeros reales positivos, pruebe que

ab bc ca 1
S= + + (a + b + c).
2c + a + b 2a + b + c 2b + c + a 4

 Prueba. Sean m, n R+ , entonces

4mn (m + n)2
4 m+n

m+n mn
4 1 1
+
m+n m n

Aplicando dicho resultado en el ejercicio tenemos:


4 1 1
+
(c + a) + (c + b) c+a c+b
4ab ab ab
+ ; similarmente
(c + a) + (c + b) c+a c+b
4bc bc bc
+
(a + b) + (a + c) a+b a+c
4ca ca ca
+
(b + c) + (b + a) b+c b+a

sumando miembro a miembro:


b(a + c) a(c + b) c(a + b)
4S + + =a+b+c
a+c c+b a+b
4S a + b + c

1
S (a + b + c).
4
2
207

65. Si a b c d e, entonces

(a + b + c + d + e)2 8(ac + bd + ce).

Cuando ocurre la igualdad?

 Prueba. Veamos

(a + b + c + d + e)2 8(ac + bd + ce)


= (a + b + c + d + e)2 8c(a + b + c + d + e) + 16c2 +8c(a + b + c + d + e)
| {z }
8(ac + bd + ce) 16c2
= (a + b + c + d + e 4c)2 + 8c(a + b + c + d + e) 8(ac + bd + ce) 16c2
= (a + b + d + e 3c)2 + 8(bc c2 + cd bd)
= (a + b + d + e 3c)2 + 8[b(c d) + c(d c)]
= (a + b + d + e 3c)2 + 8(c d)(b c) 0

pues c d b c.
La igualdad ocurre si

a+d+e a+b+e
b=c= c=d= .
2 2
2

umeros reales positivos tales que xyz xy + yz + zx.


66. Sean x, y, z n
Pruebe que:

xyz 3(x + y + z).

 Prueba. De xyz xy + yz + zx tenemos:

1 1 1
1 + +
z x y
208 12. Soluciones

1 1 1
Haciendo = a, = b, = c
x y z

a + b + c 1,

en seguida basta probar que


 
1 1 1 1
3 + +
abc a b c
1 3(bc + ac + ab)

como

1 a + b + c 1 (a + b + c)2
1 a2 + b2 + c2 + 2(ab + bc + ca) 3(ab + bc + ca)
1 3(ab + bc + ca).
2

67. Sean x, y n
umeros reales positivos tales que x + y = 1, pruebe que

x y 2
+p .
1 x2 1 y2 3

 Prueba. Del Lema de Titu, se tiene que

x y x2 y2 (x + y)2
+p = + p p
1 x2 1 y2 x 1 x2 y 1 y 2 x 1 x2 + y 1 y 2
1
= p
x 1 x2 + y 1 y 2

luego es suficiente probar que


1 2 p 3
p x 1x +y 1y
2 2
x 1 x2 + y 1 y 2 3 2

p 3
x x x3 + y y y 3
2
209

aplicando Cauchy-Schwarz

 p 2  2   2 p 2

3 3 2 3 3
x xx + y yy x + y xx + yy
= (x + y) (x + y (x3 + y 3 ))
= 1 (x3 + y 3 )

Como x + y = 1, aplicando la Media Potencial, tenemos:


1
x3 + y 3
4
1
(x3 + y 3 )
4
3
1 (x3 + y 3 )
4
entonces
 p 2 3
x x x3 + y y y 3
4
de donde

p 3
x 1 x2 + y 1 y 2 .
2
2

68. Sean a1 , a2 , . . . , an , b1 , b2 , . . . , bn n
umeros reales positivos, tales que a1 + a2 +
+ an = b1 + b2 + + bn . Pruebe que

a21 a22 a2n 1


+ ++ (a1 + a2 + + an ).
a1 + b1 a2 + b2 an + bn 2

 Prueba. Aplicando el Lema de Titu, tenemos

a21 a22 a2n


+ ++
a1 + b1 a2 + b2 an + bn
(a1 + a2 + + an )2

(a1 + a2 + + an ) + (b1 + b2 + + bn )
210 12. Soluciones

a21 a22 a2n (a1 + a2 + + an )2


+ ++
a1 + b1 a2 + b2 an + bn 2(a1 + a2 + + an )
1
= (a1 + a2 + + an )
2

de donde

a21 a22 a2n 1


+ + + (a1 + a2 + + an ).
a1 + b1 a2 + b2 an + bn 2
2

69. Sean a, b, c n
umeros reales positivos, pruebe que

2ab 2bc 2ca


+ + a + b + c.
a+b b+c c+a

 Prueba. Aplicando MH MA, tenemos:

2ab a+b

a+b 2
2bc b+c

b+c 2
2ca c+a
,
c+a 2

sumando miembro a miembro, tenemos

2ab 2bc 2ca


+ + a + b + c.
a+b b+c c+a
2

70. (Rusia 1992). Sean x, y, z n


umeros reales positivos, pruebe que


x4 + y 4 + z 2 8xyz.
211

z2 z2
 Prueba. umeros x4 , y 4,
Aplicando MA MG a los n ,
2 2
r
4 z2 z2
4 4
4 4
z2 z2
x +y + + 4 x y
2 2 2 2
xyz 4
x4 + y 4 + z 2 4 4
= xyz
4 2

x4 + y 4 + z 2 8xyz.

71. Sean a, b, c n
umeros reales positivos con a + b + c = 1, pruebe que
   
1 1 1
+1 +1 +1 64.
a b c

 Prueba. Efectuando el primer miembro


   
1 1 1 1 1 1 1 1 1 1
1+ 1+ 1+ =1+ + + + + + +
a b c a b c ab ac bc abc

como
r
1 1 1 1 3
+ + 33 =
a b c abc 3
abc
r
1 1 1 1 3
+ + 33 2 2 2 = 2
ab ac bc abc 3
abc
 3
1 1
=
abc 3
abc

luego

     3  3
1 1 1 3 3 1 1
1+ 1+ 1+ 1+ + 2+ = 1+
a b c 3
abc 3 abc 3
abc 3
abc
212 12. Soluciones

De la hipotesis a + b + c = 1, tenemos que

1 3 1
abc 3
3 3
abc
1
1+ 3
4
abc
 3
1
1+ 3
64
abc

de donde
   
1 1 1
+1 +1 +1 64.
a b c
2

72. Sean a, b, c n
umeros reales, y

x= b2 bc + c2 ; y = c2 ca + a2 ; z = a2 ab + b2

pruebe que

xy + yz + zx a2 + b2 + c2 .

 Prueba. Escribiendo convenientemente:


r
c 2 3c2
x= b2 bc + c2 = + b
2 4
r 
c 2 3c2
y = c2 ca + a2 = a +
2 4

entonces, aplicando Cauchy-Schwarz


   
2 2 c 2 3c2 c 2 3c2
xy = b + a + ;
2 4 2 4
 2
c c  3c2
b a +
2 2 4
213

ac bc
xy ab + c2 ; similarmente
2 2
ab bc
xz ac + b2
2 2
ab ac
yz bc + a2
2 2
sumando miembro a miembro, se tiene

xy + yz + zx a2 + b2 + c2 .

La igualdad ocurre si y solo si a = b = c. 2

73. Sean x, y, z n
umeros reales positivos tales que x + y + z = 3, pruebe que

(1 x)2 (1 y)2 (1 z)2


+ + 0.
1 x4 1 y4 1 z4

 Prueba. La desigualdad es equivalente a:

1x 1y 1z
2 3
+ 2 3
+ 0
1+x+x +x 1+y+y +y 1 + z + z2 + z3

Podemos tomar un orden x y z, entonces 1 x 1 y 1 z y

1 1 1

1 + x + x2 + x3 1 + y + y2 + y3 1 + z + z2 + z3

esto significa que podemos aplicar la desigualdad de Chebyshev

1x 1y 1z
2 3
+ 2 3
+
1+x+x +x 1+y+y +y 1 + z + z2 + z3
 
1 1 1
(1 x + 1 y + 1 z) + +
1 + x + x2 + x3 1 + y + y 2 + y 3 1 + z + z 2 + z 3
 
1 1 1
= (3 (x + y + z)) + +
| {z } 1 + x + x2 + x3 1 + y + y 2 + y 3 1 + z + z 2 + z 3
3

= 0.
214 12. Soluciones

1x 1y 1z
2 3
+ 2 3
+ 0.
1+x+x +x 1+y+y +y 1 + z + z2 + z3
2

74. Si a1 , a2 , . . . , an son n
umeros reales, pruebe que

n X
X n
i j cos (ai aj ) 0.
i=1 j=1

 Prueba.
n X
X n
i j (cos ai cos aj + sen ai sen aj )
i=1 j=1
n X
X n
= (i j cos ai cos aj + i j sen ai sen aj )
i=1 j=1

X n
n X n X
X n
= ij cos ai cos aj + ij sen ai sen aj
i=1 j=1 i=1 j=1
n
X n
X n
X n
X
= i cos ai j cos aj + i sen ai j sen aj
i=1 j=1 i=1 j=1
| {z } | {z }
n
X Xn n
X Xn
= i cos ai i cos ai + i sen ai i sen ai
i=1 i=1 i=1 i=1
n
!2 n
!2
X X
= i cos ai + i sen ai 0.
i=1 i=1
2

75. Sean a, b, c n
umeros reales, tales que

x= b2 + bc + c2 ; y = c2 + ca + a2 ; z = a2 + ab + b2

pruebe que

xy + yz + zx (a + b + c)2 .
215

 Prueba. Utilizando la idea del problema 72 tenemos que

ac bc
xy ab + c2 + +
2 2
ab bc
xz ac + b2 + +
2 2
ab ac
yz bc + a2 + +
2 2

sumando miembro a miembro, tenemos

xy + yz + zx a2 + b2 + c2 + 2(ab + ac + bc)

xy + yz + zx (a + b + c)2 .

76. Sean a, b, c, d, e, f n
umeros reales positivos, pruebe que

a b c d e f
+ + + + + 3.
b+c c+d d+e e+f f +a a+b

 Prueba. Para aplicar el Lema de Titu, el primer miembro de la desigual-


dad se expresa de la siguiente manera

a b c d e f
S = + + + + +
b+c c+d d+e e+f f +a a+b
a2 b2 c2 d2 e2 f2
= + + + + +
ab + ac bc + bd cd + ce ed + f d ef + ea af + bf
(a + b + c + d + e + f )2

ab + ac + bc + bd + cd + ce + ed + f d + ef + ea + af + bf
(a + b + c + d + e + f )2
=
M
216 12. Soluciones

donde

M = ab + ac + bc + bd + cd + ce + ed + f d + ef + ea + af + bf
2M = (a + b + c + d + e + f )2 (a + d)2 (b + e)2 (c + f )2 (12.12)

como aparece (a + d)2 + (b + e)2 + (c + f )2, entonces aplicando Cauchy-Schwarz

 
(a + d)2 + (b + e)2 + (c + f )2 12 + 12 + 12 (a + b + c + d + e + f )2


3 (a + d)2 + (b + e)2 + (c + f )2 (a + b + c + d + e + f )2 (12.13)

De (12.12) tenemos

(a + d)2 + (b + e)2 + (c + f )2 = (a + b + c + d + e + f )2 2M
3 [(a + d)2 + (b + e)2 + (c + f )2 ] = 3(a + b + c + d + e + f )2 6M

luego de (12.13) se tiene que:

3(a + b + c + d + e + f )2 6M (a + b + c + d + e + f )2
2(a + b + c + d + e + f )2 6M
(a + b + c + d + e + f )2
3.
M

a b c d e f
+ + + + + 3.
b+c c+d d+e e+f f +a a+b
2

77. Sean a, b, c, x, y, z, t, v, u n
umeros reales positivos, entonces

  
a3 + b3 + c3 x3 + y 3 + z 3 t3 + u3 + v 3 (axt + byu + czv)3 .
217

 Prueba. Observemos que

a3 b3 c3
+ + +
a3 + b3 + c3 a3 + b3 + c3 a3 + b3 + c3
x3 y3 z3
+ + +
x3 + y 3 + z 3 x3 + y 3 + z 3 x3 + y 3 + z 3
t3 u3 v3
+ + =3
t3 + u3 + v 3 t3 + u3 + v 3 t3 + u3 + v 3

luego, aplicando MA MG, tenemos que

a3 x3 t3
+ +
a3 + b3 + c3 x3 + y 3 + z 3 t3 + u3 + v 3
3axt
p ,
3
(a3 + b3 + c3 ) (x3 + y 3 + z 3 ) (t3 + u3 + v 3 )
b3 y3 u3
+ +
a3 + b3 + c3 x3 + y 3 + z 3 t3 + u3 + v 3
3byu
p ,
3
(a3 + b3 + c3 ) (x3 + y 3 + z 3 ) (t3 + u3 + v 3 )
c3 z3 v3
+ +
a3 + b3 + c3 x3 + y 3 + z 3 t3 + u3 + v 3
3czv
p ,
3
(a + b + c ) (x + y 3 + z 3 ) (t3 + u3 + v 3 )
3 3 3 3

sumando miembro a miembro, tenemos

3axt + 3byu + 3czv


3 p
3
(a3 + b3 + c3 ) (x3 + y 3 + z 3 ) (t3 + u3 + v 3 )
p
3
(a3 + b3 + c3 ) (x3 + y 3 + z 3 ) (t3 + u3 + v 3 ) axt + byu + czv
(a3 + b3 + c3 ) (x3 + y 3 + z 3 ) (t3 + u3 + v 3 ) (axt + byu + czv)3 .

A continuacion veremos como una aplicacion del problema 77 un problema


planteado en la IMO 2001.
218 12. Soluciones

78. (IMO 2001). Pruebe para a, b, c n


umeros reales positivos

a b c
+ + 1.
a2 + 8bc b2 + 8ca c2 + 8ab

 Prueba. Aplicando el problema anterior:


 
a b c
+ +
a2 + 8bc b2 + 8ca c2 + 8ab
 
a b c
+ +
a2 + 8bc b2 + 8ca c2 + 8ab
[a (a2 + 8bc) + b (b2 + 8ca) + c (c2 + 8ca)] (a + b + c)3
 2
a b c
+ +
a2 + 8bc b2 + 8ca c2 + 8ab
(a (a2 + 8bc) + b (b2 + 8ca) + c (c2 + 8ca)) (a + b + c)3

basta demostrar que

(a + b + c)3 a (a2 + 8bc) + b (b2 + 8ca) + c (c2 + 8ca)


(a + b + c)3 a3 + b3 + c3 + 24abc
a3 + b3 + c3 + 3(a + b)(a + c)(b + c) a3 + b3 + c3 + 24abc
(a + b)(a + c)(b + c) 8abc

pero esta u
ltima desigualdad es verdadera.

a b c
+ + 1.
a2 + 8bc b2 + 8ca c2 + 8ab
2

1 1 1
79. (APMO 2002). Sean a, b y c n
umeros reales positivos que satisfacen + + =
a b c
1, pruebe que


a + bc + b + ca + c + ab abc + a+ b+ c.
219

 Prueba. Como

1 1 1
+ + =1
a b c
 
1 1 1
abc + + = abc
a b c
r r r
bc ac ab
+ + = abc.
a b c

Luego la desigualdad es equivalente a:

rr r
bc ac ab
a + bc + b + ca + c + ab + + + a+ b+ c
a b c
r ! r 
bc ac
a + bc + b + ca + c + ab + a + + b
a b
r !
ab
+ + c
c

Veamos que es suficiente probar que


r
bc
a + bc + a; elevando al cuadrado
a
bc
a + bc + 2 bc + a
a
 
1 1 1 1
bc bc + 2 bc; = 1
a a b c
 
1 1
bc bc 1 + 2 bc
b c

bc bc (c + b) + 2 bc

b+c 2 bc
 2
b c 0

2
220 12. Soluciones

80. Sean a, b, c n
umeros reales positivos que satisfacen abc = 1, pruebe que

3 6
1+ .
a+b+c ab + ac + bc

1 1 1
 Prueba. Haciendo = x, = y, = z xyz = 1.
a b c
La desigualdad es equivalente a:

3 6
1+
1 1 1 1 1 1
+ + + +
x y z xy xz yz
3 6
1+ (12.14)
xy + xz + yz x+y+z

Pero

(x + y + z)2 3(xy + xz + yz)


3 9

xy + xz + yz (x + y + z)2
3 9
1+ 1+ (12.15)
xy + xz + yz (x + y + z)2

De (12.14) y (12.15) basta demostrar

9 6
1+ 2

(x + y + z) x+y+z
 2
3
1 0.
x+y+z
2

81. Sean a, b, c, x, y, z n
umeros reales positivos tales que x + a = y + b = z + c = 1,
pruebe que
 
1 1 1
(abc + xyz) + + 3.
ay bz cx
221

 Prueba. De x+a = y +b = z +c = 1 se tiene que x = 1a, y = 1b, z =


1 c entonces

abc + xyz = abc + (1 a)(1 b)(1 c)


= abc + 1 (a + b + c) + (ab + ac + bc) abc
= 1 (b + c) + bc + ab + ac a
= (1 b)(1 c) + ab + ac a
= (1 b)(1 c) + a(b 1) + ac
abc + xyz (1 b)(1 c) ac
= 1+
a(1 b) a(1 b) a(1 b)
abc + xyz 1c c
= + 1; similarmente
ay a 1b
abc + xyz 1a a
= + 1
bz b 1c
abc + xyz 1b b
= + 1
cx c 1a

sumando miembro a miembro:

 
1 1 1 1c c 1a a 1b
(abc + xyz) + + = + + + +
ay bz cx a 1b b 1c c
b
+ 3
1a
   
1c a c 1b
= + + +
a 1c 1b c
 
1a b
+ + 3
b 1a
2 + 2 + 2 3 = 3.
2

82. Sean x, y, z n
umeros reales positivos, pruebe que

x+y+z xy + yz + zx
p p .
3 3 x2 + xy + y 2 + y 2 + yz + z 2 + z 2 + zx + x2
222 12. Soluciones

 Prueba. Como

x2 + y 2 2xy 4x2 + 4y 2 + 4xy 3x2 + 3y 2 + 6xy


4 (x2 + xy + y 2) 3(x + y)2
3
x2 + xy + y 2 (x + y)2
4

p 3
x2 + xy + y 2 (x + y); similarmente
2

p 3
y 2 + yz + z 2 (y + z)
2

3
z 2 + zx + x2 (z + x)
2

sumando miembro a miembro, tenemos

p p
x2 + xy + y 2 + y 2 + yz + z 2 + z 2 + zx + x2

3
(x + y + y + z + z + x)
2
= 3(x + y + z)
1
p p
x + xy + y + y + yz + z 2 + z 2 + zx + x2
2 2 2

1

3(x + y + z)
xy + yz + zx
p p
x + xy + y + y 2 + yz + z 2 + z 2 + zx + x2
2 2

xy + yz + zx
(12.16)
3(x + y + z)

Pero

x2 + y 2 + z 2 xy + xz + yz
(x + y + z)2 3(xy + xz + yz)
x+y+z xy + xz + yz
(12.17)
3 3 3(x + y + z)
223

Por transitividad de (12.16) y (12.17)

xy + yz + zx x+y+z
p p .
x2 + xy + y2 2 2 2
+ y + yz + z + z + zx + x2 3 3
2

umeros reales tales que x2 + y 2 1 y z 2 + t2 1, pruebe que


83. Sean x, y, z, t n
p p
(x + z)2 + (y + t)2 + (x z)2 + (y t)2 2 2.

 Prueba. Como sabemos (a + b)2 2(a2 + b2 )


p p 2
2 2 2
(x + z) + (y + t) + (x z) + (y t) 2

2 ((x + z)2 + (y + t)2 + (x z)2 + (y t)2 )


p p
(x + z)2 + (y + t)2 + (x z)2 + (y t)2
hp i
2 2 2 2
2 (x + z + y + t ) 2

p  
2 2(1 + 1)

= 2 2
p p
(x + z)2 + (y + t)2 + (x z)2 + (y t)2 2 2.
2

umeros reales tales que a2 +b2 +c2 8, pruebe que ab+bc+2ca


84. Dados a, b, c n
8.

 Prueba. Como

8 a2 + b2 + c2
8 + ab + bc + 2ca a2 + b2 + c2 + ab + bc + 2ca
b2 3b2
= a2 + + c2 + ab + bc + 2ca +
| 4 {z } 4
 2
b 3b2
= a+ +c + 0
2 4
224 12. Soluciones

La igualdad ocurre cuando

b
a+ + c = 0 b = 0 a2 + b2 + c2 = 8,
2

de donde

(a, b, c) = (2, 0, 2) (a, b, c) = (2, 0, 2).

85. Dados a, b, c n
umeros reales no negativos tales que a + b + c = 1, pruebe que
4 4 4 256
a b+b c+c a .
3125

 Prueba. Sin perdida de generalidad, supongamos que a = max {a, b, c},


entonces

b4 c a3 bc y c4 a c2 a3 ca4

3c c
y desde que , tenemos:
4 2

a4 b + b4 c + c4 a a4 b + a3 bc + c4 a
4 3 c4 a c4 a
= a b + a bc + +
2 2
4 2 3
ca c a
a4 b + a3 bc + +
2 2
3
ac
= a3 b(a + c) + (a + c)
2
  
3 c 3 3c
= a (a + c) b + a (a + c) b +
2 4
225

 
3c
a4 b + b4 c + c4 a a3 (a + c) b +
4
  
4 a a a a+c 3c
= 4 b+
4 4 4 4 4

a a a a+c 3c 5
+ + + +b+
4
44 4 4 4 4
5
 5  5
4 a+b+c 41 256
= 4 =4 = .
5 5 3125
2

86. Si a, b, y c son los lados de un triangulo, pruebe que

a b c
+ + 3.
b+ca c+ab a+bc

 Prueba. Haciendo b + c a = x, c + a b = y, a + b c = z, tenemos


y+z x+z x+y
que a = ,b= ,c= .
2 2 2
Reemplazando, la desigualdad es equivalente a:

y+z x+z x+y


+ + 3
2x 2y 2z
y+z x+z x+y
+ + 6
x y z
    
y x z x z y
+ + + + + 6
x y x z y z

m n
es verdadera pues + 2, m, n R+ . 2
n m
umero entero 2, pruebe que
87. Sea n un n
!4
X  X
8 xi xj x2i + x2j xi .
1i<jn 1in
226 12. Soluciones

 Prueba. Recordar

(x1 + x2 + + xn )2 = (x21 + x22 + + x2n ) + 2 (x1 x2 + + xn1 xn )


X X
= x2i + 2 xi xj ,
1in 1i<jn

luego, tenemos que

!4 !2 2 !2
X X X X
xi = xi = x2i + 2 xi xj
1in 1in 1in 1i<jn

! !
X X
4 x2i 2 xi xj
1in 1i<jn
!
X X
= 8 x2i xi xj
1in 1i<jn

 X
= 8 x21 + x22 + + x2n xi xj
1i<jn
X 
= 8 xi xj x21 + x22 + + x2n
1i<jn
X 
8 xi xj x2i + x2j .
1i<jn

umeros reales positivos y S = a1 + a2 + + an , pruebe


88. Sean a1 , a2 , . . . , an n
que

S S S n2
+ ++ .
S a1 S a2 S an n1
227

 Prueba. Como MA MH, tenemos que

(S a1 ) + (S a2 ) + + (S an ) n

n 1 1 1
+ ++
S a1 S a2 S an

 
1 1 1
nS a1 + a2 + + an + ++ n2
| {z } S a1 S a2 S an
S
 
1 1 1
(n 1)S + ++ n2
S a1 S a2 S an
S S S n2
+ ++ .
S a1 S a2 S an n1

89. Sean a, b, c n
umeros reales no negativos tales que a + b + c = 3, pruebe que


a 1 + b3 + b 1 + c3 + c 1 + a3 5.

 Prueba. Teniendo presente que

p
a 1 + b3 = a (1 + b) (1 b + b2 )

La desigualdad es equivalente a:

p p p
a (1 + b) (1 b + b2 ) + b (1 + c) (1 c + c2 ) + c (1 + a) (1 a + a2 ) 5

pero
 
p (1 + b) + (1 b + b2 )
a (1 + b) (1 b + b2 ) a
2
p 1
a (1 + b) (1 b + b2 ) (a) (2 + b2 )
2
228 12. Soluciones

luego basta demostrar que

1 1 1
a (2 + b2 ) + b (2 + c2 ) + c (2 + a2 ) 5
2 2 2
2(a b + }c) + ab + bc2 + ca2 10
| +{z
2

6 + ab2 + bc2 + ca2 10


ab2 + bc2 + ca2 4.

Supongamos sin perdida de generalidad que

cba
a(b a)(b c) 0
ab2 + a2 c abc + a2 b

entonces es suficiente probar

abc + a2 b + bc2 4
b (ac + a2 + c2 ) 4

Pero

b (ac + a2 + c2 ) b(a + c)2


  
a+c a+c
= 4b
2 2
 3
a+b+c
4 =4
3

la igualdad ocurre cuando c = 0, b = 1, a = 2 y sus correspondientes permuta-


ciones. 2

90. Sean a, b, c, d n
umeros reales no negativos, pruebe que
    
2a 2b 2c 2d
1+ 1+ 1+ 1+ 9.
b+c c+d d+a a+b
229

 Prueba. Efectuando convenientemente:


    
2a + b + c 2b + c + d 2c + d + a 2d + a + b
9
b+c c+d d+a a+b
    
2a + b + c 2b + c + d 2c + d + a 2d + a + b
9
a+b b+c c+d d+a
    
a+c b+d c+a d+b
1+ 1+ 1+ 1+ 9
a+b b+c c+d d+a

utilizando el problema 2 tenemos que

    2
a+c a+c 2(a + c)
1+ 1+ 1+
a+b c+d a+b+c+d

    2
b+d d+b 2(b + d)
1+ 1+ 1+
b+c d+a a+b+c+d
    
a+c b+d c+a d+b
1+ 1+ 1+ 1+
a+b b+c c+d d+a

 2  2
2(a + c) 2(b + d)
1+ 1+
a+b+c+d a+b+c+d

luego basta demostrar que

 2  2
2(a + c) 2(b + d)
1+ 1+ 9
a+b+c+d a+b+c+d
  
2(a + c) 2(b + d)
1+ 1+ 3.
a+b+c+d a+b+c+d
230 12. Soluciones

Haciendo a + c = m y b + d = n, tenemos equivalente a


  
2m 2n
1+ 1+ 3
m+n m+n

2(m + n) 4mn
1+ + 3
m+n (m + n)2

4mn
3+ 3
(m + n)2

4mn
0.
(m + n)2

91. Si a, b, c son n
umeros reales positivos, entonces
r r r
2a 2b 2c
+ + 3.
a+b b+c c+a

 Prueba. La desigualdad a probar es equivalente a:


v v v
u 2 u 2 u 2
u u u
t b
+t c +t a 3
1+ 1 + 1 +
a b c

b c a
Haciendo = x2 , = y 2 , = z 2 , con x, y, z positivos (xyz)2 = 1
a b c
Reemplazando tenemos:
r r r
2 2 2
+ + 3
1 + x2 1 + y2 1 + z2
231

Pero

r r 2  
2 2 2 2 2 2
1+ 1 (1 + 1 ) +
1 + x2 1 + y2 1 + x2 1 + y 2
 
1 1
= 4 +
1 + x2 1 + y 2
 
1 (xy)2
= 4 1+
(1 + x2 ) (1 + y 2 )
 
1 (xy)2
4 1+
(1 + xy)2
8 8z
= =
1 + xy z+1
r r
2 2 2 2z
+
1 + x2 1 + y2 z+1

luego basta demostrar que

r
2 2z 2
+ 3
z+1 1 + z2

Pero

(z + 1)2 2 (1 + z 2 )
2 4
2

1+z (z + 1)2
r
2 2
2
,
1+z (z + 1)

luego sera suficiente probar que


2 2z 2
+ 3
z+1 z+1
232 12. Soluciones

p
2 2z(z + 1) + 2 3 + 3z
p p
0 (3z + 1) 2 2z(z + 1) = 2z 2 2z(z + 1) + (z + 1)

0 ( 2z z + 1)2 . [8, pag 34]
2

92. Si x1 , x2 , . . . , xn n
umeros reales positivos tales que

1 1 1
+ ++ = 1,
1 + x1 1 + x2 1 + xn

pruebe que

x1 x2 xn (n 1)n .

 Prueba. De la condicion

1 1 1 1
+ ++ + =1
1 + x1 1 + x2 1 + xn1 1 + xn

tenemos

1 1 1 1 xn
+ ++ =1 =
1 + x1 1 + x2 1 + xn1 1 + xn 1 + xn

Luego aplicando MA MG:

1 1 1
+ ++ r
1 + x1 1 + x2 1 + xn1 1 1 1
n1

n1 1 + x1 1 + x2 1 + xn1
xn n1
p
1 + xn (1 + x1 ) (1 + x2 ) (1 + xn1 )
n1

xi n1
v ; i = 1, 2, . . . , n
1 + xi u Y n
u
u (1 + xj )
u
u
n1
t j=1
j 6= i
233

Multiplicando sucesivamente para todo i = 1, 2, . . . , n, tenemos

x1 x2 xn (n 1)n .

umeros reales positivos tales que a2 + b2 + c2 = 3, pruebe que


93. Sean a, b, c n

ab bc ca
+ + 3.
c a b

 Prueba. Como a, b, c > 0, la desigualdad es equivalente a


!
a2 b2 b2 c2 c2 a2 2
+ 2 + 2 +2 a b2 + c}2 9
| + {z
c2 a b
3
2 2 2 2 2 2
ab bc c a
2
+ 2 + 2 3
c a b
2 2 2 2 2 2
ab bc c a
2
+ 2 + 2 a2 + b2 + c2
c a b

Aplicando MA MG:
r
a2 b2 b2 c2 a2 b2 b2 c2
+ 2 2 2 = 2b2
c2 a c2 a
r
a2 b2 c2 a2 a2 b2 c2 a2
+ 2 2 2 = 2a2
c2 b c 2 b
r
b2 c2 c2 a2 b2 c2 c2 a2
+ 2 2 2 = 2c2
a2 b a2 b

Sumando:

a2 b2 b2 c2 c2 a2
+ 2 + 2 a2 + b2 + c2 .
c2 a b
2
234 12. Soluciones

94. Sean a, b, c n
umeros reales positivos, pruebe que

a+b+c
aa bb cc (abc) 3 .

 Prueba. Sean a b c y ln a ln b ln c, aplicando la Desigualdad de


Chebyshev
  
a ln a + b ln b + c ln c a+b+c ln a + ln b + ln c

3 3 3
 
a b c a+b+c
ln a + ln b + ln c ln abc
3

a+b+c

ln aa bb cc ln abc 3

de donde

a+b+c
a b c 3
a b c (abc) .

95. Sean a, b, c n
umeros reales positivos, pruebe que

a b c c+a a+b b+c


+ + + + .
b c a c+b a+c b+a

a b c
 Prueba. Haciendo = x, = y, = z, tenemos xyz = 1 y ademas
b c a
a+c 1 + xy 1x
= =x+
b+c 1+y 1+y

y as similarmente para los otros.


235

Reemplazando tenemos una desigualdad equivalente a la primera

1x 1y 1z
x+y+z x+ +y+ +z+
1+y 1+z 1+x
x1 y1 z1
+ + 0
y+1 z+1 x+1
(x2 1) (z + 1) + (y 2 1) (x + 1) + (z 2 1) (y + 1) 0
x2 z + y 2x + z 2 y + x2 + y 2 + z 2 (x + y + z) 3 0
x2 z + y 2x + z 2 y + x2 + y 2 + z 2 x + y + z + 3
p
como x2 z + y 2x + z 2 y 3 3
x3 y 3 z 3 = 3 y x2 + y 2 + z 2 x + y + z, pues
(x + y + z)2
x + y + z 3 y x2 + y 2 + z 2 .
3
x2 z + y 2 x + z 2 y + x2 + y 2 + z 2 x + y + z + 3 es verdadero. 2

96. Si a, b, c, d son n
umeros reales positivos, entonces

ab bc cd da
+ + + 0.
b+c c+d d+a a+b

 Prueba. Tenemos que

ab bc cd da
+ + +
b+c c+d d+a a+b
       
ab bc cd da
= +1 + +1 + +1 + +1 4
b+c c+d d+a a+b
a+c b+d c+a d+b
= + + + 4
b+c c+d d+a a+b
   
1 1 1 1
= (a + c) + + (b + d) + 4
b+c a+d c+d a+b

pero

1 1 4 1 1 4
+ ; +
b+c a+d (b + c) + (d + a) c + d a + b (c + d) + (a + b)
236 12. Soluciones

entonces
 
1 1 4(a + c)
(a + c) +
b+c a+d a+b+c+d
 
1 1 4(b + d)
(b + d) +
c+d a+b a+b+c+d

luego sumando miembro a miembro, tenemos que


   
1 1 1 1
(a + c) + + (b + d) + 4
b+c a+d c+d a+b

ab bc cd da
+ + + 0.
b+c c+d d+a a+b
2

97. Sean a, b, c n
umeros reales positivos, pruebe que

a3 b3 c3 3(ab + ac + bc)
2 2
+ 2 2
+ 2 2
.
b bc + b c ac + a a ab + b a+b+c

3(ab + ac + bc)
 Prueba. Como (a+b+c)2 3(ab+ac+bc) a+b+c .
a+b+c
Demostraremos la siguiente desigualdad que es mas fuerte

a3 b3 c3
+ + a + b + c.
b2 bc + b2 c2 ac + a2 a2 ab + b2

Luego para aplicar el Lema de Titu en el primer miembro de la desigualdad,


buscamos un equivalente adecuado, en efecto, tenemos que

a3 b3 c3
+ +
b2 bc + b2 c2 ac + a2 a2 ab + b2
a4 b4 c4
= + +
ab2 abc + ab2 bc2 abc + a2 b a2 c abc + b2 c
2
(a2 + b2 + c2 )

ab2 + ac2 + bc2 + ba2 + a2 c + b2 c 3abc
237

entonces basta demostrar que

2
(a2 + b2 + c2 )

(a + b + c) (ab2 + ac2 + bc2 + ba2 + a2 c + b2 c 3abc)

a4 + b4 + c4 + 2 (a2 b2 + a2 c2 + b2 c2 )

a2 b2 + a2 c2 + abc2 + a3 b + a3 c + acb2 3a2 bc

+ab3 + abc2 + b2 c2 + b2 a2 + bca2 + cb3 3ab2 c

+acb2 + ac3 + bc3 + a2 bc + a2 c2 + b2 c2 3abc2

a4 + b4 + c4 + a2 bc + ab2 c + abc2

ab (a2 + b2 ) + bc (b2 + c2 ) + ca (c2 + a2 )

a2 (a2 (b + c)a + bc) + b2 (b2 (c + a)b + ac) + c2 (c2 (b + a)c + ab) 0

a2 (a b)(a c) + b2 (b a)(b c) + c2 (c a)(c b) 0.

98. Si a, b, c, d son n
umeros reales positivos, entonces
 2  2  2  2
a b c d
+ + + 1.
a+b b+c c+d d+a

 Prueba. La desigualdad es equivalente a:


2 2 2 2
1 1 + 1 1
+ + 1
b 1+
c d 1+
a
1+ b 1+ d
a c
238 12. Soluciones

Haciendo

b c
= x, = y,
a b
d a
= z, = w.
c d

La desigualdad se convierte en:

 2  2  2  2
1 1 1 1
+ + + 1
1+x 1+y 1+z 1+w

con xyzw = 1.
Aplicando el problema 21, tenemos que

1 1 1
2 + 2 ; similarmente
(1 + x) (1 + y) 1 + xy
1 1 1 xy xy
2 + 2 = =
(1 + z) (1 + w) 1 + zw xy + xyzw xy + 1

sumando miembro a miembro, tenemos que

1 1 1 1 1 + xy
2 + 2 + 2 + 2 =1
(1 + x) (1 + y) (1 + z) (1 + w) xy + 1

1 1 1 1
2 + 2 + 2 + 1.
(1 + x) (1 + y) (1 + z) (1 + w)2

99. Si a, b, c son n
umeros reales no negativos, entonces

  
3 1 a + a2 1 b + b2 1 c + c2 1 + abc + a2 b2 c2 .
239

 Prueba. Desarrollar todo el primer miembro es muy tedioso y aprovechan-


do la forma de la expresion, efectuaremos los dos primeros factores, veamos

2 (1 a + a2 ) (1 b + b2 )

= 2 2b + 2b2 2a + 2ab 2ab2 + 2a2 2a2 b + 2a2 b2

= 1 + a2 b2 + (a2 2ab + b2 ) + a2 + b2 + a2 b2 + 1 2a2 b 2ab2 2a 2b

+4ab

= 1 + a2 b2 + (a b)2 + (a 1)2 + b2 (1 2a + a2 ) 2b (a2 2a + 1)

= 1 + a2 b2 + (a b)2 + (a 1)2 (1 + b2 2b)

= 1 + a2 b2 + (a b)2 + (a 1)2 (b 1)2

2 (1 a + a2 ) (1 b + b2 ) 1 + a2 b2
1
(1 a + a2 ) (1 b + b2 ) (1 + a2 b2 )
2
3
3 (1 a + a2 ) (1 b + b2 ) (1 c + c2 ) (1 + a2 b2 ) (1 c + c2 )
2

Luego basta demostrar que

3  
1 + a2 b2 1 c + c2 1 + abc + a2 b2 c2
2

en efecto, transformando a una desigualdad cuadratica en c, tenemos:

3 (1 + a2 b2 ) c2 + 3 (1 + a2 b2 ) (1 c) 2 + 2abc + 2a2 b2 c2

(3 + 3a2 b2 2a2 b2 ) c2 (3a2 b2 + 2ab + 3) c + (3 + 3a2 b2 2) 0



3 + a2 b2 c2 (3a2 b2 + 2ab + 3) c + (1 + 3a2 b2 ) 0
| {z }
(+)
240 12. Soluciones

calculando su discriminante

2
= (3a2 b2 + 2ab + 3) 4 (3 + a2 b2 ) (1 + 3a2 b2 )

= 9a4 b4 + 4a2 b2 + 9 + 12a3 b3 + 18a2 b2 + 12ab 4 (3 + 10a2 b2 + 3a4 b4 )

= 3a4 b4 + 12a3 b3 18a2 b2 + 12ab 3

= 3 [a4 b4 + 4a3 b3 6a2 b2 + 4ab 1]

= 3[ab 1]4 0

Como 0, entonces la desigualdad se cumple para todo c, a, b R, en


particular para a, b, c R+
0.

La igualdad ocurre si y solo si a = b = c = 1. 2

100. Sean a, b, c son n


umeros reales no negativos, pruebe que

a2 + b2 + c2 + 2abc + 1 2(ab + ac + bc).

 Prueba. La desigualdad es equivalente a:

a2 + b2 + c2 + 2abc + 1 2(ab + ac + bc) 0


a2 + b2 + c2 + 2abc + 1 2ab 2ac 2bc 0
a2 2ab + b2 + (c2 2c + 1) + 2c 2ac + 2abc 0
(a b)2 + (c 1)2 + 2c(1 (a + b) + ab) 0
(a b)2 + (c 1)2 + 2c(1 a)(1 b) 0 (12.18)

es importante observar que tambien podramos haber agrupado de de las si-


guientes maneras

(a 1)2 + (b c)2 + 2a(1 b)(1 c) 0 (12.19)


(b 1)2 + (a c)2 + 2b(1 a)(1 c) 0 (12.20)
241

umeros (1 a), (1 b), (1 c), tenemos que al


Aplicando el problema 3 a los n
menos una de las siguientes desigualdades es verdadera

(1 a)(1 b) 0
(1 b)(1 c) 0
(1 c)(1 a) 0.

Sin perdida de generalidad supongamos que (1 a)(1 b) 0, luego

(a b)2 + (c 1)2 + 2c(1 a)(1 b) 0.

umeros reales positivos tales que abc 1, pruebe que


101. Sean a, b, c n

a3 + b3 + c3 ab + ac + bc.

 Prueba. Podemos asumir a b c, a2 b2 c2


Aplicando Chebyshev
  2 
a a2 + b b2 + c c2 a+b+c a + b2 + c2

3 3 3

3
a3 + b3 + c3 abc (ab + ac + bc)
1(ab + ac + bc)

a3 + b3 + c3 ab + ac + bc.

102. Sean a, b, c n
umeros reales positivos tales que a + b + c = 1, pruebe que


ab + c + bc + a + ca + b 1 + ab + bc + ca.
242 12. Soluciones

 Prueba. Como

a + b 2 ab

a + b + c 2 ab + c

1 2 ab + c

c 2 abc + c2

ab + c ab + 2 abc + c2

ab + c ( ab + c)2

ab + c ab + c; similarmente

bc + a bc + a

ca + b ca + b

sumando miembro a miembro tenemos:



ab + c + bc + a + ca + b a + b
| {z }+ c + ab + bc + ca
1


ab + c + bc + a + ca + b 1 + ab + bc + ca.

103. Sean a, b numeros reales positivos tales que a + b 1, halle el mnimo valor
de f (a, b) = ab + (ab)1 .

1
Solucion. Como a, b > 0 entonces ab + 2 y decimos que el mnimo de
ab
f es 2, pero veamos cuando ocurre este mnimo.
1
El mnimo ocurre si ab = =1
ab
entonces

a+b 1
1= ab , contradiccion!
2 2

esto significa que el mnimo de f no es 2.


243

Haciendo

1 1 1 1
h= h=  2  2 = 4
ab ab a+b 1
2 2

entonces el problema se transforma en:


1
Si h 4, halle el mnimo de h + .
h
En estos casos se recomienda el siguiente procedimiento:
Tomamos h = 4, luego buscamos un parametro adecuado

h 4
= 4 1
= = 16
1 1 4
=
h 4

Luego
  r
1 h 1 15h h 1 15h
h+ = + + 2 +
h 16 h 16 16 h 16
2 15h 2 15 4 17
= + + =
4 16 4 16 4

de donde

1 17
h+
h 4

entonces

1 17
ab +
ab 4

17 1
el mnimo es y ocurre cuando a = b = c = .
4 2
2
244 12. Soluciones

3
104. Sean a, b, c numeros reales positivos tales que a + b + c , halle el mnimo
r r r 2
1 1 1
de f (a, b, c) = a2 + 2 + b2 + 2 + c2 + 2 .
b c a
Soluci
on. Aplicando MA MG:
sr r r
3 1 1 1
f (a, b, c) 3 a2 + 2 b2 + 2 c2 +
b c a2
s   
1 1 1
= 3 6
a2 + 2 b2 + 2 c2 + 2
b c a
s r r r
6 1 1 1
3 2 a2 2 2 b2 2 2 c2 2
b c a

= 3 2


entonces f (a, b, c) 3 2, el mnimo de f es 3 2 y ocurre cuando

1 1 1
a=b=c= = = = 1,
a b c

3
entonces a + b + c = 3, pero a + b + c contradiccion!
2

Esto significa que el mnimo no es 3 2.
Analizamos la expresion y aprovechando la forma simetrica estimamos que el
1
mnimo de f (a, b, c) ocurre cuando a = b = c = , luego como en el problema
2
anterior hacemos

1

a2 = b2 = c2 =
1 4
a=b=c=
2
1 = 1 = 1 = 4

a2 b2 c2
1 4
= = 16
4
245

luego tenemos que

v v
u 1 1 u
u
f (a, b, c) = ua2 + + + + ub2 + 1 + + 1
2 2 u 2 2
t |16b {z 16b } t |16c {z 16c }
16 sumandos 16 sumandos

v
u 1 1
+u
uc2 + 2
++ 2
t |16a {z 16a }
16 sumandos

s r s r s r
2 2
17 a 17 b 17 c2
17 + 17 + 17
1616 b32 1616 c32 1616 a32
"r r r #
a b c
= 17 17 8 16
+ 17 8 16
+ 17 +
16 b 16 c 168 a16
sr
r
3 17 abc 1
3 17 24 16 16 16
= 3 17 17
16 a b c 16 (abc)5
8


3 17 3 17 3 17
= 17
p = 17p = 17p
168 (abc)5 217 215 (abc)5 2 ((2a)(2b)(2c))5

3 17 3 17
r ;
17 2a + 2b + 2c 5 2
2 ( )
3

1
como vemos para a = b = c = ocurre el mnimo de f (a, b, c) y dicho mnimo
2
3 17
es . 2
2
umeros reales tales que a 2, b 6 y c 12, halle el maximo
105. Sean a, b, c n
valor de

bc a 2 + ca 3 b 6 + ab 4 c 12
f (a, b, c) = .
abc
246 12. Soluciones

Soluci
on.

bc p bc (a 2) + 2 abc
bc a 2 = (a 2) 2 =
2 2 2 2 2
ca 3p ca (b 6) + 3 + 3 abc
ca 3 b 6 = (b 6) 9 =
3
9 3
9 3 339
ab p ab (c 12) + 4 + 4 + 4 abc
ab 4 c 12 = 4
(c 12) 64 =
4
64 4
64 4 4 4 64
abc
=
8 2

entonces
 
1 abc abc abc 5 1
f (a, b, c) + + = +
abc 2 2 339 8 2 8 2 339

el mnimo ocurre para

a 2 = 2, b 6 = 3, c 12 = 4
a = 4, b = 9, c = 16

5 1
max f (a, b, c) = + .
8 2 339
2

a3 b3 c3
106. Sean a, b, c n
umeros reales positivos, pruebe que + + ab + bc + ca.
b c a
 Prueba. Aplicando MA MG:
r
a3 b3 3 a
3
b3
+ + bc 3 bc = 3ab
b c b c
r
b3 c3 3 b
3
c3
+ + ca 3 ca = 3bc
c a c a
r
3 3 3
c a 3 c a3
+ + ab 3 ab = 3ca
a b a b
247

sumando miembro a miembro:


 
a3 b3 c3
2 + + + bc + ca + ab 3ab + 3bc + 3ca
b c a

a3 b3 c3
de donde + + ab + bc + ca.
b c a
La igualdad ocurre si y solo si a = b = c. 2

107. Sean a, b, c, d n
umeros reales positivos, halle el mnimo valor de la funcion

a b c d
f (a, b, c, d) = + + +
b+c+d c+d+a d+a+b a+b+c
b+c+d c+d+a d+a+b a+b+c
+ + + + .
a b c d

Soluci
on. Si aplicamos MA MG, tenemos que

q
a b c d b+c+d c+d+a d+a+b a+b+c
f (a, b, c, d) 8 8 b+c+d c+d+a
d+a+b
a+b+c
a
b
c
d

f (a, b, c, d) 8

mn f = 8.
Cuando ocurre tal mnimo?


a= b+c+d




b = c + d + a
mn f = 8

c = d+a+b




d = a + b + c

sumando tenemos

|a + b {z
+ c + d} = 3(a
| + b {z
+ c + d})
(+) (+)

1 = 3 imposible.
248 12. Soluciones

Luego el mnimo de f no es 8.
Sean a = b = c = d > 0

a b c d 1
= = = =
b+c+d c+d+a d+a+b a+b+c 3
b+c+d c+d+a d+a+b a+b+c 3
= = = =
a b c d

1 3
igualando = = 9.
3
Esto significa que podemos agrupar de esta manera:

a b c d b+c+d c+d+a
+ + + + +
b+c+d c+d+a d+a+b a+b+c 9a 9b
d+a+b a+b+c
+ +
9c 9d
 
8 b+c+d c+d+a d+a+b a+b+c
+ + + +
9 a b c d
q
a b c d
8 8 b+c+d c+d+a d+a+b a+b+c b+c+d
9a
c+d+a
9b
d+a+b
9c
a+b+c
9d
 
8 b c d c d a d a b a b c
+ + + + + + + + + + + +
9 a a a b b b c c c d d d
r
8 8 b c d c d a d a b a b c
+ 12 12
3 9 a a a b b b c c c d d d
8 32 40
= + =
3 3 3

40
fmn = y ocurre cuando a = b = c = d > 0.
3
2

108. Halle el maximo valor de

a3 (b + c + d) + b3 (c + d + a) + c3 (d + a + b) + d3 (a + b + c)

donde a, b, c, d son n
umeros reales tales que la suma de cuadrados es 1.
249

Soluci
on.

f (a, b, c, d) = a3 (b + c + d) + b3 (c + d + a) + c3 (d + a + b)
+d3 (a + b + c)
= (a3 b + ab3 ) + (a3 c + ac3 ) + (a3 d + ad3 )
+ (b3 c + bc3 ) + (b3 d + bd3 ) + (c3 d + cd3 )
= ab (a2 + b2 ) + ac (a2 + c2 ) + ad (a2 + d2 )
+bc (b2 + c2 ) + bd (b2 + d2 ) + cd (c2 + d2 )

Pero

(a b)4 = a4 4ab (a2 + b2 ) + 6a2 b2 + b4


a4 + b4 + 6a2 b2 (a b)4
ab (a2 + b2 ) =
4

luego

f (a, b, c, d)
3 (a2 + b2 + c2 + d2 ) + 6 (a2 b2 + a2 c2 + a2 d2 + b2 c2 + b2 d2 + c2 d2 )
=
4
(a b)4 + (a c)4 + (a d)4 + (b c)4 + (b d)4 + (c d)4

4
2
1
z }| {
2 2 2 2
3 a + b + c + d [(ab)4 +(ac)4 +(ad)4 +(bc)4 +(bd)4 +(cd)4 ]

= 4
3 1
= [(a b)4 + (a c)4 + (a d)4 + (b c)4 + (b d)4 + (c d)4 ]
4 4
3
.
4

1
La igualdad ocurre cuando a = b = c = d = . 2
2
250 12. Soluciones

109. Sean a, b, c n
umeros reales positivos, pruebe que

 
a+b+c
3 2  2 2
abc max a b , b c , c a .
3

Soluci
on. Sin perdida de generalidad podemos tomar el orden

abc

entonces debemos demostrar que

a+b+c
3 2
abc c a .
3

Definamos la funcion

a+x+c x+a+c
f (x) = 3 axc = 3 acx
3 3

donde a x c, luego

1 1 2/3
f (x) = x
3 3
2 5/3
f (x) = x ac 0, x 0
9

entonces f es concava, luego

f (x) max {f (a), f (c)}

supongamos que f (c) f (a), entonces f (a) es el maximo de f , en efecto basta


demostrar que

2
f (a) ( c a)
1 3
(2a + c) a2 c c + a 2 ac
3
251


3 1 2
a2 c + a + c 2 ac 0
3 3

3
3 a2 c + a + 2c 6 ac

Pero vemos que aplicando MA MG se tiene


q
3
a2 c +
3
a2 c +
3
a2 c + a + c + c 6
3 3
a2 c a c2
6

6
= a3 c3

ac
=

3
3 a2 c + a + 2c 6 ac

 
a+b+c
3 2  2 2
abc max a b , b c , c a .
3

La igualdad ocurre cuando a = b = c. 2

110. Si a, b, c son n
umeros reales no negativos, pruebe que
 3
3 3 b+c 3
a +b +c +2 a 3abc.
2

b+c
 Prueba. Si a 0, entonces la desigualdad se cumple trivialmente
2
3 3 3
pues a + b + c 3abc.
b+c
Si a < 0 definimos
2
 3
3 3 3 b+c
f (a, b, c) = a + b + c 3abc + 2 a
2

Haciendo b = a + 2p c = a + 2q, tenemos

 3
3 3 3 2a + 2p + 2q
f = a + (a + p) + (a + q) 3a(a + p)(a + q) + 2 a
2
252 12. Soluciones

efectuando

f = 12a (p2 pa + q 2 ) + 6(p + q)(p q)2 6(p + q)(p q)2


f 6(p + q)(p q)2
  2
b+c bc
f (a, b, c) 6 a
2 2

3
b + c a (b c)2 0
f (a, b, c)
2 | 2 {z }
(+)

La igualdad ocurre si y solo si (a, b, c) = (1, 1, 1) [(a, b, c) = (0, 1, 1)] y sus


correspondientes permutaciones. 2

111. Sean x1 , x2 , x3 , x4 , x5 n
umeros reales tales que x1 + x2 + x3 + x4 + x5 = 0,
pruebe que

|cos x1 | + |cos x2 | + |cos x3 | + |cos x4 | + |cos x5 | 1.

Soluci
on. Sabemos que

| sen (x + y)| mn {| cos x| + | cos y|, | sen x| + | sen y|}


| cos (x + y)| mn {| sen x| + | cos y|, | sen x| + | cos y|}

entonces
! !
5
X 5
X

cos xi | cos x1 | + sen xi

i=1 i=2

| cos x1 | + | cos x2 | + | cos (x3 + x4 + x5 )|

entonces

!
5
X

cos xi | cos x1 | + | cos x2 | + | cos x3 | + | cos x4 | + | cos x5 |

i=1

| cos (0)| = 1 | cos x1 | + | cos x2 | + | cos x3 | + | cos x4 | + | cos x5 |


253

|cos x1 | + |cos x2 | + |cos x3 | + |cos x4 | + |cos x5 | 1.

112. Sean a, b, c n
umeros reales diferentes, pruebe que

a2 b2 c2
+ + 2.
(b c)2 (c a)2 (a b)2

 Prueba. Aplicando el desarrollo de un trinomio al cuadrado.

 2
a b c
+ +
bc ca ab
 2  2  2
a b c
= + +
bc ca ab

ab ac bc

+2 + +
(b c)(c a) (b c)(a b) (c a)(a b)
| {z }
(1)
 2
a b c
+ + +2
bc ca ab
 2  2  2
a b c
= + +
bc ca ab

Como

 2
a b c
+ + +22
bc ca ab

entonces

a2 b2 c2
+ + 2.
(b c)2 (c a)2 (a b)2
254 12. Soluciones

La igualdad ocurre si y solo si

a b c
+ + = 0.
bc ca ab
2

113. (IMO 1960). Resuelva

4x2
2 < 2x + 9.
1 1 + 2x

Soluci
on.

i) Calculamos el conjunto de restricciones:



1 + 2x 0 1 1 + 2x 6= 0
x 1/2 x 6= 0;

luego tenemos el conjunto de n


umeros reales a donde pertenece el con-
junto solucion

U = [1/2, +i {0}.

ii) Racionalizando el denominador:

2
4x2 1 + 1 + 2x
2 2 < 2x + 9
1 1 + 2x 1 + 1 + 2x
2
4x2 1 + 1 + 2x
< 2x + 9
(2x)2
2
4x2 1 + 1 + 2x
< 2x + 9
4x2

1 + 2 1 + 2x + 1 + 2x < 2x + 9
255


2 1 + 2x < 7
4(1 + 2x) < 49
8x < 45
45
x< ,
8
45
de donde x h, i.
8
Luego

45 45
C.S. = h, i U = [1/2, i {0}.
8 8
2

114. (IMO 1964). Si a, b, c son los lados de un triangulo, pruebe que

a2 (b + c a) + b2 (a + c b) + c2 (a + b c) 3abc.

 Prueba. La desigualdad es equivalente a:

a2 b + a2 c + b2 a + b2 c + c2 a + c2 b 3abc + a3 + b3 + c3
0 a3 a2 (b + c) + abc + b3 b2 (a + c) + abc + c3 c2 (a + b) + abc
0 a(a b)(a c) + b(b a)(b c) + c(c a)(c b)
a(a b)(a c) + b(b a)(b c) + c(c a)(c b) 0,

la cual es verdadera, pues es la desigualdad de Schur. 2

115. (IMO 1975). Considere las sucesiones (x1 , x2 , . . . , xn , . . .) y (y1 , y2, . . . , yn , . . .)


tales que x1 x2 xn , y1 y2 yn , y (z1 , z2 , . . . , zn ) es una
permutacion de (y1 , y2 , . . . , yn ), pruebe que

(x1 y1 )2 + + (xn yn )2 (x1 z1 )2 + + (xn zn )2 .


256 12. Soluciones

 Prueba. La desigualdad es equivalente a:

x21 + + x2n + y12 + + yn2 2(x1 y1 + + xn yn )


x21 + + x2n + z12 + + zn2 2(x1 z1 + + xn zn )
x1 z1 + + xn zn x1 y1 + + xn yn ()

pues y12 + + yn2 = z12 + + zn2 .


La desigualdad () es verdadera, pues ya fue demostrada en el captulo de
Reordenamientos. 2

116. (IMO 1978). Sean x1 , x2 , . . . , xn n


umeros enteros diferentes, pruebe que

x1 x2 xn 1 1 1
2
+ 2 ++ 2 + ++ .
1 2 n 1 2 n

 Prueba. Sea (a1 , a2 , . . . , an ) unapermutacion de (x1 , x2 , . . . , xn ) con a1


1 1 1
a2 an y sea (b1 , b2 , . . . , bn ) = 2
, 2
, . . . , 2 , donde se observa
n (n 1) 1
1
que bi = , i = 1, 2, . . . , n.
(n + 1 i)2
Consideremos la permutacion (c1 , c2 , . . . , cn ) de (a1 , a2 , . . . , an ) donde ci =
xn+1i , i = 1, 2, . . . , n.
Luego

x1 x2 xn
+ + + c1 b1 + c2 b2 + + cn bn
12 22 n2
an b1 + an1 b2 + + a1 bn
= a1 bn + a2 bn1 + + an b1
a1 a1 an
= 2 + 2 ++ 2
1 2 n

como a1 1, a2 2, . . . , an n entonces

a1 a1 an 1 2 n
+ + + + + +
12 22 n2 12 22 n2
1 1 1
= + ++
1 2 n
257

de donde se obtiene

x1 x2 xn 1 1 1
2
+ 2 ++ 2 + ++ .
1 2 n 1 2 n
2

117. (IMO 1997). Sean x1 , x2 , . . . , xn n


umeros reales que satisfacen

n+1
|x1 + x2 + + xn | = 1 y |xi | , para todo i = 1, 2, . . . , n,
2

pruebe que existe la permutacion (y1 , y2 , . . . , yn ) de (x1 , x2 , . . . , xn ) tal que

n+1
|y1 + 2y2 + + nyn | .
2

 Prueba. Sin perdida de generalidad tomamos x1 x2 xn y como


1 < 2 < < n entonces por el teorema de reordenamientos se tiene que

1x + 2x2 + + nxn nx1 + (n 1)x2 + + 1xn


| 1 {z } | {z }
M N

M + N = (n + 1) (x1 + x2 + + xn )
|M + N| = (n + 1) |x1 + x2 + + xn | = (n + 1) 1 = n + 1
|M + N| = n + 1

de donde M + N = n + 1 o M + N = (n + 1).

n+1
i) Si M + N = n + 1 N < <M y
2
(n + 1)
ii) Si M + N = (n + 1) N < < M.
2
Graficamente

n+1
 n+1
N 2 M N 2 M
258 12. Soluciones

 
(n + 1) n + 1
Si tomaramos el primer caso, entonces N , y en el segundo
  2 2
(n + 1) n + 1
caso M , , es decir M o N se encuentran en el intervalo
 2 2
(n + 1) n + 1 (n + 1) n+1
, y ademas N < M.
2 2 2 2
Sean (y1 , y2 , . . . , yn ) una permutacion de (x1 , x2 , . . . , xn ) tal que P = 1y1 +
2y2 + + nyn .
(n + 1)
Tomamos el maximo valor de P y tambien tomamos i tal que
2
y1 y2 yn y yi > yi+1 y consideramos

Q = 1y1 + 2y2 + + (i 1)yi1 + (i + 1)yi + iyi+1 + + nyn

entonces

Q P = iyi+1 + (i + 1)yi (iyi + (i + 1)yi+1 ) = yi yi+1 > 0

Q P > 0 de donde Q > P


n+1 n+1
y desde que |yi | , |yi+1 | , tenemos que
2 2
 
n+1 n+1
yi yi+1 +
2 2
yi yi+1 n + 1
QP n+1
(n + 1) n+1
QP +n+1 +n+1=
2 2
n+1
Q ,
2
   
n+1 n+1 n+1
de otro lado Q , entonces Q .
2 2 2
n+1
|Q| .
2
2
259

118. (IMO 2004). Sea n un n umero entero positivo 3 y sean t1 , t2 , . . . , tn n


umeros
reales positivos tales que
 
2 1 1 1
n + 1 > (t1 + t2 + + tn ) + ++ ,
t1 t2 tn

pruebe que ti , tj , tk son los lados de un triangulo para todo i, j, k con 1 i <
j < k n.

 Prueba. Basta demostrar sin perdida de generalidad que t1 < t2 + t3 por


la simetra de la desigualdad.
Desarrollamos:
 
1 1 1
(t1 + t2 + + tn ) + ++
t1 t2 tn
X  ti tj  X  ti tj 
= n+ + =n+ +
i6=j
tj ti 1i<jn
tj ti

X  
t1 t2 t1 t3 ti tj
= n+ + + + + +
t2 t1 t3 t1 tj ti
1i<jn
(i, j) 6= (1, 2)
(i, j) 6= (1, 3)
  X  
1 1 t2 + t3 ti tj
= n + t1 + + + +
t2 t3 t1 tj ti
1i<jn
(i, j) 6= (1, 2)
(i, j) 6= (1, 3)
 
2t1 2 t2 t3 n(n 1)
= n+ + +2 2
t2 t3 t1 2

t1
Haciendo = a, tenemos
t2 t3
 
1 1 1 2
(t1 + t2 + + tn ) + ++ n + 2a + + n2 n 4
t1 t2 tn a
260 12. Soluciones

como
 
2 1 1 1
n + 1 > (t1 + t2 + + tn ) + ++
t1 t2 tn

por transitividad

2
n2 + 1 > n + 2a + + n2 n 4
a
2
2a + 5 < 0; a > 0
a
2a2 5a + 2 < 0
(2a 1)(a 2) < 0
1
<a<2
2
1 t1
< <2
2 t2 t3

t1 < 2 t2 t3

Como 2 t2 t3 t2 + t3 ,

t1 < t2 + t3 .

119. (IMO 2000). Sean a, b, c n


umeros reales positivos tales que abc = 1, pruebe
que
   
1 1 1
a1+ b1+ c1+ 1.
b c a

 Prueba. La desigualdad es equivalente a:

(ab b + 1)(bc c + 1)(ca a + 1) abc


(ab b + 1)(bc c + 1)(ca a + 1) 1 (12.21)
261

Como abc = 1, podemos hacer el cambio:

x y z
a= , b = , c = , x, y, z > 0
y z x

en efecto la desigualdad (12.21) se transforma en:

x y  y z  z x 
+1 +1 +1 1
z z x x y y
(x y + z)(y z + x)(z x + y) xyz (12.22)

Haciendo u = x y + z, v = y z + x, w = z x + y

u + v = 2x, u + w = 2z, v + w = 2y

en efecto la desigualdad (12.22) es equivalente a:

   
u+v v+w u+w
uvw
2 2 2
8u v w (u + v)(v + w)(u + w) (12.23)

Si x, y, z son lados de un triangulo, entonces u, v, w son positivos, entonces la


desigualdad 8u v w (u + v)(v + w)(u + w) se verifica.
En seguida veamos cuando

i) Uno de ellos es negativo.


Supongamos que u < 0
entonces

8u v w (u + v)(v + w)(u + w)

se verifica pues

8u v w < 0 (u + v)(v + w)(u + w) > 0

es decir el miembro de la izquierda es negativo y el de la derecha es


positivo, verificando as la desigualdad (12.23).
262 12. Soluciones

ii) Dos de ellos son negativos.


Supongamos que u < 0 y v < 0
en efecto sabemos que u + v = 2x < 0, esto contradice pues x > 0.
iii) Los tres son negativos.
Si u, v, w < 0 entonces

u + v = 2x < 0
u + w = 2z < 0
v + w = 2y < 0

esto nuevamente se contradice, no es posible este caso.

120. Pruebe que

n
X nX
|i j| |xi xj | = |xi xj |
i,j=1
2 i,j

si x1 x2 xn .

 Prueba.

n
X X
|i j| |xi xj | = |(i j)(xi xj )|
i,j=1 i,j

n

Xn X X n Xn

= (i j)(x i x )
j = (i j)(xi xj );
| {z }
i=1 j=1 i=1 j=1
(+)

(pues x1 x2 xn )
n X
X n
= (ixi ixj jxi + jxj )
i=1 j=1
n X
X n n X
X n n X
X n n X
X n
= ixi ixj jxi + jxj
i=1 j=1 i=1 j=1 i=1 j=1 i=1 j=1
263

n
X n
X n
X n
X n
X n
X
= nixi i xj xi j+ njxj
i=1 i=1 j=1 i=1 j=1 j=1
n
X n
X n
X n
n(n + 1) n(n + 1) X
= nixi xj xi + nixi
i=1
2 j=1 i=1
2 i=1
| {z }
n
X n
X
= 2nixi n(n + 1) xi
i=1 i=1
Xn
= n (2i (n + 1)) xi
i=1

efectuando la sumatoria

n
nX X
|xi xj | = n |xi xj |
2 i,j=1 i<j
X
=n (xi xj )
1i<jn

= n [(xn x1 ) + (xn x2 ) + + (xn xn1 )]


+n [(xn1 x1 ) + (xn1 x2 ) + + (xn1 xn2 )]
+
+n [(x3 x1 ) + (x3 x2 )]
+n [(x2 x1 )]
= n [(n 1)xn + (n 3)xn1 + (n 5)xn2 + + (1 n)x1 ]
n
X
=n (2i (n + 1)) xi ,
i=1

n
X nX
|i j| |xi xj | = |xi xj |.
i,j=1
2 i,j

2
264 12. Soluciones

121. Pruebe que

n
X n2 (n2 1)
(i j)2 = .
i,j=1
6

 Prueba.

n
X n
X 
(i j)2 = i2 2ij + j 2
i,j=1 i,j=1

n
X n
X n
X
= i2 2 ij + j2
i,j=1 i,j=1 i,j=1

X n
n X n X
X n n X
X n
= i2 2 ij + j2
i=1 j=1 i=1 j=1 i=1 j=1

n
X n
X Xn
2 n(n + 1)
= ni 2 i +n j2
i=1 i=1
2 j=1

n(n + 1)(2n + 1) n(n + 1) n(n + 1)


= n 2
6 2 2
n(n + 1)(2n + 1)
+n
6
n2 (n + 1)(2n + 1) n2 (n + 1)2
=
3 2
   
2 2n + 1 n + 1 2 4n + 2 3n 3
= n (n + 1) = n (n + 1)
3 2 6
n2 (n + 1)(n 1)
=
6
n2 (n2 1)
= .
6

2
265

122. (IMO 2003). Sea n un n umero entero 2 y los n


umeros reales x1 , x2 , . . . , xn
con x1 x2 xn , pruebe que

n
!2 n
X 2 2 X
|xi xj | n 1 (xi xj )2 .
i,j=1
3 i,j=1

 Prueba. Para probar esta desigualdad utilizaremos resultados anteriores


como:
X nX
a) |i j| |xi xj | = |xi xj |,
i,j
2 i,j
X n2 (n2 1)
b) (i j)2 = ,
i,j
6

aplicamos la desigualdad de Cauchy-Schwarz

!2
X X X
|i j| |xi xj | (i j)2 (xi xj )2
i,j i,j i,j
| {z }
!2
nX n2 (n2 1) X
|xi xj | (xi xj )2
2 i,j 6 i,j

n2 P 2 n2 (n2 1) X
i,j |xi xj | (xi xj )2
4 6 i,j
P 2 2 2 X
i,j |xi xj | (n 1) (xi xj )2 .
3 i,j

123. (IMO 2006). Determine el menor valor de M para que la desigualdad


ab(a2 b2 ) + bc(b2 c2 ) + ca(c2 a2 ) M a2 + b2 + c2 2

se verifique para cualesquiera a, b, c n


umeros reales.
266 12. Soluciones

 Prueba. Factorizamos el primer miembro de la desigualdad


ab(a2 b2 ) + bc(b2 c2 ) + ca(c2 a2 ) = |(a b)(b c)(c a)(a + b + c)|

Haciendo

a b = x, b c = y, c a = z x + y + z = 0

y ademas haciendo = a + b + c

2 = a2 + b2 + c2 + 2ab + 2ac + 2bc,


x2 = a2 + b2 2ab,
y 2 = b2 + c2 2bc,
z 2 = c2 + a2 2ca,

sumando tenemos:

2 + x2 + y 2 + z 2 = 3 (a2 + b2 + c2 )
2
2 2 2 2 (2 + x2 + y 2 + z 2 )
(a + b + c ) = .
9

Reemplazando tenemos una desigualdad equivalente


!
2
(2 + x2 + y 2 + z 2 )
|xyz| M
9
2
9|xyz| M (2 + x2 + y 2 + z 2 ) ,

como

x + y + z = 0 z = x + y
|z| = |x + y|,
267

entonces

9|xyz| = 9|xy(x + y)|,

luego tenemos la desigualdad


2
9|xy(x + y)| M 2 + x2 + y 2 + z 2 ,

(x + y)2
como xy
4
9
9|xy(x + y)| |||x + y|3.
4

9 2
Luego debemos encontrar la relacion |||(x + y)3| M (2 + x2 + y 2 + z 2 ) .
4
Aplicando MA MG:

22 + (x + y)2 + (x + y)2 + (x + y)2 p


4 22(x + y)6
4
 2 4
2 + 3(x + y)2
22 (x + y)6
4
2
(22 + 3(x + y)2)
2|||(x + y)3 |
42
 2 2
2 + 3(x + y)2
4 2|||(x + y)3 |
2
 2
3
2 + (x + y)2 4 2|||(x + y)3 |
2
 2
9 3 9 2 3 2
|||(x + y) | + (x + y)
4 16 2 2
 2
9 2 3
= 2 + (x + y)2 .
32 2
268 12. Soluciones

Pero

(x + y)2 2(x2 + y 2)
3(x + y)2 2(x2 + y 2) + 2 (x + y)2
| {z }
3(x + y)2 2(x2 + y 2) + 2z 2
3
(x + y)2 (x2 + y 2 + z 2 )
2
3
2 + (x + y)2 2 + x2 + y 2 + z 2
2
 2
2 3 2 2
+ (x + y) (2 + x2 + y 2 + z 2 )
2
 2
9 2 2 3 2 9 2 2 2
+ (x + y) ( + x2 + y 2 + z 2 )
32 2 32

9 9 2 2 2
|||(x + y)3 | ( + x2 + y 2 + z 2 ) ,
4 32

9 2
entonces M
32

9 2
de donde el menor valor de M es y esto ocurre si y solo si (a, b, c) =
  32
3 3
1 + , 1, 1 . 2
2 2

124. (IMO 1983). Sean a, b, c los lados de un triangulo, pruebe que

a2 b(a b) + b2 c(b c) + c2 a(c a) 0.

 Prueba. Haciendo


a + b c = 2y


a = x + y, b = y + z, c = z + x a + c b = 2x



b + c a = 2z
269

como a, b, c son lados de un triangulo entonces x, y, z > 0.


Reemplazando en la desigualdad, tenemos otra equivalente:

(x + y)2(y + z)(x z) + (y + z)2 (z + x)(y x) + (z + x)2 (x + y)(z y) 0

efectuando tenemos:

zx3 + xy 3 + yz 3 xyz 2 + yzx2 + zxy 2


zx3 + xy 3 + yz 3 xyz(z + x + y)
x2 y 2 z 2
+ + x + y + z.
y z x

Para probar este desigualdad utilizaremos la desigualdad de Titu

x2 y 2 z 2 (x + y + z)2
+ + =x+y+z
y z x y+z+x

de donde

x2 y 2 z 2
+ + x + y + z.
y z x
2

125. Sean a, b, c, d, e n
umeros reales positivos tales que abcde = 1, pruebe que

a + abc b + bcd c + cde d + dea


+ + +
1 + ab + abcd 1 + bc + bcde 1 + cd + cdea 1 + de + deab
e + eab 10
+ .
1 + ea + eabc 3

 Prueba. Como abcde = 1, entonces podemos hacer el cambio:

x y z w t
a= , b= , c= , d= , e=
y z w t x
270 12. Soluciones

en efecto

x x 1 1
+ +
a + abc y w y w
= x x =
1 + ab + abcd 1+ + 1 1 1
z t + +
x z t

tambien es conveniente otro cambio de variable, es decir

1 1 1 1 1
= m, = n, = p, = q, =r
x y z w t

luego

a + abc n+q
= ; similarmente
1 + ab + abcd m+p+r
b + bcd p+r
= ;
1 + bc + bcde n+q+t
c + cde q+m
= ;
1 + cd + cdea p+n+r
d + dea r+n
= ;
1 + de + deab m+p+q
e + eab m+p
= ;
1 + ea + eabc n+q+r

entonces basta demostrar la desigualdad

n+q p+r q+m r+n m+p 10


+ + + +
m+p+r n+q+t p+n+r m+p+q n+q+r 3
     
n+q p+r q+m
+1 + +1 + +1
m+p+r n+q+t p+n+r
   
r+n m+p 10
+ +1 + +1 +5
m+p+q n+q+r 3
271

m+n+p+q+r m+n+p+q+r m+n+p+q+r


+ +
m+p+r n+q+t p+n+r
m+n+p+q+r m+n+p+q+r 25
+ +
m+p+q n+q+r 3
3(m + n + p + q + r)
 
1 1 1 1 1
+ + + + 25
m+p+r n+q+t p+n+r m+p+q n+q+r

Haciendo

m + p + r = a , n + q + t = b , p + n + r = c ,
m + p + q = d , n + q + r = e ,

la desigualdad es equivalente a:
 
1 1 1 1 1
(a + b + c + d + e ) + + + + 25
a b c d e
a + b + c + d + e 5
,
5 1 1 1 1 1
+ + + +
a b c d e

esta desigualdad es verdadera, pues MA MH. 2

126. Sean a, b, c que satisfacen a2 + b2 + c2 = 1, halle el maximo valor de

P = (a b)(a c)(b c)(a + b + c).

 Prueba. Como

" #2
2 2
3(a b2 + c}2 )
| + {z = [2(a b)2 + 2(a c)(b c) + (a + b + c)2 ]
1

8(a c)(b c) (2(a b)2 + (a + b + c)2 )



16 2(a c)(b c)(a b)(a + b + c)
272 12. Soluciones


32 16 2 (a c)(b c)(a b)(a + b + c)
| {z }
P
9
P .
16 2

9 2
el maximo de P es . 2
32

127. (IMO 2008).

(a) Demuestre que

x2 x2 x2
+ + 1 (12.24)
(x 1)2 (x 1)2 (x 1)2

para todos los n


umeros reales x, y, z, distintos de 1, con xyz = 1.
(b) Demuestre que existen infinitas ternas de n
umeros racionales x, y, z, dis-
tintos de 1, con xyz = 1 para los cuales la expresion (12.24) es una
igualdad.

 Prueba.

(a) Haciendo x 1 = a, y 1 = b, z 1 = c, tenemos que x = a + 1, y =


b + 1, z = c + 1 y como xyz = 1 entonces (a + 1)(b + 1)(c + 1) = 1. De
donde abc + ab + ac + bc + a + b + c = 0.
Luego la desigualdad es equivalente a

(a + 1)2 (b + 1)2 (c + 1)2


+ + 1
a2 b2 c2

con abc + ab + ac + bc + a + b + c = 0.
Efectuando tenemos

b2 c2 (a + 1)2 + a2 c2 (b + 1)2 + a2 b2 (c + 1)2 a2 b2 c2


2a2 b2 c2 + 2ab2 c2 + 2ba2 c2 + 2ca2 b2 + a2 b2 + a2 c2 + b2 c2 0
2a2 b2 c2 + 2abc(bc + ac + ab) + a2 b2 + a2 c2 + b2 c2 0. (12.25)
273

Ademas, como

abc + a + b + c = (ab + ac + bc) (12.26)

elevando al cuadrado

a2 b2 c2 + (a + b + c)2 + 2abc(a + b + c)
= a2 b2 + a2 c2 + b2 c2 + 2abc(a + b + c)
a2 b2 c2 + (a + b + c)2 = a2 b2 + a2 c2 + b2 c2 ,

reemplazando en (12.25)

2a2 b2 c2 + 2abc(abc (a + b + c)) + a2 b2 c2 + (a + b + c)2 0


a2 b2 c2 2abc(a + b + c) + (a + b + c)2 0
(a + b + c abc)2 0.

(b) La igualdad ocurre si y solo si

a + b + c abc = 0
a + b + c = abc,

reemplazando en (12.26) tenemos que

2(a + b + c) + ab + ac + bc = 0
2(x + y + z 3) + (x 1)(y 1) + (x 1)(z 1)
+(y 1)(z 1) = 0
2(x + y + z) 6 + xy (x + y) + 1 + xz (x + z) + 1
+yz (y + z) + 1 = 0
xy + xz + yz = 3
1 1
xy + + = 3, pues xyz = 1
y x
 
2 1
yx + 3 x + 1 = 0,
y
274 12. Soluciones

como x es un n
umero racional, entonces es necesario que su discriminante
sea cuadrado perfecto, en efecto
 2
1 9y 2 6y + 1
= 3 4y = 4y
y y2
4y 3 + 9y 2 6y + 1 (y 1)2 (1 4y)
= = ,
y2 y2

luego para que sea cuadrado perfecto basta que 1 4y = k 2 , con k Q,


1 k2 2(k 1)
luego y = , reemplazando en la ecuacion se obtiene x =
4 (1 + k)2
2(1 + k)
x= , y de xyz = 1, se encuentra el valor de z.
(1 k)2
Por lo tanto existen infinitas ternas como
 
2(k 1) 1 k 2 2(1 + k)
(x, y, z) = , ,
(1 + k)2 4 (1 k)2
 
2(1 + k) 1 k 2 2(k 1)
(x, y, z) = , , .
(1 k)2 4 (1 + k)2

2
BIBLIOGRAFIA

[1] Elon Lages Lima: An


alisis Real vol. 1.
IMPA.

[2] Mircea Becheanu; Bogdan Enescu: Balkan Mathematical


Olympiads 1984-2006.
GIL Publishing Home.
n; L
[3] Plam van Thua e Vi: Batdang Thuc.
Suy Ivan & Khan Pha

[4] Titu Andreescu; Dorin Andrica: 360 Problems for Mathematical


Contests.
GIL Publishing Home.

[5] Andrei Negut: Problems for the Mathematical Olympiads.


GIL Publishing Home.
Go
[6] Radmila Bulajich; Jose mez; Rogelio Valdez: Cuaderno de
Olimpiadas de Matem
atica.
Instituto de Matematicas UNAM.

[7] Titu Andreescu; Vasile Cirtoaje; Gabriel Dospinescu; Mircea Las-


cu: Old and New Inequalities.
GIL Publishing Home.

[8] Vasile Cirtoaje: Algebric Inequalities Old and New Methods.


GIL Publishing Home.

275
276 BIBLIOGRAFIA

[9] Pham Kim Hung: Secrets in Inequalities, Volumen 1 - Basic Inequal-


ities.
GIL Publishing Home.

[10] Tran Phuing: Diamonds in Mathematical Inequalities.


Hanoi Publishing House.

[11] Titu Andreescu: I Seminario de Educaci


on Matematica Iberoamer-

icana con Enfasis en Resoluci
on de Problemas (SEMI 2005).
Cartagena de Indias, Colombia, Setiembre 2005.

Potrebbero piacerti anche